You are on page 1of 218

के न्द्रीय विद्यालय सगं ठन

KENDRIYA VIDYALAYA SANGATHAN


MUMBAI REGION

CLASS X

STUDENT SUPPORT MATERIAL

ENGLISH LANGUAGE & LITERATURE


[CODE 184]

AS PER LATEST CBSE PATTERN

SESSION 2022-23
PATRONS

Mrs. SONA SETH Mr. S.V. JOGLEKAR


DEPUTY COMMISSIONER ASSISTANT COMMISSIONER
KVS MUMBAI REGION KVS MUMBAI REGION

COURSE DIRECTORS

Mr. R A PATIL (Principal, K V Bambolim Camp,Goa)

Mrs. LAKSHMI NEGI (Principal, K V Colaba No 3)

RESOURCE PERSONS-

1. Mrs. Nirmala Kanti Yadav, K V IIT Powai

2. Mrs. P Janet Asokan, K V CME Pune

3. Mrs. Vidya Thulasee, K V Bambolim Camp, Goa

4. Mrs. Jency George, K V ONGC Panvel

5. Mrs. Suvarna Saner, K V Dhule

2
CONTRIBUTORS

S. No. NAME OF THE TEACHERS Designation NAME OF THE KV

1 MS. PREETI SOJWAL TGT- ENGLISH JALGAON

2 MR. ANIL MAHAJAN TGT- ENGLISH ISP NASHIK

3 MR. U S SINGH TGT- ENGLISH ISP NASHIK

4 MR. RAVIKIRAN D PATIL TGT- ENGLISH AFS DEVLALI NASHIK

5 MRS. SHEETAL TGT- ENGLISH AFS DEVLALI NASHIK

6 MRS. PRATIBHA BHAGAT TGT- ENGLISH NO. VASCO DA GAMA GOA

7 MRS. NAMITA SINHA TGT- ENGLISH CME PUNE

8 MR. MANOJ BACHHAV TGT- ENGLISH NO. 1 DEVLALI NASHIK

9 MR. RAM AKBAL YADAV TGT- ENGLISH NO. 1 DEVLALI NASHIK

10 MR. SANGMESHWAR U N TGT- ENGLISH NO.1 DEHU ROAD

11 MRS. SARIKA S JOGDAND TGT- ENGLISH NO. 1 DEHU ROAD

12 MR. JAYESH NARKHEDE TGT- ENGLISH NO. 1 DEHU ROAD

13 MRS. BINDU ANILKUMAR TGT- ENGLISH NO.2 DEHU ROAD

14 MR. V K MUNDE TGT- ENGLISH NO. 2 DEHU ROAD

15 MRS. JYOTI NEHE TGT- ENGLISH NO. 2 DEHU ROAD

16 MRS. CHANDRAKANTA S TGT- ENGLISH NDA PUNE

17 MR. RUPESH MAKHARE TGT- ENGLISH NDA PUNE

18 MR. GANESH MALI TGT- ENGLISH OF VARANGAON

19 MRS. AISHWARYA RANA TGT- ENGLISH DIAT GIRINAGAR PUNE

20 MR. MANISH SEN TGT- ENGLISH INS LONAVALA

21 MS GARIMA PUNIA TGT- ENGLISH INS LONAVALA

22 MS MANSI SHARMA TGT- ENGLISH NO 2 VASCO DA GAMA GOA

23 MRS. SEEMA BATHAM TGT- ENGLISH OF BHUSAWAL

24 MR. RAHUL BAND TGT- ENGLISH 9 BRD PUNE

25 E J RAJULA TGT- ENGLISH 9 BRD. PUNE

26 MRS. SHALINI PUN TGT- ENGLISH 9 BRD PUNE

27 MR. SURESH SHIRSATH TGT- ENGLISH MIRC AHMEDNAGAR

28 MR KIRAN WARKAD TGT- ENGLISH NO 1 AHMEDNAGAR

29 MRS SHOBHA GUPTA TGT- ENGLISH GANESHKHIND PUNE

3
30 MRS. MAMTA PARDESHI TGT- ENGLISH GANESHKHIND PUNE

31. MR. YOGESH DHORE TGT- ENGLISH GANESHKHIND PUNE

32 MRS.SANJIVANI GAVIT TGT- ENGLISH NASHIK ROAD CAMP

33 TUSHAR S WARDE TGT- ENGLISH NASHIK ROAD CAMP

34 MRS. AKANKSHA KESTWAL TGT- ENGLISH NASHIK ROAD CAMP

35 MRS. SUMAN SINGH TGT- ENGLISH BAMBOLIM GOA

36 MRS.VIDYA THULASEE TGT- ENGLISH BAMBOLIM GOA

37 MRS. RAJANI SHUKLA TGT- ENGLISH AFS OJHAR NASHIK

38 MRS. VAISHALI KAMBLE TGT- ENGLISH AFS OJHAR NASHIK

39 MR. DARASING TGT- ENGLISH AFS OJHAR NASHIK

40 MR. VINOD MALI TGT- ENGLISH CR MANMAD

MRS. NADIYA SAYYED


41 TGT- ENGLISH 9 BRD PUNE
(Constractual teacher)

42 MR. SANTOSH KUMAR YADAV TGT- ENGLISH AMBERNATH SHIFT2

43 MR. SANDEEP RAMESH PATIL TGT- ENGLISH DHULE

44 MR. MUKESH KALE TGT- ENGLISH NAD, KARANJA

45 MRS. SARITA M MOHAN TGT- ENGLISH NAD KARANJA

46 MR. SUDHIR KUMAR GAWALI TGT- ENGLISH ONGC PANVEL

47 MRS. ALKA JAIN TGT- ENGLISH ONGC PANVEL

48. MR. PARASNATH TGT- ENGLISH NO2 AFS PUNE

MRS. SUBHANGI
49. TGT- ENGLISH NO. 2 AFS PUNE
MISHRIKOTKAR

N0.2 AFS PUNE


50. MOMIN ZULEKHA TGT- ENGLISH

51. MRS. PRIYANKA GARUD TGT- ENGLISH CRPF TALEGAON

51. MRS. RAJNI KAUR TGT- ENGLISH BEG PUNE

52. MR. VISHWAS V MIRAJE TGT- ENGLISH BEG PUNE

53 MR. LAXMAN GHODAKE TGT- ENGLISH BEG PUNE

54. MRS. ALKA TGT- ENGLISH NO1. COLABA MUMBAI

55. MRS. ALPANA GUPTA TGT- ENGLISH NO1. COLABA MUMBAI

56. MRS. ANJU BAI TGT- ENGLISH KOLIWADA

57. MR. RAHUL PAWAR TGT- ENGLISH KOLIWADA

58 NEELAM GUPTA TGT- ENGLISH KOLIWADA

59. BIDYUTLATA DASH TGT- ENGLISH KOLIWADA

4
60. NEETI PAL TGT- ENGLISH S C PUNE

61 MRS.NIRMALA MUKHERJEE TGT- ENGLISH RHE PUNE

62 MR. VISHWASRAO BHADANE TGT- ENGLISH RHE PUNE

63 SMITA SHIJU TGT- ENGLISH AFS THANE SHIFT 1

64 MANISHA S WAKODE TGT- ENGLISH OF AMBERNATH SHIFT 1

65 DIVYA GAUR TGT- ENGLISH OF AMBERNATH SHIFT 1

66 MRS. SUJATA DEBNATH TGT- ENGLISH MANKHAURD MUMBAU

67 MRS. ROSELIN KUJUR TGT- ENGLISH MANKHAURD MUMBAU

68 RASHMI BEHERA TGT- ENGLISH MANKHAURD MUMBAU

69 SAVITA NAYAK TGT- ENGLISH ARMY AREA PUNE

70 TANAJI P CHOUGALE TGT- ENGLISH ARMY AREA PUNE

71 DEVANAND KHADE TGT- ENGLISH ARMY AREA PUNE

72 ITI SHRIVASTAVA TGT- ENGLISH INS HAMLA

73 DATTATRAY SUROD TGT- ENGLISH NO.1 AFS PUNE

74 P A BHALERAO TGT- ENGLISH NO.1 AFS PUNE

75 DATTA SHAKARRAO JAGTAP TGT- ENGLISH NO.1 AFS PUNE

76 UMA SHANKAR SINGH TGT – ENGLISH KV ISV NASIK

5
ENGLISH LANGUAGE AND LITERATURE
Class X
Study Material
2022-23

INDEX
Sl. Contents

1 Syllabus

2 Reading

3 Grammar

4 Creative Writing Skills

5 First Flight

6 Footprints without Feet

7. Sample Papers

ENGLISH LANGUAGE AND LITERATURE (Code No. 184) CLASS – X (2022-23)


SECTION - WISE WEIGHTAGE

Sections

A Reading Skills (40 periods)

B Writing Skills with Grammar (40 periods)

Literature Textbooks and


C (50 periods)
Supplementary Reading Text

6
7
Section A Reading Skills 20 Marks
Reading Comprehension through Unseen Passage
I. Discursive passage of 400-450 words. (10 marks)
II. Case-based passage (with visual input- statistical data, chart etc.) of 200-250 words.
(10 marks)
(Total length of two passages to be 600-700 words) Multiple Choice Questions / Objective Type
Questions will be asked to assess inference, analysis, interpretation, evaluation and vocabulary.
Section B Grammar 10 Marks
• Tenses
• Modals
• Subject – verb concord
• Reported speech
o Commands and requests
o Statements
o Questions
o Determiners
The courses at the secondary level seek to cement high professional grasp of grammatical items
and levels of accuracy. Accurate use of spelling, punctuation and grammar in context will be
assessed through Gap Filling/ Editing/Transformation exercises. Ten out of 12 questions will
have to be attempted.
Creative Writing Skills 10 marks
This section will have short as well as long writing tasks including compositions.
I. Formal Letter based on a given situation in 100-120 words. One out of two questions is to
be answered. 5 marks
II. Writing an Analytical Paragraph (100-120 words) on a given Map / Chart / Graph / Cue/s.
One out of two questions is to be answered. 5 marks
Section C Literature 40 Marks
V. Reference to the Context
I. One extract out of two from Drama / Prose. II. One extract out of two from poetry.
5+5 = 10 Marks
Multiple Choice Questions / Objective Type Questions will be asked to assess inference, analysis,
interpretation, evaluation and vocabulary.
VI. Short & Very Long Answer Questions 30 Marks
8
I. Four out of Five Short Answer Type Questions to be answered in 40-50 words from the
book
FIRST FLIGHT 4x3=12 marks
II. Two out of Three Short Answer Type Questions to be answered in 40-50 words each from
FOOTPRINTS WITHOUT FEET. 2x3=6 marks
III. One out of two Long Answer Type Questions from FIRST FLIGHT to be answered
in about 100-120 words each to assess creativity, imagination and extrapolation
beyond the text and across the texts. This can be a passage-based question taken from a
situation/plot from the texts. 6 mark
IV. One out of two Long Answer Type Questions from FOOTPRINTS WITHOUT FEET
on theme or plot involving interpretation, extrapolation beyond the text and inference or
character sketch to be answered in about 100-120 words. 6 marks

9
SECTION A: READING (20 Marks)

Reading passage is to comprehend the unseen passages and answer the questions that follow. The
questions include both comprehension and vocabulary as well.

Reading for comprehension, conceptual understanding, critical evaluation, decoding, inference,


interpreting analysis and vocabulary are the skills to be tested.

Some tips to answer unseen passage questions:


1. Read the questions first. It saves time to know what you are reading for.
2. Make sure you understand the questions. Underline the key word(s) in the questions and look
in for the key words in the passage.
3. Now, Read the passage as quickly as you can. Look for the answer as you read. When you
find the answer, take notice of it.
4. Use the key words from the question to locate the answer in the passage. In this way you can
be sure that your answer is the best and complete.
5. Try to get a general idea of the passage. You are not expected to know the meaning of all the
words in the passage. You should work in collaboration with Eyes, Hand and Brain.
6. After 1set reading, read the questions once again. The questions will give you the idea from
which part, the questions are asked. It will make answering the questions easy and correct.
7. Synonyms will be asked from the passage. If you do not know the word, read the referred
paragraph again. You will understand what the line is trying to convey and hence there is a
possibility that the meaning will become clear to you and you will be able to answer its synonym.
You just have to puzzle out the meaning.
8. Solve previous year‘s exam papers: - (CODE NO.184 LANGUAGE & LITERATURE)

By solving the previous papers you can understand and identify what kind of questions are framed
in the examination. This will provide you sufficient practice along with building up your
confidence and enable you to mentally prepare for those kinds of questions.
DISCURSIVE PASSAGES
PASSAGE 1

Read the passage given below:


(1) Happiness lies within the mind of an individual. No amount of external wealth may be
helpful in making him happy. Our forefathers had lifestyles based mainly on the concept
of “simple living & high thinking”. Excessive material wealth did not mean much to them.
(2) The structural framework of our forefather’s families was different from those of ours.
The bond of love which they shared cannot be easily found today. The family provided an
emotional cushioning effect against tension and stress. However, in today’s nuclear
family, we are detached from feelings towards our kith and kin. This may finally destroy
the family psyche.
(3) The joint family system provided a proper environment for the child to grow up. The
values of respect, tolerance, responsibility, and integrity were internalized in the child. In
the long run, they became better human beings, compared to those in the present
generation. Our forefathers felt happiest, if their children became honest human beings.
10
But today, we are happy only if we attain our coveted material ends and are ready to resort
to any means to attain that end.
(4) Our forefathers had a vision to make India the best. To attain their ambitions, they were
ready to make all sorts of personal sacrifices. On the other hand, today people are ready to
migrate to the west, to enjoy a comfortable life. Often, they become successful in foreign
lands. But, in the process, they become alienated from their motherland. Also, distance
from their ailing parents is a worrying factor and keeps them perturbed. It is not easy for
them to return, as their children will not be able to adjust
the Indian environment and way of life. Thus, this is a crisis and a frame of mind worse
than their forefathers.
(5) In our generation, tradition and modernity have intermingled to form a special system.
We are happier than our forefathers in being able to lessen evils like the rigid caste system,
untouchability, and child marriage, but we have failed to totally eradicate them. I fact,
electoral politics has made use of the caste system, through the issuance of party tickets
on the basis of caste, community, and religion.
On the basis of your understanding of the passage, answer questions that follow:

1 × 10 = 10
(a) How are our forefathers different from us?
(i) They had lifestyles based mainly on the concept of “simple living and high thinking”
(ii) They had different structural framework of families.
(iii) They made all sorts of personal sacrifices to attain their ambitions
(iv) All of these
(b) What impact does a joint family system have on a child?
(i) It helps a child to become a better human being.
(ii) It internalizes the values of respect, tolerance, responsibility and integrity in a child
(iii) Both (i) & (ii)
(iv) Neither (i) nor (ii)

(c ) In our generation, ---------------- and ------------ have intermingled to form a special


system.
(i) Untouchability, intolerance
(ii) Tradition: modernity
(iii) Religion: caste
(iv) Politics; corruption

(d) based on your reading of the passage, choose the incorrect statement from the
following.
(i) Our youngsters had a vision to make India the best
(ii) Happiness lies within the mind of an individual.
(iii) The family provided an emotional cushioning effect against tension and stress.
(iv) Our forefathers had lifestyles based mainly on the concept of simple living and high
thinking.
(e) choose the option that correctly states the two meanings of “cushioning”, as used in
the passag
(A) Sharpening (B) Shielding (C) Intensifying (D) Softening (E) Irritating
(i) A, E (ii) B, D (iii) C, E (iv) A, C
(f) The ------------ provided a proper environment for the child to grow up.
(i) neighbourhood (ii) joint family system (iii) nuclear family system (iv) friends & relatives
11
(g) “In our generation, tradition and modernity have intermingled to form a special
system.” Substitute the underlined word with the most appropriate option from the following:
(i) Divided (ii) Blended (iii) Scattered (iv) Detached
(h) We are happier than our forefathers in being able to lessen:
(i) child marriage (ii) unemployment (iii) religious values (iv) none of these
(i) select the option that makes the correct use of “sorts”, as used in the passage, to fill in
the blank space.
(i) Policemen have to know all ----------- of different things.
(ii) The machine ------- it all out.
(iii) The computer -------- the words into alphabetical order.
(iv) He always ---------- the books according to the author’s name.
(j) What are the values internalised in the child?
(i) Respect (ii) Tolerance (iii) Responsibility (iv) All of these

ANSWER KEY-
(a) Iv. All of these
(b) (iii) Both (i) & (ii)
(c) (ii) Tradition: modernity
(d) (i) Our youngsters had a vision to make India the best
(e) (ii) B, D
(f) (ii) joint family system
(g) (ii) Blended
(h) (i) child marriage
(i) (i) Policemen have to know all ----------- of different things.
(j) (iv) All of these-
PASSAGE 2
Read the following passage carefully and answer the questions it follows. [ANY 10]
The choices we make on a daily basis—wearing a seatbelt, lifting heavy objects correctly or
purposely staying out of any dangerous situation—can either ensure our safety or bring about
potentially harmful circumstances.
You and I need to decide that we are going to get our lives in order. Exercising self-control, self-
discipline and establishing boundaries and borders in our lives are some of the most important
things we can do. A life without discipline is one that’s filled with carelessness.
We can think it’s kind of exciting to live life on the edge. We like the image of “Yeah! That’s
me! Living on the edge! Woo-hoo!” It’s become a popular way to look at life. But if you see,
even highways have lines, which provide margins for our safety while we’re driving. If we go
over one side, we’ll go into the ditch. If we cross over the line in the middle, we could get killed.
And we like those lines because they help to keep us safe. Sometimes we don’t even realize how
lines help to keep us safe.
I’m not proud of this, but for the first 20 years of my life at work, I ignored my limits. I felt
horrible, physically, most of the time. I used to tell myself “I know I have limits and that I’ve
reached them, but I’m going to ignore them and see if or how long I can get by with it.” I ran to
doctors, trying to make myself feel better through pills, vitamins, natural stuff and anything I
could get my hands on. Some of the doctors would tell me, “It’s just stress.” That just made me
mad. I thought stress meant you don’t like what you do or can’t handle life, and I love what I do.
But I kept pushing myself, traveling, doing speaking engagements and so on— simply exhausting
myself.
Finally, I understood I was living an unsustainable life and needed to make some changes in my
outlook and lifestyle.
12
You and I don’t have to be like everyone else or keep up with anyone else. Each of us needs to
be exactly the way we are, and we don’t have to apologize for it. We’re not all alike and we need
to find a comfort zone in which we can enjoy our lives instead of making ourselves sick with an
overload of stress and pressure.

On the basis of understanding of the passage, answer ANY TEN questions that follow.
(1×10=10)
Q1) Which of the characteristics are apt about the writer in the following context: “I
know I have limits and that I’ve reached them, but I’m going to ignore them and see if or
how long I can get by with it.”?

1. negligent
2. indecisive
3. spontaneous
4. reckless
5. purposeless
6. patient

a) 2 and 5
b) 3 and 6
c) 1 and 4
d) 2 and 3
Q2) The reason why living on the edge has become popular, is because of the
a) constant need for something different.
b) population being much younger.
c) exhausting effort to make changes.
d) strong tendency to stay within our limits.
Q3) The phrase “potentially harmful circumstances” refers to circumstances that can
(a) certainly, be dangerous.
(b) be fairly dangerous.
(c) be possibly dangerous.
(d) seldom be dangerous.
Q4). Choose the option that correctly states the two meanings of ‘outlook’, as used in the
passage.
1. A person’s evaluation of life
2. A person’s experiences in life
3. A person’s point of view towards life
4. A person’s regrets in life
5. A person’s general attitude to life
a) (1) and (4)
b) (2) and (3)
c) (3) and (5)
d) (4) and (5)
Q5) Choose the option that best captures the central idea of the passage from the given
quotes.
1. It’s all about quality of life and finding a happy balance between work and friends.
2. To go beyond is as wrong as to fall short.
3. Life is like riding a bicycle. To keep your balance, you must keep moving.
4. Balance is not something you find, it’s something you create.

13
a) Option (1)
b) Option (2)
c) Option (3)
d) Option (4)
Q6) The author explains the importance of discipline and boundaries in our lives using the
example of
a) road accidents.
b) traffic rules.
c) lines on the highway.
d) safe driving.
Q7) The author attempts to __________________ the readers through this write-up.
a) rebuke
b) question
c) offer aid to
d) offer advice to
Q8) What is the message conveyed in the last paragraph of the passage?
a) Love what you do.
b) Love yourself to love others.
c) Be the best version of yourself.
d) Be yourself
Q9) Which of the following will be the most appropriate title for the passage?
a) Much too soon
b) Enough is enough
c) How much is too much?
d) Have enough to do?
Q10) The author uses colloquial words such as “yeah” and “Woo-hoo!”. Which of the
following is NOT a colloquial word?
a) hooked
b) guy
c) stuff
d) stress
ANSWER KEY
Ans 1) (c) 1 and 4
Ans 2) (a) constant need for something different.
Ans 3) (c) be possibly dangerous
Ans 4) (c) (3) and (5)
Ans 5) (d) Option (4)
Ans 6) (c) lines on the highway.
Ans 7) (d) offer advice to
Ans 8) (c) Be yourself
Ans 9) (c) How much is too much?
Ans 10) (d) stress

PASSAGE 3

Read the following passage carefully and answer the questions:


1. As the family finally sets off from home after many arguments. there is a moment of lull
as the car takes off. "Alright. so where are we going for dinner now?" asks the one at the driving
wheel. What follows is a chaos, as multiple voices make as many suggestions.

14
2. By the time, order is restored and a decision is arrived at, tempers have risen, feelings
injured and there is at least one person grumbling.
3. Twenty years ago, you would step out of home, decision of meal and venue already made
with no arguments or opposition and everybody looked forward to the meal with equal
enthusiasm. The decision was made by the head of the family and the others fell in line. Today
every member of the family has a say in every decision which also promotes a sense of
togetherness and bonding.
4. We empower our kids to take their own decisions from a very early age. We ask them the
cuisine they prefer, the movie they want to see, the holiday they wish to go on and the subjects
they wish to study.
5. It's a closely connected world out there, where children consult and guide each other. A
parent's well-meaning advice can sound like nothing more than unnecessary preaching. How then
do we reach our children through all the conflicting views and make the voice of reason be heard?
Children today question choices and prefer to go with the flow.
6. What then is the best path to take? I would say the most important thing one can do is to
listen. Listen to your children and their silences. Ensure that you keep some time aside for them,
insist that they share their stories with you. Step into their world. It is not as complicated as it
sounds; just a daily half an hour of the quality time would do the trick.
(i) According to the passage, who said, "so, where are we going for dinner now?"
a. narrator
b. one at the steering
c. one of the passengers
d. one of the guests
(ii) Twenty years ago, according to the passage, decisions were
a. unanimous, with no argument
b. made by the children
c. taken by the head of the family
d. not taken by elders
(iii) The passage concludes that in modern times, children are to take their own decision
from a very early age.
a. forbidden
b. trained
c. not given permission
d. allowed
(iv) Choose the antonym of the below given word as given in para 6 of this passage.
Word- ‘Simple’
(v) Twenty years ago, the decision was made by the head of the family and the others
fell in line. How the decision is taken now a days?
(VI) What is the message conveyed by this passage? Write any two points.
(vii) After arriving at a decision almost everyone is_____ in the family.
a. happy
b. excited
c. hurt
d. elated
(viii) In modern times, the say each family member in decision making points towards in
the family.
a. grudges
b. perfect bonding
c. individual opinions
15
d. none of these
(ix) The writer used word "hurt" in para 2. What is the synonym of hurt as used in this
paragraph
(x) The word which means the same as a style or method of cooking in para 4 is
a. gourmet
b. cuisine
c. gastric
d. science

ANSWER KEY

(i) (b) one at the steering


(ii) (c) taken by the head of the family
(iii) (d) allowed
(iv) complicated
(v) Today every member of the family has a say in every decision which also promotes a sense
of togetherness and bonding.
(vi)Parents should:
1. give children some space
2. listen to each need of theirs
3. let them solve their problems by helping them from the back
(vii) (c) hurt
(viii) (b) perfect bonding
(ix)injured
(x) (b) cuisine
PASSAGE 4
Read the following passage carefully:

A youngster quit Facebook in December after spending over three years on the social networking
site. With that one act, he bid a silent adieu to more than 300 contacts that he had added to his
account during the period. Like almost everyone from his "friends' circle," the 20–year–old was
a regular on the service, visiting it every day to post photos and status updates. But last week, a
new feature on Facebook called timeline forced him to reconsider the pros and cons of being on
the networking site.

'Everyone has some skeletons in their closet, and I am just not comfortable with Facebook
digging up and displaying all the facets of my life on a bulletin board,' says this youngster who
joined the network in July 2007 while he was in Class 11.
Facebook, you see, had compressed the time he spent on the site and arranged it in chronological
order. And while he initially liked the new, neatly organised scrapbook-like feature, he wasn't
happy to reveal posts from the past, those that, until recently, were hidden under layers and layers
of recent updates. Just clicking on a date on the timeline could transport his friends back in time
and enable them to view every embarrassing comment, link or photo he had posted on his profile
"I think it's a recipe for disaster," he says. "In 2007, I had some wall posts, which seemed
appropriate at the time, but now after a lapse of four years, I have moved on and don't want them
to be openly displayed for all to see.
" And he is not alone. Many users, worried about how the Facebook activity could affect their
offline lives, choose to commit 'Facebook suicide'. While some have privacy concerns, others
feel that the site meant to bring them closer to their friends does the opposite – it reduces their
friendship to something superficial.
16
"Poking and liking are not enough to keep a friendship going," says a business analyst. Having
quit Facebook three years ago, she prefers meeting her 'real' friends’ face–to–face instead of
reading their trite posts online.
"On Facebook, people hype everyday issues including what they ate and where they went on a
daily basis," says this analyst who continues to use Twitter.
Similarly, an engineering student quit Facebook last December, four years after joining it. He
exported all the data from his account into a little zip file and hit the delete button one fine day.
"I realised that when it came to my friends who mattered, I could keep in touch with them over
the phone or by meeting them in real life," he wrote on his blog.

Based on your understanding of the passage, answer the following questions by


choosing the most appropriate.

1. The feature of Facebook where one can see the posts, the embarrassing comments and
thoughts, photos and links that a person has put on his profile in the past is:
a. Create a story b. Facebook live c. Timeline d. Messenger
2. According to the passage, the social media:
a. Causes more harm than good. b. Is just a waste of time.
c. Connects one to real life. d. Provides an opportunity to meet people face
to face.
3. The feature 'Timeline' on Facebook, according to the passage, forces the youngster to
reconsider:
a. Meeting friends face to face. b. Increasing use of social media.
c. Saving time on social media. d. The advantages and disadvantages of being on Facebook
4. Choose the option that best captures the central idea of the passage from the given quotes
a. "What is interesting is the power and the impact of social media. So, we must try to use social
media in a good way." - Malala Yousafzai
b. "The great thing about social media was how it gave a voice to voiceless people."-Jon Ronson
c. "Facebook is not your friend. It is a surveillance engine." - Richard Stallman
d. "Social media is awesome because I can somewhere paint myself the way I want people to see
me." -Kevin Abstract
5. Many people are getting out of Facebook as they feel:
a. Addicted to it
b. That their Facebook activity could affect their real life
c. They should join Instagram, WhatsApp or Twitter d. Poking and liking is not enough
6. The meaning of the word 'share' in para 3 is?
a. liked b. timeline c. hidden d. reveal
7. Which of the following is the OPPOSITE in meaning to the word 'retained' as used in the
passage?
a. Exported b. delete c. Account d. quit
8. Which of the following statements is NOT TRUE in the context of the passage?
a. People love being poked and liked on Facebook.
b. People visit social media every day to post photos and for status updates.
c. No one prefers meeting 'real' friends face–to–face instead of reading their trite posts online.
d. Few think that social media reduces their friendship to something superficial.
9. The phrase "Facebook suicide" refers to get someone:
a. Poking and liking b. Quitting Facebook as it could affect their real
life.
c. clicking on a date on the timeline d. using Twitter
17
10. Which of the following statements is TRUE in the context of the passage?
a. Time spent on social media is better than meeting face‐to‐ face.
b. social media use could increase the time teens spend together in person.
c. Stalking, personal attacks, and misuse of information are some of the threats faced by social
media users.
d. Facebook activity does not affect real life.
ANSWER KEY
(1) c. Time line
(2) b. is just a waste of time
(3) c. Saving time on social media
(4) (b) that their facebook activity could affect their real life.

(5) b. That their Facebook activity could affect their real life
(6) (d)reveal
(7) delete
(8) (c) No on prefers meeting 'real' friends face-to-face, instead of reading their trite posts online.
(9) (b) quitting facebook as it could affect their real life.
(10) (a) Time spent on social media is better than meeting face-to-face.
PASSAGE 5
Read the passage given below.

For millions of people in India, the river Ganga is the most sacred river. It is considered as mother
and goddess. It is also a lifeline of millions of Indians who live on the banks of its course and depend
on it for their daily

needs. River Ganga is the third largest river globally by the amount of water that flows through it. It
is the longest river in India. The river water of Ganga is used for irrigation, transportation and fishing.
The Gangetic

plain formed by the river Ganga is one of the most fertile lands on earth. This is why almost 10% of
the world population lives here and earns its livelihood. The Ganga in India is the most worshipped
body of water. The

irony here is that despite being the most revered river, it is also the dirtiest one. It carries metals
thrown out by tanneries, waste produced by industries and urban waste from different cities. All this
has made river

Ganga the fifth most polluted river in the world. Another major reason that adds to the Ganga River
pollution is the coal-based power plants on its banks which burn tons of coal every year and produce
a lot of fly ash. This ash mixed with domestic wastewater is released into the river. This bad situation
calls for an urgent need to make efforts to reduce pollution and revive river Ganga. To achieve these
objectives, the government of India has started a programme named “Namami Gange Programme”.
The main pillars of this programme are sewage treatment, river surface cleaning, afforestation,
riverfront development and public awareness. The importance of the success of the “Namami Gange
Programme” depends on the commitment and dedication of the people and administration.

Based on your understanding of the passage, answer any eight out of the ten
questions by choosing the correct option.
1. __________________ river is a lifeline for millions of
Indians.
18
A. Krishna
B. Ganga
C. Narmada
D. Yamuna
2. _________________ of world population lives near river
Ganga.
A. 15%
B. 20%
C. 30%
D. 10%
3. Ganga is the ___________________ most polluted river in the
world.
A. Second
B. Third
C. First
D. Fifth
4. What are the parameters in deciding Ganga as the third largest
river in the world?
A. Millions of people live on its bank.
B. The amount of water that flows through it.
C. Millions of people depend on it for their livelihood.
D. All of the above
5. What is the feature of the river Ganga that makes it so unique?
A. It is the longest river in the world.
B. Its water is used for irrigation.
C. Gangetic plain is one of the most fertile lands on earth.
D. Millions of people live on its bank.
6. If we talk about Ganga, the irony with Ganga
is_____________________ .
A. Despite being the most worshipped body of water, it is
also, the dirtiest one.
B. It is considered as mother and goddess.
C. People throw waste in it.
D. Its water is used for irrigation.
7. What makes river Ganga polluted?
A. Metals thrown by tanneries.
B. Waste produced by industries.
C. Urban waste (sewage)
D. All the above
8. Which programme has been initiated to ensure the revival of clean Ganga?
A.Cleaning The River
B. Namami Gange
C. Mission Gangotri
D. Purify Ganga
9. The main pillars of this programme are____________________________.
A. River surface cleaning and afforestation
B. Sewage treatment and public awareness
C. Riverfront development
D. All the above
19
10. Another significant threat to Ganga is _____________________________.
A. Development
B. Careless attitude of people
C. Dumping all kinds of waste in it.
D. Coal-based power plants.
ANSWER KEY
1. A. Krishna
2. D. 10%
3. D. Fifth
4. B. The amount of water that flows through it
5. C. Gangetic plain is one of the most fertile lands on earth.
6. In spite of being the most worshiped body of water, it is also
the dirtiest one.
7. D. All the above
8. B. Nanami Ganga
9. D. All the above
10. D. Coal based power plants.
UNSOLVED PASSAGE 1
(1) During our growing up years, we as children have been taught both at home and also school-
to worship the photos and idols of the Gods of our respective religions. When we grew a little
older, we were told to read holy books like the Bhagwad Gita, the Bible and the Quran; we were
told that there are a lot of life lessons to be learnt from these holy books. We were then introduced
to stories from our mythologies which have taught us about ethics and morality-what is good and
what is bad. I also learnt to be respectful towards my parents who have made my life comfortable
with their hard work and love and care, and my teachers who have guided me to become a good
student and also a responsible citizen.

(2) Much later in life, I realised that though we have learnt much from our holy books, there is a
lot to learn from our surroundings. This realisation dawned upon me when I learnt to enquire and
to explore. Everything around us the sun, the moon, the stars, rain, rivers, stones, rocks, birds,
plants and animals-teach us many valuable life lessons.

(3) No wonder that besides the scriptures in many cultures, nature is also actively worshipped.
The message that we get is to save our environment and to maintain ecological balance. People
are taught to live in harmony with nature and recognise that there is God in all aspects of nature.

(4) Nature is a great teacher. A river never stops flowing. If it finds an obstacle in its way in the
form of a heavy rock, the river water fights to remove it from its path or finds an alternative path
to move ahead. This teaches us to be progressive in life, and to keep the fighting spirit alive.
(5) Snakes are worshipped as they eat insects in the field that can hurt our crops, thus protecting
the grains for us. In fact, whatever we worship is our helper and makes our lives easy for us.
There are many such examples in nature, but we are not ready to learn a lesson yet. Overcome
with greed, we are destroying the nature. As a result, we face natural disasters like drought, flood
and landslides. We don't know that nature is angry with us.
However, it is never too late to learn. If we learn to respect nature, the quality of our life will
definitely improve.
On the basis of your understanding of the above passage, answer the questions that follow:
Q.1. When we are children, we are taught to
(A) go to school every day
20
(B) help your parents
(C) study regularly
(D) to worship the photos and idols of the Gods of our respective religions
Q.2. What do we learn from the holy books?
(A) life lessons (B) good manners (C) about nature (D) about our past
Q. 3. Why should we respect our parents
(A) They teach us good values.
(B) They work hard to make our life comfortable.
(C) It is our duty.
(D) All of these
Q. 4. The narrator learnt that we learn a lot from our surroundings when he learnt to
(A) ask questions
(B) enquire and explore
(C) analyse the scripture
(D) decipher the scriptures
Q.5. In many cultures besides scriptures, …………. is also worshipped.
(A) Idols
(B) planets
(C) nature (D) water
Q. 6. According to the author, where do we learn more than our holy books?
(A) parents (B) grandparents (C) surroundings (D) family
Q. 7. What message do we get from the scriptures?
(A) to save our environment and maintain ecological balance
(B) to respect our parents
(C) to be kind
(D) to be virtuous Ans.
Q. 8. What does the river teach us?
(A) to be progressive in life
(B) to face the obstacles
(C) to never give up
(D) to face the challenges
Q. 9. The synonym of ‘advised’ is ………….
(A) ethics (B) respective (C) guided (D) morality
Q. 10. The synonym of 'search' in para 2is_
(A) dawned (B) enquire (C) valuable (D) explore
UNSOLVED PASSAGE 2
I. Read the passage given
below. 1. Hobbies help us grow as
a person. The best way to have a new hobby is to try something new. All of us are unique, and
this is the reason why our hobbies and interests are different. Once we find an activity we are
passionate about, we can explore that activity more. When you get hooked, you will realize that
your hobby has become an integral part of your life. Having a hobby that we enjoy brings us joy
and refreshes us. Hobbies help us to manage our leisure and unplanned time more productively.
It also affords you the opportunity to learn new skills in your work. But these are not the only
benefits of having a hobby.
2. It feels great to be skilled and good at something, doesn’t it? And this is what that makes you
confident. It can take some time to develop your hobby so that you may be able to tell that you
are skilled. But, the journey of experiencing your hobby is very rewarding in itself. With the
exposure to different types of activities these days, it doesn’t matter which activity you choose.
21
Whether you are pursuing a craft, sports, puzzles, or skill development, your hobby should be a
diversion and a passion. Simultaneously, if your hobby gives you a sense of purpose, then you
will be more confident about challenging yourself in your hobby and help you prepare for learning
new things at work.
3. Hobbies give you an opportunity to enhance your life Hobbies allow you to de-stress yourself
while remaining mentally productive. Having hobbies promotes better health and may lower the
risk of having high blood pressure. Enjoying a few hours of your hobby a week can also reduce
the risk of depression and dementia. Hobbies refocus your mind on to something that you enjoy
doing. Hobbies that include physical activities create chemical changes in your body that help to
reduce stress.
4. Even if it doesn’t involve physical activities, you can still reap the benefits of having a hobby.
Getting a short break from work and doing something you are passionate about can rejuvenate
your mind and help prepare you to handle challenges in the future in a better way.
5. Hobbies may provide an opportunity for you to socialize with people and that can be an
additional benefit for your overall well-being. You can connect with people who enjoy doing the
same things that you do. You can meet new people, discuss your hobby and get connected to a
bigger circle that may even help you turn your hobby into your profession. The internet provides
numerous groups and forums to connect with people that enjoy doing the same things that you
want to do.
6. Some hobbies require creativity and if you develop creativity through your hobby, it can be
beneficial. Creativity can help you experience new things at school and work. The skill of being
creative is essential in today’s world.
Based on your understanding of the passage, answer the questions given below.
i. When a person gets hooked on a hobby, he realises that:
(a) it has become an integral part of our life.
(b) it has much more benefits than we realised.
(c) it is much more enjoyable to have a hobby.
(d) it is quite essential for us to learn some skills through our hobbies.
ii. No matter which sort of hobby you are pursuing, be it a craft, sports, puzzles, or
skill development, your hobby should be:
(a) useful for you in the long run.
(b) a skill that improves your present circumstances.
(c) a diversion and a passion.
(d) followed through till the end.
iii. The journey of experiencing a hobby is quite:
(a) challenging for a person.
(b) rewarding in itself.
(c) demanding in itself.
(d) inspirational for a person.
iv. State any one way through which hobbies enhance your life?
v. Getting a hobby means having a short break from work and doing something you
are passionate about which can
(a) give you a peace of mind from all the other problems of daily hectic life and make
you feel refreshed.
(b) help you enhance and improve your skills and push you forward on your path to
success.
(c) make you enjoy and appreciate your life much more as you are happy pursuing what
you love.

22
(d) rejuvenate your mind and help prepare you to handle challenges in the future in a
better way.
vi. List any two benefits of having a hobby.
1.
2.
vii. What is essential in today’s world?
viii. Give an appropriate title for the passage.
ix. Supply 1 point to justify the following:
Hobby provides you with an opportunity to socialise with people.
x. Choose an option that is clearly a synonym of the word ‘hooked’.
(a) opposed
(b) indifferent
(c) willing
(d) captivated
FACTUAL PASSAGES

PASSAGE 1

Q1 Read the passage given below:

Marie was born in 1867 in Warsaw, Poland, where her father was a Professor of Physics. At an
early age, she displayed a brilliant mind and a blithe personality. Her great exuberance for learning
prompted her to continue with her studies after high school. She became disgruntled, however,
when she learned that the University of Warsaw was closed to women. Determined to receive a
higher education, she defiantly left Poland and in 1891, entered the Sorbonne, a French university,
where she earned her master’s degree and a doctorate in physics.

EVENT YEAR

Marie was born on the 7th of November 1867

Her sisters Bronya and Zosia became ill and Zosia passed away. 1876

Curie’s mother died from grief. 1878

She graduates high school 1883

Leaves Warsaw for a governess position 1884

Her sister Bronya graduated medical school. 1891 1891

23
Marie graduated with a master’s degree in Mathematics and first in her class for
Physics. 1893

She married Pierre Curie 1895

Marie Curie was

a. The first woman with a degree in Physics.

b. The first woman to graduate in France.

c. The first woman to obtain a Nobel Prize.

d. The first woman to obtain a chair at the Sorbonne.

e. The first scientist to obtain two Nobel Prizes.

(2) Marie was fortunate to have studied at the Sorbonne with some of the greatest scientists of her
days, one of whom was Pierre Curie. Marie and Pierre were married in 1895 and spent many
productive years working together in the physics laboratory. A short time after they discovered
radium, Pierre was killed by a horse-drawn wagon in 1906. Marie was stunned by this horrible
misfortune and endured heart-breaking anguish. Despondently she recalled their close relationship
and the joy that they had shared in scientific research. The fact that she had two young daughters to
raise by herself greatly increased her distress.

(3) Curie’s feeling of desolation finally began to fade when she was asked to succeed her husband as
a physics professor at the Sorbonne. She was the first woman to be given a professorship at the world-
famous university. In 1911, she received the Nobel Prize in chemistry for isolating radium. Although
Marie Curie eventually suffered a fatal illness from her long exposure to radium, she never became
disillusioned about her work. Regardless of the consequences, she had dedicated herself to science
and to revealing the mysteries of the physical world.

On the basis of your understanding of the passage, answer any ten questions from the twelve
that follow: 1 × 10 = 10

Q1. In the line “…… she defiantly left Poland,” the word “defiantly” DOES NOT refer to

(a) Coldly (b) Challenging (c) Resistant (d) Openly

Q2. The events in the life of Marie Curie could reveal the following points:

(a) She was determined to receive higher education.

(b) She was eager for women empowerment.

(c) She had a brilliant mindset and rude behaviour.

24
(d) She never liked her personal and professional life with Pierre Curie.

Q3. Marie …………………… by leaving Poland and travelling to France to enter the Sorbonne.

(a) challenged authority (b) showed intelligence (c) got happy (d) was distressed

Q4. Choose the option that lists the correct answers for the following:

(a) Ashwini, a physicist was working hard on her research on the black hole. She worked really very
hard to make her research a unique one. She did a lot of experimental work to come up with better
inferences. What kind of researcher is she?

(b) Lata was a lady of firm determination. She lost her husband after three years of marriage. She had
two daughters whom she raised all her own. She felt the void in her life but was never disappointed
or disheartened by that. What kind of lady is she?

(a) (1) disappointed and (2) experimental

(b) (1) determined (2) independent

(c) (1) independent (2) determined (d) (1) diligent and (2) resolute
Q5. Based on your understanding of the passage, choose the option that lists the qualities of
the professor.
(1) Scholar (2) Distressed (3) Exuberant (4) Lethargic (5) A bad student (6) Thoughtful
(a) (2) and (4)
(b) (5) and (6)
(c) (1) and (3)
(d) (3) and (6)
Q6. Even though she became fatally ill by working with radium, Marie Curie was never.
(a) cheerful (b) enthusiastic (c) staunch (d) disillusioned
Q7. Marie became ‘aggrieved’ when she found that
(a) Her husband was dead.
(b) The University of Warsaw was closed for women.
(c) She was ill.
(d) She will have to leave her birthplace.
Q8. According to the passage, how long did the married life of Marie & Pierre Curie
survive?
(a) 12 years (b) 10 years (c) 11 years (d) 15 years
Q9. Marie’s mother was …………………… and died after the death of …………………….
(a) heartbroken, her friends
(b) depressed, two of her daughters
(c) heartbroken, one of her daughters
(d) heartbroken, her cousin
Q10. Choose the option that lists statements that are NOT TRUE.
(a) Marie was brilliant in her studies.
(b) Marie was the first scientist to obtain two Nobel Prizes.
(c) Marie succeeded her husband Pierre at Sorbonne university.
(d) Marie didn’t reveal many facts and mysteries of the physical world.

ANSWER KEY
1. (d) Openly
25
2. ANS- (a) She was determined to receive a higher education.
3. ANS- a) challenged authority
4. ANS- (d) (1) diligent and (2) resolute
5. ANS- (c) (1) and (3)
6. ANS- (d) disillusioned
7. ANS- (a) Her husband was dead.
8. ANS-(c) 11 years
9. ANS- (c) heart broken, one of her daughters
10. ANS-(d) Marie didn’t reveal much facts and mysteries of the physical world.
PASSAGE-2
Read the following passage carefully: 10m
Climate change is considered to be one of the most serious threats to sustainable development,
with adverse impacts on the environment, human health, food security, economic activity, natural
resources, and physical infrastructure. The global climate varies naturally. According to the
Intergovernmental Panel on Climate Change (IPCC), the effects of climate change have already
been observed, and scientific findings indicate that precautionary and prompt action is necessary.
Vulnerability to climate change is not just a function of geography or dependence on natural
resources; it also has social, economic, and political dimensions which influence how climate
change affects different groups. Poor people rarely have insurance to cover loss of property due
to natural calamities like droughts, floods, super cyclones etc. Poor communities are already
struggling to cope with the existing challenges of poverty and climate variability and climate
change could push many beyond their ability to cope or even survive. It is vital that these
communities are helped to adapt to the changing dynamics of nature.
Adaptation is the process through which societies make themselves better able to cope with an
uncertain future. Adapting to climate change entails taking the right measures to reduce the
negative effects of climate change (or exploit the positive ones) by making the appropriate
adjustments and changes. These ranges from technological options such as increased sea defences
or flood-proof houses on stilts, to behavioural change at the individual level, such as reducing
water use in times of drought.
Other strategies include early warning systems for extreme events; better water management;
improved risk management; various insurance options; and biodiversity conservation. Because
of the speed at which climate change is happening due to global temperature rise, it is urgent that
the vulnerability of developing countries to climate change be reduced, their capacity to adapt be
increased, and national adaptation plans be implemented. Adapting to climate change will entail
adjustments and changes at every level, from community to national and international.
Communities must build their resilience, including adopting appropriate technologies while
making the most of traditional knowledge and diversifying their livelihoods to cope with current
and future climate stress. Local coping strategies and knowledge need to be used in synergy with
government and local interventions. The need for adaptation interventions depends on national
circumstances. There is a large body of knowledge and experience within local communities on
how to cope with climatic variability and extreme weather events.
Local communities have always aimed to adapt to variations in their climate. To do so, they have
prepared based on their resources and their knowledge accumulated through experience of past
weather patterns. This includes times when they have also been forced to react to and recover
from extreme events such as floods, droughts, and hurricanes.
Local coping strategies are an important element of planning for adaptation. Climate change is
leading communities to experience climatic extremes more frequently, as well as new climate
conditions and extremes. Traditional knowledge can help to provide efficient, appropriate, and

26
time-tested ways of advising and enabling adaptation to climate change in communities who are
feeling the effects of climate change due to global warming.
On the basis of your understanding of the passage, answer any ten questions from the
twelve that follow: 1 × 10 = 10

1. Which among the following is not a factor in the vulnerability of poor people to climate
change?
a) Their dependence on natural resources
(b) Geographical characteristics
(c) Lack of financial resources
(d) A scarcity of traditional knowledge
2. Adaptation as a process enables societies to cope with:
(a) An uncertain future
(b) Adjustments and changes
(c) Negative and positive impacts of climate change
(d) (d) All of the preceding
3. To address the challenge of climate change, developing countries urgently require:
(a) Imposition of climate change tax
(b) Implementation of national adaptation policy at their level
(c) Adoption of short-term plans
(d) Adoption of technological solutions
4. The traditional knowledge should be used through:
(a) Its dissemination
(b) Improvement in national circumstances
(c) Synergy between government and local interventions
(d) Modern technology
5. The main focus of the passage is on:
(a) Combining traditional knowledge with appropriate technology
(b) Co-ordination between regional and national efforts
(c) Adaptation to climate change
(d) Social dimensions of climate change
6. Choose the option which best opposes the term 'resilience'
(a) Flexibility
(b) Vulnerability
(c) Adaptability
(d) Elasticity

7. Which statement is not true, according to the passage?


(a) Climate change favourably impacts the environment, human health, food security,
economic activity, natural resources, and physical infrastructure.
(b) It is essential that poor communities are helped to adjust to the changing dynamics of
nature.
(c) (c) Adapting to climate change will entail adjustments and changes at every level, from
community to national and international.
(d) By adopting appropriate technologies while making the most of traditional knowledge
and diversifying their livelihoods, communities can cope with current and future climate stress.
II Answer the following questions in short-
8. What has helped the local communities to adapt to the variations in their climate?

27
9. What is the major reason mentioned in the passage for climate change?
10. What care should be taken by the developing countries in respect of climate change?
ANSWER KEY-
(I) 1. (d) Scarcity of traditional knowledge
2. (d) All the above
3. (b) Implementation of national adaptation policy at their level.
4. (c) Synergy between Government and local interventions
5. (c) Adaptation to climate change
6. (b) Vulnerability
7. (a) climate change favourably impacts the environment, human health, food security
economic
activity, natural resources and physical infrastructure.
(II) (8) The preparations made by the local communities based on their resources and their
knowledge accumulated through experience of past weather patterns have helped them to adapt
to the variations in their climate change.
9. The major reason mentioned in the passage for climate change is global warming/Global
temperature rise
10. Developing countries should take care to reduce vulnerability to climate change and to
increase their capacity to adapt to climate change.
PASSAGE 3
Read the following passage carefully:
More than three lakh workers will be employed in the solar and wind energy sectors to meet the
country’s target of generating 175 gigawatts of electricity from renewable sources by 2022, an
International Labour Organization (ILO) report said. The report titled, World Employment and
Social Outlook (WESO) 2018: Greening with Jobs, quoted from a study conducted by the
Council on Energy, Environment and Water (CEEW) and the Natural Resources Defence
Council (NRDC), on the changes in sectoral employment that will occur in order to meet India’s
target. The study was based on surveys of solar and wind companies, developers and
manufacturers.
India is rapidly increasing its share of renewable energy sources, but still relies on coal, oil,
natural gas, and the related carbon emissions for 80% of its electricity,” the report released on
Tuesday said. This formed a small part of the report, which focused on the trajectory of the
labour market in the backdrop of environmentally sustainable production practices. Tackling
the misconception that green economies pave the way for economically undesirable outcomes,
the report said rather than a trade- off between the two, their development goes hand in hand.
According to the ILO report, there will be a net increase of 18 million jobs across the globe as
a result of environmentally sustainable measures taken in the production and use of energy.
This net figure is based on the estimation that the resultant job losses of six million will
eventually lead to an increase of 24 million jobs as greener practices are adopted. Of this, 14
million jobs created will be in Asia and the Pacific.
The transition to a green economy will inevitably cause job losses in certain sectors as carbon
and resource-intensive industries are scaled down, but they will be offset by new job
opportunities,” the report said. However, the report emphasized that the net increase of 18
million jobs is dependent on a supportive policy framework to aid displaced workers and skill
development programs to help ease them into jobs that require new skills. It mentioned that
although India does have a specific body or council to address the skills development for green
transition, it has no existing institutional mechanism to anticipate skills needs and adapt training
provision. Of the 27 countries surveyed, India and seven others fall under this category.

28
“Developing and emerging economies have relatively weaker institutional capacity for
integrating skills and environmental sustainability,” the report said
The report stressed on the urgency of economies adopting sustainable practices, adding, in 2013,
humanity used 1.7 times the amount of resources and waste that the biosphere was able to
regenerate and absorb. The report reads, “It is striking that in a context of scarce resources and
limited ability to absorb waste, current patterns of economic growth rely largely on the
extraction of resources, manufacturing, consumption and waste.” It explained this urgency from
the perspective of the job market by connecting labour productivity to climate change. “Looking
ahead, projected temperature increases will make heat stress more common, reducing the total
number of working hours by 2% globally by 2030 and affecting workers in agriculture, and
developing countries,” the report said.
Based on your understanding of the passage, answer the questions given below.
1.Which of the following weakens the argument of increasing the share of renewable energy
in the energy sector?
i. The environmental impacts associated with renewable energy include habitat loss, water use,
and the use of hazardous materials in manufacturing, they cause more harm than good in the long
run.
ii. It is easy to harness and store renewable energy than the traditional sources of energy.
iii.The shift to renewable energy source would create jobs
2. Which of the following statements weakens the argument about the urgency of economies
in adopting sustainable practices?
i. The increase in temperature because of climate change would reduce the number of working
hours hence productivity will suffer.
ii. The resource available to us is limited and nature's ability to replenish them is lim iii. The
impact of sustainable solutions for energy needs would be limited because for it to be effective
all the countries need to participate which is not the case.
3. As per the passage, which of the following could be a/some reason/s for the
misconceptions surrounding green economies.
i. There is a belief that adopting a green economy would lead to loss of jobs which could severely
impact the economy
ii. The shift from traditional energy sources to renewable energy would be expensive.
iii. The technological capability required to transform into a green economy is still in the
elementary phase.
4. As per your understanding of the passage, which of the following can be said to be
example/s of steps which contribute towards a green econ
i. The government announces tax incentives for those using public transport.
ii. The government provides subsidy on diesel cars so that it becomes affordable
iii. The government supports start-ups working on the development of electric cars.
5. “Transition “refers to
I. Hibernate II process of changing from one state to another III stagnate
6. Pick out a word from the passage that is similar to “make suitable for a new use or
purpose
i. Integrate ii. Scarce iii. Adapt
7. Complete the following sentences by filling the blanks using suitable words.
i. One of the steps that can be used by India to improve its institutional capacity for
integrating skills and environmental sustainability is by funding to support the shift of _
ii. The unfavourable effect of the transition to a green economy ————————.
iii. ———————————— would assist displaced workers to get new jobs.
iv. Increase in temperature will affect ————————————————.
29
8. Write the synonym of the word ‘EMERGING’ of the passage.

ANSWER KEY-

1.The environmental impacts associated with renewable energy include habitat loss, water use,
and the
use of hazardous materials in manufacturing, they cause more harm than good in the long run.
2. Option 3
The impact of sustainable solutions for energy needs would be limited because for it to be
effective all
the countries need to participate which is not the case.
3. option 1
The technological capability required to transform into a green economy is still in the
elementary phase.
4. option 3
The government supports start-ups working on the development of electric cars.
5. Option 2
Process of changing from one state to another
6. option 3
Adapt
7.
I. displaced workers into jobs that require new skills
II. Job losses in certain sectors
III. supportive policy framework
IV. Workers in agriculture and developing countries because it will reduce the total number of
working hours.
PASSAGE 4
Read the passage given below.
1.The novel corona virus has given rise to a global pandemic that has destabilized most
institutional settings. While we live in times when humankind possesses the most advanced
science and technology, a virus invisible to the naked eye has massively disrupted our lives,
economies, healthcare, and education systems worldwide.
2.Given the corona virus’s current situation, some households have also had time to introspect on
gender roles and stereotypes. For instance, women are expected to carry out household chores
like cooking, cleaning, and looking after the family. With men sharing household chores
responsibilities during the lockdown period, it gives hope that they will realize the burden that
women have been bearing and will continue sharing such responsibilities.
3.This tough period also gave people some time to reflect on the importance of keeping
themselves fit. With sufficient time in hand, people started investing their time learning new ways
to exercise. Those who never exercised before, giving excuses of busy lives, too developed some
new habits of Yoga, Pranayama and exercises during the lockdown period. These new habits and
people’s increased focus on their health, wellness and immunity will surely change the way we
lead our lives even in future.
4.The nature too healed itself during the lockdown period. Restricted human movement led to
better air quality, cleaner water bodies and joyful wildlife movements. The human beings, we
hope, reflected during this time, how some of their unconscious activities cause disruption in
nature and worked out ways to adopt environmental-friendly options for their activities in future.

30
5.This situation also affected the education sector to a great extent. It has forced us to shift from
offline to online mode of teaching-learning process, almost immediately without prior
preparation. Is it giving us a peek into the reality ahead? Technology- enabled teaching is
definitely the future we are looking towards, but it is important to identify key challenges for
students and teachers in the current scenario. Once identified, academic leadership and the
government can address these through innovations in the focused areas to minimise the effect of
pandemic on the education of the students.
6. The current scenario has also affected our economies to the extent wherein many businessmen
had to bear heavy losses in their businesses. The governments and individuals need to take actions
to mitigate risk and minimize transmission while maintaining social and economic activities.
However, relaxed control measures, declining risk perception and the understandable desire to
return to normalcy have led to reduced protective behaviour and more social and workplace
interactions, often in confined, close-contact settings, where the virus spreads really fast.
7.It is our responsibility that we take all necessary precautions through mask-wearing, physical
distancing, hand hygiene as part of daily life. It is highly important to make these new behaviour
part of our everyday habits. Travelling to new places, casual café visits with a large bunch of
friends, spending our weekends in shopping, window-shopping and casual strolls, large
gatherings in birthday parties and other celebrations; will require some modifications and
patience to fit into “New Normal” keeping all the safety norms in mind.
8.We are sure that regular communication from authorities, improved understanding of individual
responsibility and, subsequently, a greater willingness to adopt infection prevention practices can
be a stepping stone to a “new future”.
On the basis of your understanding of the passage answer questions that follow.
(1x10=10)
1. The outbreak of COVID-19 is called a pandemic because-
(a) it has spread across the globe. (b)it has spread across India
(c) it is invisible to naked eye (d)it has disrupted many institutional settings
2. According to the passage the lockdown period made people introspect on gender
roles and stereotypes because –
a) Women started handling all the household
responsibilities
b) Men started handling all the household responsibilities alone
c) People talked about gender stereotypes during lockdown period
d) Men started sharing responsibilities related to household chores
3. Choose the option that is NOT TRUE: People, who never exercised before, started
exercising during the lockdown period because-
(a) they had sufficient time in hand
(b) exercise was the only way to treat people from the novel corona virus.
(c) people learnt new ways to exercise their body.
(d) people understood the importance of health and wellness in the face of the pandemic
4. A positive change was seen in nature during lockdown period in terms of cleaner
air and water bodies because
(a) there was less human movement due to lockdown
(b) the virus helped in cleaning air and water
(c) the government made extra efforts to clean air and water
(d) People got together to clean water bodies.
5. How did schools continued educating students during the pandemic?
(a) Through offline mode of teaching
31
(b) Through online mode of teaching
(c) By calling students to school on weekly basis
(d) Students were asked to study at home themselves.
6. Which of the following has NOT led to reduced protective behaviours amongst
people?
(a) relaxed control measures (b) declining risk perception
(c) physical distancing (d) understandable desire to return to normalcy
7. Which of the following is OPPOSITE in meaning to the word ‘mitigate’ as used in
the passage?
(a) Lessen (b) reduce (c) aggravate (d)weaken
8. Which of the following is NOT TRUE in the context of COVID Appropriate
Behaviour?
(a) Wearing Mask (b) Being in crowded places (c) Washing hands (d) Maintaining physical
distancing
9. The phrase “stepping stone” refers to:
(a) Stones and pebbles lying on the road
(b) Something used as a way to progress
(c) The destination of our journey
(d) Blocks and problems in your path
10. Which of the following statements is NOT TRUE in the context of the passage?
(a) People started introspecting on gender roles and stereotypes.
(b) People started realising the importance of keeping themselves fit.
(c) There was a boom in the economy.
(d) School started following Technology-enabled online teaching

ANSWER KEY

1. a (a) it has spread across the globe.


2. d Men started sharing responsibilities related to household chores
3. B. exercise was the only way to treat people from the novel corona virus.
4. A. there was less human movement due to lockdown
5. Through online mode of teaching
6. (c) physical distancing
7. (c) aggravate
8. (b) Being in crowded places
9. (b) Something used as a way to progress
10. C. There was a boom in the economy.
PASSAGE 5
Read the passage given below.
1. The funding crisis at many zoos has reopened the debate over the value of zoos and whether
they should be allowed to exist at all.
2. People who are in favour of zoos argue that they perform an essential role in conserving rare
animal species. Conservationists estimate that today at least 1000 species of animals are
threatened. Over the past 20 years, zoos have developed programmes designed to help preserve
endangered species. This involves breeding animals in captivity in “captive breeding
programmes” and then reintroducing them into their natural habitats, to replenish the number
living in the wild.
3. Zoos cooperate with each other in order to ensure the success of their breeding programmes.
Animals are passed from one zoo to another in order to prevent inbreeding. If animals that are
32
closely related to one another mate, there is a danger that they will produce deformed offspring.
4. If zoos were forced to close, it would be disastrous for world conservation, say zoo supporters,
adding that most animals in captivity would have to be killed.
5. According to the National Federation of Zoos, it does not take much imagination to realise that
the closure of all zoos would mean the deliberate destruction of wildlife on a scale never before
witnessed.
6. Opponents of zoos accept that some species have been saved from extinction by the captive
breeding programmes, but they argue that this offers no solution to the worldwide conservation
crisis.
7. The number of animals protected by zoos is tiny compared with the overall problem. It costs
millions to save the Arabian oryx from dying out; but could that amount be available for every
species that is endangered? The value of zoo-breeding programmes is also questioned as some
species, such as the African elephant, do not reproduce well in captivity.
8. Captive animals are often kept in poor and inhumane conditions, the opponents say. In the
worst zoos’ animals are still displayed for the purpose of public entertainment. When animals are
placed in impoverished and unsuitable surroundings, they often behave in abnormal and neurotic
ways. It is common for polar bears to constantly pace up and down or twist their heads. This
behaviour is now recognized by scientists as a sign of stress and frustration.
9. When children visit zoos where animals are acting in neurotic and abnormal ways, they are not
being educated. Instead, opponents say, they are being given information that is inaccurate.
Q.01 On the basis of your reading of the given passage, choose the correct option.
(1x10=10)
a) What has reopened the debate over the values of zoos?
(i) Existence of many zoos (iii) cruelty towards animals
(ii) funding crisis at many zoos (iv) none of these
b) why do zoos cooperate with each other?
(i) To help preserve endangered species
(ii) To ensure the success of their breeding programmes
(iii) To conserve rare animal species (iv) none of these
c) Based on your understanding of the passage, choose the INCORRECT statement from
the following.
(i) The number of animals protected by zoos is tiny compared with the overall problem.
(ii) It is common for polar bears to constantly pace up and down or twist their head
(iii) Conservationists estimate that today at least 1,000 species of animals are threatened.
(iv) Animals are passed from one zoo to another in order to prevent infection.
d) Why do opponents say that the children are not being educated by visiting zoos?
(i) Because animals are displayed only for the purpose of public entertainment
(ii) Because animals are placed in impoverished and suitable surroundings.
(iii) Because the children are provided with inaccurate information about the way animals
act
(iv) All of these
e) Which of the following explains the word “opponent”
i) A person who advises others
ii) A person who engages in a fight
iii) A person who supports something or someone
iv) A person who disagrees with something or someone
f) People who are in favour of zoos argue that they perform an essential role in :
i) Preserving endangered species
ii) Reintroducing them into their natural habitats
33
iii) Conserving rare animal species
iv) Replenishing the number of animals living in the wild
g) Animals are passed from one zoo to another in order to prevent:
(i) Deformed offspring (iii) inbreeding
(ii) Rare animal species (iv) preserving animal species
h) Choose the option that correctly states the two meanings of “neurotic”, as used in the
passage.
A) Laid -back B) unstable C) anxious D) calm E) level -headed
(i) A, D ii) C, E iii) B, C iv) B, D
i) If animals that are closely related to one another mate, there is a danger that they will
produce deformed offspring. Substitute the underlined word with the most appropriate
option from the following.
(i) Ancestors ii) species iii) babies iv) forefathers
j) It is common for ------------ to constantly pace up and down or twist their heads.
(i) monkeys ii) polar bears iii) lion cubs iv) none of these
ANSWER KEY
a) (ii) funding crisis at many zoos
b) (ii) To ensure the success of their breeding programmes
c) Iv) Animals are passed from one zoo to another in order to prevent infection.
d) (iii)Because the children are provided with inaccurate information about the way
animals act
e) Iv) A person who disagrees with something or someone
f) (iii) Conserving rare animal species
g) (iii) inbreeding
h) iii) B, C
i) iii) babies
j) ii) polar bears
UNSOLVED 1
Read the passage given
below. 10 1. Conference series
LLC Ltd organises a conference series of 1000+ global events inclusive of 300+ conferences,
900+ upcoming and previous symposiums and workshops in USA, Europe and Asia with
support from 1000 more Scientific societies and publishes 700+ open access journals which
contain over 30000 eminent personalities, reputed scientists as Editorial board members.
2. Recycling and Waste Management Convention 2018 proudly invites contributors across the
globe to 9th World Convention on Recycling and Waste Management during October 22-23,2018
in Osaka, Japan, which includes prompt keynote presentations, oral talks, poster presentations
and exhibitions. We are delighted to say that it is the 9th World Convention on Recycling and
Waste Management which will be held in a beautiful city of Osaka, Japan and hence we invite
you all to attend and register.
3. 9th World Convention on Recycling and Waste Management is mainly based on the theme
“Advocating Waste Disposal and Recycling Practices for Clean and Green Environment.” We
warmly welcome all the participants - leading scientists, researchers and scholars of the world to
attend the convention. We provide a platform for young researchers and students to present their
research through oral presentations through which they can develop a foundation for
collaboration among young researchers.
4. The organizing committee aims at setting a platform for all the budding scientists and
researchers to present their real-time work and share their views and aspects related to the theme
of the conference. The organising committee is gearing up for an exciting and informative
34
conference programme including plenary lecture, symposia, workshops on a variety of topics,
poster presentations and various programmes for participants from all over the world.
Based on your understanding of the passage, answer the questions given below.
i. Infer one reason for the following, based on information in the passage. Waste
management and recycling it.
ii. Select the appropriate option to fill in the blanks.
has been considered as the second nature to us.
(a) Sorting of waste
(b) Recycling of waste
(c) Waste management
(d) Implementing steps framed and deployed in convention
iii. Complete the following analogy correctly with a word/phrase from paragraph 2:
Delhi: India:: Tokyo :
(Clue: Delhi is capital of India, similarly Tokyo is capital of...)
iv. What theme is being highlighted in the given passage?
(a) Efficient means of reducing garbage
(b) Various techniques to be employed to recycle the goods
(c) Efficient ways of waste disposal
(d) Both (b) and (c)
v. Which of the following notion in the conference has been emphasised to be deployed?
(a) Waste disposal management
(b) Waste management
(c) Recycling of goods
(d) Reshaping Earth’s future
vi. Fill in the blank by selecting the correct option.
According to the pie-chart, Branches & wood has been recycled at the rate of .
(a) 1%
(b) 24%
(c) 4%
(d) 3%
vii. Substitute the word ‘present’ with ONE WORD similar in meaning, in the following
sentence from
paragraph 3:
We provide a platform for young researchers and students to present their research...
viii. List any two points that the given data represents.
ix. Which activity contributes collectively towards paper waste recycled percentage?
x. What has been considered as an efficient means to reduce landfill area?
(a) Sorting of waste
(b) Recycling or composting waste
(c) Both (a) and (b)
(d) None of these

UNSOLVED 2
Read the following passage carefully:
(10 Marks)
India is one of the fastest growing economies in 2020. Poverty is on the decline with close to 44
Indians escaping extreme poverty every minute, according to world poverty clock. According to
Brookings report, today India has 73 million people living in extreme poverty which makes up
5.5% of total Indian population. Two - third people of India live in poverty.68.8%. of the Indian

35
population live on less than Rs160 per day. Over 30% even have less than Rs100 per day and
such people are considered extremely poor.
More than 800 million people in India are considered poor. Most of them live in countryside and
keep afloat with odd jobs. The lack of employment which provides a liveable wage in rural area
is driving many Indians into rapidly growing metropolitan cities, such as Mumbai, Kolkata, Delhi
and Bengaluru.
In 2019, the Indian government stated that 6.7% of population is below its official poverty limit.
According to Oxfam, India's top 1 % of population now holds 73% of the wealth while 670
million citizens, comprising the country's poorest half, saw their wealth rise by just 1%. As per
the Tendulkar poverty line, poverty reduced from 14.9 percentages in 2011 to 7.0% in 2017-
fastest pace the country has ever seen yet. So, 2020 is an extremely challenging year not only for
government and business but also for those who are on the borderline of poverty. Fiscal, monetary
and administrative policies have to be designed in a way that provides a safety for those living at
the brink of poverty. Lifting people out of poverty ought to be on the top of the government’s
priority.
1. As per the graph, which of these is/are the cause(s) of poverty in India?
a) Colonial past b) Lack of education c) Population explosion d) All of these
2. According to the graph, which state has the highest number of people living below
poverty line?
a) Rajasthan b) Bihar c) Madhya Pradesh d) Uttar Pradesh
3.Today, how many people in India live in extreme poverty?
a) About 5 crore b) About 6 crore c) About 7 crore d) About 8 crores
4. What percentage of Indian population live on less than Rs100 per day?
a) 68.8% b) 30% c) 40% d) 60%
5. Based on your understanding of the passage, choose the option that lists the reason for
people migrating to metropolitan cities.
1. Poverty 2. More employment opportunities in metropolitan cities 3. Government
scheme 4. Lack of employment5. Better crops
a) 1,2 and 5 b) 1,2and 4 c) 2,4 and 5 d) 2,3 and 4
6. How many people in our country are considered poor?
a) 10 crore b) 20 crore c) 50 crore d) 80 crore
7. One of the policies that is required to be designed in a way to provide safety for those
living at the brink of poverty is:
a) Fiscal b) Monetary c) Administrative d) All of above
8. According to the Indian Government, what percentage of Indian population were below
poverty line in 2019?
a) 4% b) 5.5% c) 6.7% d) 73%
9. In the line “________ employment which provides a liveable wage” the word ‘liveable’
does not refer to:
a) Good enough b) Acceptable c) Frugal d) Comfortable
10. Choose the option that lists statement that is NOT TRUE.
a) One –third of people in India live in
poverty. b) Corruption
contributes to poverty.
c) Chandigarh has the lowest number of people living below poverty line.
d) Poverty may develop due to limited resources.

36
UNSOLVED 3
Call it a blessing or a curse of Mother Nature, we have to breathe in over 10,000 litres of air in a
day to remain alive. God has wished that we try to keep the air we breathe clean. Several harmful
and noxious substances can contaminate the air we breathe. Generally, much is said and written
about outdoor air pollution, it is most important to recognize that the air we breathe in at home or
in offices can be polluted. It can be a cause of ill-health. The indoor air pollution can lead to
allergic reactions and cause irritation to the skin, the eyes and the nose. It can lead to the
development of fresh breathing problems, especially in those who have allergic tendencies, or it
can worsen the existing respiratory illnesses like asthma and bronchitis.
There can be several sources of indoor air pollution. Tobacco smoke is one of the most important
air pollutants in closed places. “Passive smoking” or environmental tobacco smoke (ETS)
pollution can lead to all the harmful effects of tobacco smoking seen in the smokers in their non-
smoking companions. ETS as a health hazard has been unequivocally proven and is also getting
social recognition now. The exposure of young children to ETS leads to increased respiratory
problems. Several studies, including those done at the PGI, have shown an increased risk of lung
cancer among women exposed to passive smoking.
The next most important source of indoor air pollution is the allergens. House dust mites (HDM)
are very small insects not visible to the naked eye and are the commonest source of allergy in the
house. Modern houses present ample breeding spaces for them in the form of carpets, curtains,
mattresses, pillows, etc. Exposure to HDM can be prevented by the frequent washing of linen in
warm water or exposing them to sun and by encasing the mattresses and pillows in a non-
permeable cover. Pets form an important part of life for some of us. But they can add plenty of
allergens to our indoor atmosphere. They should be kept out of the bedrooms and washed
frequently. To remove the fur particles, one has to use vacuum cleaners as the ordinary broom
and mop are not effective. Moulds, fungi and several other microorganisms thrive in damp
conditions and can lead to allergies as well as infections.
Humidifiers in the air-conditioning plants provide an ideal environment for certain types of
bacteria and have led to major outbreaks of pneumonia. It is important to clean regularly the
coolers, air-conditioners and damp areas of the house such as cupboards, lofts, etc. to minimize
this risk.
Toxic gases can also pollute the indoor environment. Biomass fuels and coal, if burned inside,
can lead to severe contamination by carbon monoxide (CO): The poor quality of stoves and other
cooking or heating appliances that cause incomplete combustion of LPG can also lead to the
emission of CO or nitrogen dioxide. These gases can also cause irritation to the skin or the eyes,
rashes, headache, dizziness and nausea. Other indoor pollutants are toxic chemicals like cleansing
agents, pesticides, paints, solvents and inferior-quality personal-care products, especially
aerosols. Very old crumbling pipes, boilers, insulation or false roofing can also be
important sources. Asbestos is a hazardous product that can cause cancer in humans.
On the basis of your reading of the above passage and the visual input given,
answer the following questions: (10X1= 10)
i. Which area is not much discussed about in terms of air pollution?
(Paragraph 1)
ii. The toxic gases cause …………………. (Paragraph 1)
(a) rashes (b) headache (c) dizziness (d) all of the above.
iii. Air conditioning plants become the cause of ………………… (Paragraph 4)
(a) allergies (b) pneumonia (c) heart attack (d) infection.
iv. Asbestos is a hazardous product because it can
(a) cause cancer in humans (b) cause respiratory problems
(c) prove fatal to the children (d) none of these.
37
v. Which among the following is not a source of Indoor air pollution.
(a) Tobacco smoke (b) Mold spores
(c) House dust mites (d) None of the above.
vi. Mention 2 health hazards caused due to ETS as mentioned in paragraph 2?
1. _________________
2. _________________
vii. Complete the following :
The risk of allergies can be minimized by___________ (Paragraph 3)
viii. How can the toxic gases pollute the indoor environment? (Paragraph 5)
ix. A tiny, microscopic flecks of skin shed by cats, dogs, rodents etc.
(a) Dust mites (b) Pollen (c)Dander (d) All of the above
x. contaminated means:
(a) alloyed (b) impure (c) adulterated (d) All of the above
GRAMMAR
USE OF TENSES (REVISION)
Tenses (verbs)
There are three main tenses:
1. Present Tense
2. Past Tense
3. Future Tense
PRESENT TENSE
Simple present
Usage - It is used:
1. To talk about a present action
2. To talk about a habitual action
3. To talk about a universal truth.
4. To talk about a permanent condition
5. To talk about a scientific fact.
6. To express proverbs
Form
Subject Verb
I, You, We, They, He, She. It Root form of verb, root form +‘s‘ or ‗as‘
(a) Present Continuous
Tense Usage. It is used;
1. To talk about an action that is going on at the time of speaking.
2. For future time reference if we are sure of the action.

Form
Subject Verb
I, You, We, They, He, she. It is present participle
Are +present participle
Is +present participle
(B) Present Perfect Tense
Usage - It is sued to talk about an action that has just been completed.
Form
Subject Verb
I, You, We, They, He, She, It
Have + past participle
Has+ past participle
38
(a) Present Perfect Continuous Tense
Usage - It is used:
(i) To express an action that began in the past and has been in progress till the time of peaking.
(ii)To express an action that finished just a short while
Form
Subject Verb
I, You, We, They, He, she, It
Have been + present participle
Has been+ present participle
Use of ‘since’ and ‘for’
(i) ‘since’ is used to denote - point of time
(ii) ‘for’ is used to denote - period of time
For example,
I have been living in Delhi for five years.
She has been living in Agra since birth.
PAST TENSE
(a) Simple Past
Usage - It is used:
To talk about a past action.
To talk about some past habit
Form
Subject Verb
I, You, We, They, He, she, It
Second form of verb
(b)Past Continuous Tense
Usage - It is used:
To talk about an action, which was in progress at some point of time before the time of
speaking.
Form
Subject Verb
I, You, We, They, He, she, It
Was + present participle
Were + present participle
(c) Past Perfect
Usage - It is used
To talk about an action that was completed before a given moment in the past
Form
Subject Verb
I, You, We, They, He, she, It
Had + past participle
(d)Past Perfect Continuous
Usage - It is used:
To talk about an action that began before a definite moment in the past, had continued up to that
moment and was still in progress at that moment, or had recently finished.
Form
Subject Verb
I, You, We, They, He, she, It
Had been + present participle
FUTURE TENSE
39
(a) Simple Future
Usage - It is used:
To talk about an action that will take place in future
Form
Subject Verb
I, You, We, They, He, She, It
Shall + root form of Verb
Will + root form of verb
Note: In modern English ‘will’ is used with ‘I’ and ‘we’, too.
(b) Future Continuous
Usage - It is used:
To express an action that is almost certain to take place in the future.
Form
Subject Verb
I, You, We, They, He, She, It
Shall be + present participle
Will be +present participle
(c) Future Perfect
Usage - It is used:
To talk about an action completed before a given moment in the future.
Form
Subject Verb
I, You, We, They, He, She, It
Shall have+ past participle
Will have + past participle
(d) Future Perfect Continuous Tense
Usage - It is used:
To talk about an action that will begin before a definite moment in the future, will continue up
to that moment and will be in progress even at that moment.
Form
Subject Verb
I, You, We, They, He, She, It
Shall have been + present participle
Will have been +present participle
Solved Example
1. Fill in the blanks in the following passage by using the correct tense form of the verbs given
in brackets, choosing the appropriate options from the given ones.
I ____________ (1) (realize) that I ___________ (2) (know) one of the two men by sight, and I
_________ (3) (spend) a few seconds thinking why he _________ (4) (seek) me out on a
Sunday afternoon.
During this pause, three small boys __________ (5) (walk) up the passage from the house
behind me, _________ (6) (thread) a way around me and the two men outside, and silently
________ (7) (climb) like cats up into a tree in the middle of the lawn outside. There, the three
figures ________ (8) (rest), _________ (9) (become) immobile, ________ (10) (lie) on their
stomachs, deep in a secret game.

Solution
(1) realized
40
(2) knew
(3) spent
(4) sought
(5) walked
(6) threading
(7) climbed
(8) rested
(9) became
(10) lying
Questions for Practice:
1. Fill in the blanks in the following passage by using the correct tense form of the verbs
given in brackets.
When Albert Einstein ________ (1) (be) three years old, his parents ________ (2) (be) very
worried. He ________ (3) (seem) intelligent, but had not spoken a single word. They ________
(4) (take) him to many doctors who________ (5) (say) that they ________ (6) (find) nothing
wrong with the child. One day, at the dinner table, Albert suddenly said, “The soup _______ (7)
(be) too hot." The shocked parents ________ (8) (ask) him why he ______ (9) (remain) silent for
so long. The young child solemnly replied, "Everything _______ (10) (he) in order until now."
2. Fill in the blanks in the following passage by using the correct tense form of the verbs
given in brackets.
Coffee is a beverage ___1___ (drink) by many past generations of people. What most people do
not realize is that coffee is ___2___ (make) from beans ___3___ (pick) from trees. Coffee trees
___4___ (grow) either from seeds or cuttings. These trees will begin to ___5___ (bear) crops
when they are about four years old. Workers on coffee plantations have to pick the beans by
hand. After the beans ___6___ (gather), they are ___7___ (roast) at high temperatures. This
process ___8___ (give) them their fragrance. The best coffee is made from freshly ___9___
(grinding) beans. There are many different ways of ___10___ (prepare) coffee. Coffee can be
served either with or without milk.
MODALS
Modals are those auxiliary (helping) verbs which express the – ‘mode’ or ‘manner’ of the
actions indicated by the main verbs. They express modes such as ability, possibility,
probability, permission, obligation, etc.
The following are modal auxiliaries:
shall, should, will, would, can, could, may, might, must, ought to, used to, need, dare.
Use of Modal Auxiliaries
(a) Shall and Will
1. ‘Shall’ is used with the first-person subject (I, we) and ‘will’ is used with all person subject,
to express simple future.
2. ‘Shall’ with the first-person subject, expresses:
Intention e.g. I shall go home today.
Promise e.g. I shall try to do better.
Threat e.g. I shall kill him.
Determination e.g. I shall do or die.
3. (a) ‘Will’ with the second- & third-person subject, expresses:
Order e.g. You will do it just now.
Threat e.g. You will be punished.
Promise e.g. You will have a holiday.
Determination e.g. You will work for me.
Intention e.g. Will he go with you?
41
(b) ‘Would’ is used to express:
Determination e.g. I would have my way.
Habitual action e.g. he would sit all day with the book in his hand.
Willingness e.g. I would do my best to satisfy you.
Wish e.g. Would that I were healthy.
(c) ‘Should’ is used to express:
Duty e.g. we should keep our word.
Advice e.g. you should take rest.
After ‘lest’ e.g. Work hard lest you should fail.
Polite request e.g. Should I help you to solve this problem?
(d) ‘May’ is used to express:
Possibility e.g. he may succeed in his work.
Permission e.g. may I come in?
Wish e.g. may you live long.
Purpose e.g. he works hard so that he may pass.
(e) Can is used to express:
Power/ability e.g. He can swim, He cannot speak English
Permission e.g. You can go now. Can I use your bicycle?
(f) Might (past tense of May) is used to express:
Suggestion e.g. he might join a college.
Possibility e.g. it might rain.
Purpose e.g. he works hard that he might win a scholarship.
(g) Could (past tense of can) express:
Power/ability e.g. he could run fast when he was young.
Polite request e.g. could you please help me?
(h) Must express:
Compulsion e.g. you must carry out my orders.
(i) Ought to:
It is used to denote duty or moral or social obligation. It is used nearly in the same sense as
‘should’. The verb that follows ought always takes to +V1 form.
For example:
We ought to obey our teachers.
We ought not to speak ill of others.

(j) Need:
It is used in the sense of ‘require’ or ‘want’. Need not expresses the idea that there is no
compulsion, e.g. You needn’t leave today, you can leave tomorrow. You needn’t pay the whole
amount in one instalment. Need I wait till he comes?

(k) Dare:
It means to have the courage to do something.
e.g. he dares not go there.
(l) Used to +V1:
It expresses a habit in the past.
Used to +V1 e.g. I used to swim when I was young.
I used to live in Mumbai. (Now I live in Delhi).
We used to go to Shimla in winter but now we don’t.
(m) Has to/Have to:
It expresses some compulsion, obligation or necessity in the present or future tense.
42
Has to/have to +V1e.g. Mohan has to finish his work in time.
I cannot come with you because I have to finish my work in time.
(n) Had to:
It is used to express compulsion, obligation or necessity in the past.
The expression ‘had to’ is the past tense of ‘have to’
Had to +V1 e.g. yesterday my servant did not come. I had to wash my clothes myself. The
enemy had to accept defeat at last.
(o) Will have to:
It is used to express compulsion, obligation or necessity in the future.
The expression ‘will have to’ is the future tense of ‘have to’
Will have to +V1
e.g. Next week you will have to deposit your fee.

Modal Verb Expressing Example

must Strong obligation You must stop when the traffic lights
turn red.

logical conclusion / He must be very tired. He's been


Certainty working all day long.

must not Prohibition You must not smoke in the hospital.

can Ability I can swim.

Permission Can I use your phone, please?

Possibility Smoking can cause cancer.

could ability in the past When I was younger I could run fast.

polite permission Excuse me, could I just say something?

43
possibility It could rain tomorrow!

may permission May I use your phone, please?

possibility, probability It may rain tomorrow!

might polite permission Might I suggest an idea?

possibility, probability I might go on holiday to Australia next


year.

need not lack of necessity/absence of I need not buy tomatoes. There are
obligation plenty of tomatoes in the fridge.

should/ought 50 % obligation I should / ought to see a doctor. I have


to a terrible headache.

advice You should / ought to revise your


lessons

logical conclusion He should / ought to be very tired. He's


been working all day long.

had better advice You'd better revise your lessons

Solved Examples
1. ________ you be able to bring your camera when you come?
2. Priya __________ apologise for the confusion that was caused yesterday because of her
carelessness.
3. You _______ finish this first and then start with the other one.
4. ________ I make some tea for you?
5. I ______ make sure to keep everything ready by the time they reach the railway station.
6. You _______ see to it that the students are standing according to their roll numbers.
7. ______ I come in?
8. She _______ find it interesting as she likes reading adventure stories.
44
9. Do you have any idea how much all of this _______ cost?
10. ______ you please help me find my keys?
Answers -
1. Would you be able to bring your camera when you come?
2. Priya should/ought to apologise for the confusion that was caused yesterday because of her
carelessness.
3. You could finish this first and then start with the other one.
4. Shall I make some tea for you?
5. I will make sure to keep everything ready by the time they reach the railway station.
6. You must see to it that the students are standing according to their roll numbers.
7. Can/Shall/May I come in?
8. She might find it interesting as she likes reading adventure stories.
9. Do you have any idea how much all of this would cost?
10. Can/Would/Could/Will you please help me find my keys?
1.Fill in the blanks with appropriate modals.
Ram (a) ……………. keep his word as he (b) ……………. repay all his loan. He (c) …………….
be seventeen next week. I am certain that you (d) ----- see that how successfully he has
accomplished his mission. (e) …………….. he lives long! You (f) ……………. not worry. Your
money is safe.
Answer:
(a) should
(b) has to
(c) will
(d will
(e) May
(f) need
2. Look at the words and phrases below. Rearrange them to form meaningful sentences.
(a) River/when I/could/ I/ swim/was/young/across/the
(b) Parents/their/children/obey/should
(c) Dare/how/you/to him/talk/like/this
3. The following passage has not been edited. There is one error in each of the lines. Write
the incorrect word and the correction against the correct blank number.
Remember to underline the word/phrase that you have supplied.
Incorrect Correction
The evil of begging is very common into our society. E.g. into in
There can be no place which you won’t find a beggar. a) … …
Some beggars are quite health. They are able bodied (b) … --- and are
strong much to earn their living. But they (c) --- ---
are idlers and want to living without work. (d) --- …
SUBJECT-VERB CONCORD
A verb agrees with the subject in number and person. A singular takes a singular verb
and a plural subject takes a plural verb.
I. She is a good speaker.
II. They are good speakers.
In sentence (I) the subject ‘she’ is in the singular number; the verb ‘is’ also in the singular. But
in sentence (II) the subject ‘they’ is in the plural number; the verb ‘are’ is also in the plural.
For example:
(a) Girls are taking a test.
(b) Boys were weeping bitterly.
45
(c) They have not finished the task.
(d) Geeta is reciting a poem.
(e) My sister does not tell lies.
In the above sentences, the verbs have changed according to the person of the subject. Thus, we
know that the verb agrees with the subject in number and person. This is known in grammar as
subject-verb concord.
1. Two or more singular subject joined by ‘and’ takes a plural verb.
For example:
You and I are friends.
2. When two subjects are joined by as well as the verb agrees with the first subject.
For example:
Her friends as well as she are talkative
3. Either, neither, each, every an everyone is followed by a singular verb.
For example:
Each of them is lovable.
4. When two singular nouns refer to the same person or thing, the verb must be singular.
For example:
My sister and friend has come.
5. When two subjects together express one idea, the verb may be in the singular.
For example:
Two and two makes four.
6. When a plural noun expresses some specific quantity or amount considered as a whole, the
verb is in singular.
For example:
Twenty kilos of rice is not much for the month.
7. When two or more singular subjects are connected by ‘with’, ‘together with’, ‘and not’ ,
‘besides’, ‘no less than’ the verb is in singular.
For example:
He and not you is to blame.
8. The verb agrees with the number of the nouns that follow the verb.
For example:
There are five girls in the office.
9. Some nouns which are plural in form but singular in meaning, take a singular verb.
For example:
English is my favorite subject.
10. A plural noun which is the name of a country or province or the title of a book, is followed
by a singular verb.
For example:
West Indies is a black country.
‘Human values’ is written by M.K. Diwan.
11. A collective noun generally takes a singular verb when the subject stands for the collection
as a whole and a plural verb when the subject stands for the individuals of which is composed.
For example:
The whole class is present.
The teams are celebrating their victory.
12. A relative pronoun must agree with its antecedent gender, number and person.
For example:
It is I who am to blame.
Solved Example
46
1. Fill in the blanks appropriately.
(a) Neither you nor your friends…………passed the examination nor is it really shameful.
(b) None of the conspiracy ............. succeeded and this is really great.
(c) All of the food...............been consumed.
Ans.
(a) have
(b) has
(c) has
Questions for Practice
1. Fill in the blanks with the most appropriate answer.
(a)Her friends as well as Rama… ........studying for last one Hour.
(b)Each of you… ............... responsible for the tragedy.
(c) Your cousin and friend........... come.
2. Fill in the blanks with the most appropriate answer.
(a)None of the plans…........ succeeded.
(b)Some of these books… ...... really in bad position.
(c) Some of the water…........ finished.
3. Fill in the blanks with the most appropriate answer.
(a)Most of the money… ......... spent.
(b)There any food left?
(c) All of it................ been eaten.
4. Fill in the blanks with the most appropriate answer.
(a) Bread and butter.............. her favorite breakfast.
(b)Bread and butter .............very costly nowadays.
(c) Twenty kilos of flour.............. insufficient for such a large family.
REPORTED SPEECH-DIRECT AND INDIRECT
What is a Direct Speech?
In a Direct Speech, the speaker’s own words are quoted without making any change.
For example: Ravi said, ―Kartik has gone to Mumbai.
What is an Indirect Speech?
In an Indirect Speech, the essence of the Speaker’s words is given.
For Example: Ravi said that Kartik had gone to Mumbai.
Some Important Points
1. In a Direct Speech-
(a) The Speaker’s actual words are placed within inverted commas (―).
(b) His actual words start with a capital letter.
(c) There is a comma after the reporting verb.
(d) At the end of his actual words, a full stop or a question mark or a sign of exclamation is put
according to the need of the sentence.
(e) The tense of the speaker’s words remains the same.
2. In an Indirect Speech-
(a) Inverted commas are not used.
(b) Only the first letter of the sentence starts with a capital letter.
(c) There is no comma after the reporting verb.
(d) Conjunctions like that, to, if/whether are used to link the reporting verb and the reported
speech.
(e) At the end of the sentence, only a full stop is put. Question marks are not allowed.
(f) The tense of the speaker’s words undergoes a change.

47
Note: If the speaker’s words express a habitual action, a universal truth, a scientific statement, a
proverb or a permanent truth, the tense of the speaker’s words remain the same.
Changing Direct Speech into Indirect Speech
1. Tense and Verb Forms
(a) Simple Present-Simple Past
Direct: Rahul said to me, “I am your neighbour.”
Indirect: Rahul told me that he was my neighbour.
(c) Present Continuous-Past Continuous
Direct: Sanjay said, “I am eating a mango”.
Indirect: Sanjay said that he was eating a mango.
(d) Present Perfect- Past Perfect
Direct: Sanjay said, “I have drunk an apple juice”.
Indirect: Sanjay said that he had drunk an apple juice.
(e) Present Perfect Continuous – Past Perfect Continuous
Direct: Sanjay said, “I have been watching TV since morning”.
Indirect: Sanjay said that he had been watching TV since morning.
(f) Simple Past – Past perfect
Direct: Sanjay said, “I did my work”.
Indirect: Sanjay said that he had done his work.
(g) Past Continuous- Past perfect Continuous
Direct: Sanjay said, “I was writing a letter”.
Indirect: Sanjay said that he had been writing a letter.
2. Change of Time, Place
Words showing nearness of time and place change into words showing distance, time and place.
Now-then
Ago-before
This-that
These-those
Here-there
Today-that day
Tomorrow- the following day/the next day
Yesterday-the previous day/ the day before
Last night- the previous night
Last week- the previous week
Will- would
May-might
Can-could
3. Change of Pronouns
S - First person according to subject
O - Second person according to object.
N - Third person no change.
4. Change of Different Kinds of Sentences
(a) Questions
(I) “Wh‟ questions: The questions beginning with which, when, where, why, how, whose, etc.
do not need any conjunction.
For example,
He said to me, “Which movie were you watching last night?” (Direct)
He asked me which movie I had been watching the previous night. (Indirect)

48
(II) Yes/No questions: The question that begin with helping verbs like is, am, are, was, were,
will, shall, do, does, it, has, have, had, etc., that the conjunction ‘if’ or ‘whether’.
For example,
The teacher said to children, “Have you completed your work?” (Direct)
The teacher asked the children if they had completed their work. (Indirect)
(b) Commands:
(i) The imperative sentences or commands take the conjunction ‘to’.
(ii) The reporting verb ‘said’ or ‘said to’ changes into ordered, commanded, advised or
requested.
For example:
The doctor said to the patient, “Take this tablet daily”. (Direct)
The doctor advised to patient to take that tablet daily. (Indirect)
The master said to the servant, “Bring me a glass of milk.” (Direct)
The master ordered the servant to bring him a glass of milk. (Indirect)
(c) Exclamations:
(i) The reporting verb ‘said’ is changed into exclaimed, exclaimed with joy exclaimed with
sorrow, exclaimed with wonder, wished or prayed etc.
(ii) The conjunction ‘that’ is used to introduce the reported speech.
For example,
He said, “What a beautiful painting!” (Direct)
He exclaimed with wonder that it was a beautiful painting. (Indirect)
The boys said, “Hurrah! We have won the match.” (Direct)
The boys exclaimed with joy that they had won the match. (Indirect)
(d) Sentences beginning with “Let”:
(i) The man said to the servant, “Let the children play.” (Direct)
The man asked the servant to let the children play. (Indirect)
(ii) She said to her friend, “Let us go for a picnic.” (Direct)
She proposed to her friends that they should go for picnic. (Indirect)
(iii) The Principal said to the peon, “Don’t let the children enter my study.” (Direct)
The Principal ordered the peon not to let the children enter his study. (Indirect)
Solved Example
Read the conversation below and complete the passage that follows.
Interviewer: So, why do you want to be a computer programmer?
Ravi: Well, I have always been interested in computers.
Interviewer: I see. Do you have any experience?
Ravi: No, but I’m a fast learner.
Interviewer: What kind of a computer do you use?
Ravi: Computer? Uhm, let me see. I can use a Mac. I also used Windows 10 once.
Interviewer: That’s good.
Ravi recently attended an interview for the selection of a computer programmer. At the interview,
he was asked (a) ……. To this question he replied that he wanted to change his job because (b)
……. When the interviewer asked him (e) ……… he replied that he (d) ………. Finally, the
interviewer wanted to know (e) ………. Ravi replied that he could use a Mac and had also used
Windows 10 once in the past. The interviewer seemed to be pleased with his answers.
Answer:
(a) why he wanted to be a computer programmer
(b) he had always been interested in computers
(c) whether he had any experience
(d) didn’t but that he was a fast learner
49
(e) the kind of computer he used
Practice Questions
1. Rearrange the following sentences to make meaningful sentences.
(a) Friends/Archie/movie/the/asked his / what / of they / thought/
(b) preferred/said/book/had/the/his/friend/he
(c) his/was/same/Ram/answer//always/said/the
2. Read the dialogue given below a report it by filling in the blanks
Question 2.
Manu: Where are you going to?
Annu: I am going to the market. Do you want anything?
Manu asked Annu (a) …………………… Annu replied (b) …………………… Annu replied
(b) …………………… and she further asked (c) ……………………
Answer:
(a) where she was going.
(b) that she was going to the market
(c) if/whether she wanted anything.
DETERMINERS
Determiners are words placed before nouns or pronouns to decide or fix their meaning. They
tell us ‘how many’ or ‘how much’
Determiners are words which come before nouns.
They contain several classes of words, including pronouns and adjectives.
They determine or limit the noun by giving some additional information about it.
Determiners show whether a noun refers to is a general or a specific object, person or place.
They indicate which or how many things the noun refers to.
Determiners define or limit a noun to the singular or plural.
They indicate amount or quantity.
Determiners may precede numerals too.
Solved Exercises
1.Fill in the blanks using suitable determiners.
Fill in the blanks using this or that.
1. ………….. is my wife.
2. Could you bring me a book …………… I left in the garden?
Answer:
1. This
2. that
Fill in the blanks with Numeral Determiners—one, two, some, any, little, few, all, both,
much, many, several, etc.:
1. How …………. money do you want?
2. Have you read …………. stories?
3. I have read…………short stories in Hindi but only …………. in English.
4. I read the letter again and noticed that there were…………..mistakes in it.
5. How …………. milk do you take every day?
6. There are…………. countries in the world where the population is not growing fast.
7. Would you lend me your watch for ……… days?
8. Do you have …………. complaint against the clerk?
9. There are …………. eggs in the basket, but there isn’t …………. butter.
10. ………….. care could have prevented the accident.
11. ………….. of the boys has broken this window pane.
12. I have already spent ………….. rupees you gave me.
50
13. ………….. the husband and wife are members of this club.
14. ………….. the customers insisted that the shopkeeper must provide them full measure.
15. The story was published in ………… the local newspapers.
Answers:
1. much
2. many/some
3. many/a lot of; few/some
4. many
5. much
6. a few
7. some/a few
8. any
9. some, any
10. A little
11. one
12. few
13. Both
14. All
15. all
3) Fill in the blanks with suitable determiners:
1. There was …………. accident near …………. central market this morning. …………..
car hit …………. scooter and …………. man on …………. scooter was killed.
2. …………. postman put …………. letter under …………. front door just …………. hour
after you had left.
3. A cup of tea may seem …………. ordinary thing to …………. of us in India. But drinking
tea is …………. ritual which is taken very seriously in …………. Japanese homes.
4. ………….. king fought bravely but he was defeated by …………. enemies. …………..
defeated warrior left …………. battlefield and hid in …………. is cave.
5. ………….. people came to …………. site where the revolutionary general was to be
hanged. When asked if he had …………. desire, …………. smile ran on …………. dry
lips and he nodded his head.
6. It was …………. grand occasion for …………. children …………. of them had won
prizes and …………. parents felt proud …………. of them introduced their parents to their
teachers.
7. Game hunting was …………. favourite pastime of …………. officers of …………. British
Raj. This resulted in decrease in …………. number of animals in …………. wild. Recent
environmental degradation and destruction of natural habitats of wild animals have pushed
them to …………. brink of extinction.
Answers:
1. an, the, A, a, the, the
2. The, a, the, an
3. an, some, a, many
4. The, his, The, the, a
5. Many, the, any, a, his
6. a, many, some, their, Some/A few
7. a, the, the, the, the, the
Exercises for Practice:
1. Fill in the blanks with suitable determiners: (a, an, the, some, yours, much, etc.)
Two friends meet in their school canteen. They are going to have their lunch.
51
Namit: You can share my lunch and I’ll share (a) …………………….
Vipul: Oh, that’ll be nice. I’ll have (b) ………………… good things to eat.
Namit: I’m afraid. I don’t have (c) ……………………. to eat. There are only two sandwiches
and (d) ……………………. apple. Here’s your sandwich. And you can take half (e)
……………………. apple.
Vipul: Thanks. They look delicious. I have four puries and some pickles.
We can have two puries each. And we will order for (f) ……………………. drink. What would
you like tea or coffee?
Namit: I’ll have tea.
Vipul: So, shall I.
2. Complete the following passage by filling in suitable determiners:
My aunt lived on (a) …………………. ground floor of (b) …………………. old house on (c)
…………………. river Thames. She was afraid of burglars and always locked up (d)
…………………. house carefully before going to bed. She also took the precaution of looking
under (e) …………………. bed to see if (f) …………………. burglar was hiding there.
3. Fill in the blanks with suitable determiners:
(a) …………………. abbreviation is (b)…………………. shortened form of
(c)…………………. word or a group of words. It is used to save time and space
(d)…………………. abbreviations are also used while speaking. Most of
(e)…………………. dictionaries include (f)…………………. list of commonly used
abbreviations.
4. Complete the passage with suitable determiners:
My friend lived on (a)…………………. first floor of (b) …………………. recently
renovated old house on (c) …………………. river Beas. She was afraid of burglars and
always locked up (d) -------- house carefully before going to bed. She also took the precaution
of looking under (e)…………………. bed to see if (f) …………………. burglar was hiding
there.
5. Complete the following passage with suitable determiners:
I have met (a) …………………. people in my life but (b) …………………. man, who has
influenced me (c) …………………. most is Mother Teresa. To me she has been (d)
…………………. symbol of true religion (e) …………………. people can do what she has
done. If we adopt (f) …………………. ideology, the world would become a better place to live
in.
EDITING
1) Following paragraph has not been edited. There is an error in each numbered write the
correction against the correct blank number. Remember to underline has been done as an
example. (4 marks) (Board Term-12012, Set EC2,059)
When it comes to nutrients, almonds really know
how to shine! This all in one nut was snacked Error: was; Correction: is;]
with multiple benefits who help in weight a) [Error: _______; Correction: ______
management, maintain blood glucose levels b) [Error: _______; Correction: -------
and providing essential nutrients. Almonds is c) [Error: _______; Correction: -------
an excellence source of vitamin-E, magnesium, d) [Error: ______; Correction: _____
and manganese.
Answers -
Error Correction
a) who which
b) maintain maintaining
c) is are
52
d) excellence excellent
2) The following passage has not been edited. There is one error in each of these lines.
Write the incorrect word and the correction in your answer sheet. The first one has been
done for you as an example. (4 marks) (Board Term-12012, Set EC2,032)
God like qualities are planted with us at birth. Error: _with; Correction: in_
They grow amid the thorns of our fault. Many of us (a) Error: ___; Correction: ___
look at ourselves and see only the thorn, the defects. (b) [Error: ____; Correction: __
We despair, thinking such we cannot do anything good (c) [Error: ___; Correction: ___
We neglect to water a good within us, (d) [Error: ____; Correction: ___
and eventually it dies.
Answers –
Error Correction
a) fault faults
b) thorn thorns
c) such that
d) a the

LETTER WRITING

A letter is the most common and convenient method of expressing our thoughts and
opinions. it is an important mode of communication. There are two types of Letters.
Formal Letters and Informal Letters.
Formal letters: These letters are written in formal simple and polite language. These follow a
certain format. Such letters are written for official purposes to authorities, colleagues, seniors,
etc. Types of formal letters:
Formal letters can be broadly classified into four types. Complaint letters (business
/official) Enquiry letters Letter for placing or cancelling order Letter to the Editor

Parts of a Formal Letter


An effective formal letter has the following parts:
1. Sender’s Address
It is the address of the writer .it is written at the top left-hand corner of the page. if the address
consists of several parts, each part should be written in a separate line.
For example
House no 21,
Mangal Pandey Nagar Mumbai -232344
Note: don’t put a comma at the end of each line
2. Date
Either of the given formats can be used to write the date

15 January 20 xx or January 15, 20xx.


Note: Do not use abbreviations like Jan, Feb. Do not write 19 instead of 2019.
3. Receiver's Address
All official letters are addressed to the authority/ post of the person. Hence, we write the
receiver's address after addressing the official.
For example,
The Editor
The Times of India New Delhi-110001
53
4. Subject
It expresses the main theme or objective of the letter clearly. It must be as brief and concise as
possible.
5. Salutation
It is the greeting to the person to whom the letter is addressed. In the official letter, we use Sir
/Madam or
Dear sir /Dear madam
Note: When writing the salutation, we have to keep in mind the gender of the receiver if
specified in the question.
6. Body of the Letter
Contains all the information that the writer wants to convey. The body includes three main
parts.
i. An introductory or paragraph or sentence - it states the purpose of writing the letter.
ii. Informative paragraph - Gives details of the problem. it causes, effects, possible
solutions, etc.
iii. Concluding paragraph /sentence it states your hopes, comments, request
suggestions
7. Complementary Close
Is a courteous way of ending the letter? We can write. Yours sincerely - Yours faithfully -
Yours truly.
Note: the first letter of the second word here (s /f / t) is never written in the capital do not use
Yours instead of yours.
8. Signature
This is the sender’s name. If applicable, the sender’s designation may be added below the name.
Letter to the Editor
These are the letters written to the editor of a newspaper, magazine, or any other regularly
printed publication. Their purpose is to highlight a social issue or a problem, an ideal letter to
the editor should.
• Grab catches the reader’s attention.
• Explain the topic/ issue of the letter at the start
• Explain why the issue is important
• State the writer’s opinion about what should be done
• Be brief.
• Express the writer’s hope for the resolution of the problem

1. You are Nitin/ Nipun of H.No. 123/8, KG Nagar, New Delhi. You are concerned about
the consequences of reckless driving. Write a letter to the editor of a national daily, expressing
your concern. (100-120 words)
H.No. 123/8,
KG Nagar
New Delhi

20 August 2022 The Editor

The Indian Express


New Delhi

Sir
Subject: Consequences of reckless driving.
54
Through the columns of your esteemed newspaper, I would like to draw the attention of the
public to the increasing number of road accidents due to reckless driving.
The number of accidents is increasing day by day as drivers violate the traffic rules. Although the
regulations regarding speed limits do exist, many do not pay heed to them. People, who try to
cross the road become victims of rash driving. Some accidents even turn out to be fatal. Rules of
the road are hard and fast and must be observed strictly. Especially the regulations regarding
speed. Negligence would often be a menace to life and limb.
It is expected that the authorities must enforce the regulations regarding the speed limit. Severe
punishments should be imposed on those who violate them. Otherwise, this would go unabated
resulting in the injury/ death of innocent people.
Yours truly
Nitin

2. The youngsters today are greatly influenced by social networking sites. They live in the
‘reel world’ rather than the real world. Write a letter to the editor of The Hindu, expressing your
concern. You are Vishal/Vaishnavi of 12 Vikas Nagar, Kolkata.

12 Vikas Nagar
Kolkata
20 August 2022
The Editor
The Hindu
Kolkata

Sir
Subject: Influence of social networking sites on youngsters.

Through the columns of your esteemed newspaper, I would like to voice my concern over the
influence of social networking sites on youngsters.
There is no denying the fact that social networking has brought the world closer and has helped
people to stay connected. Although the advantages are many, their disadvantages, especially for
the youngsters cannot be overlooked. When electronic devices became easily available during
the pandemic, every teenager got access to social networking sites. Youngsters are more
concerned about the ‘reel world’. They try to impress followers on social media almost
neglecting the real world.
Many youngsters have become addicted to them which not only affects their studies but has a
severe effect on their health as well. Health issues like eye strain, stress, and anxiety are
common these days. Many youngsters fall prey to cyberbullying and privacy-related issues
which prompts them to take some drastic steps.
This is an important issue to be resolved. Children must be educated about the negative impact
of social media and be made aware of cyber laws. Also allotting screen time and following that
strictly would avoid the menace to a good level.

Yours truly
Vaishnavi

55
Letter of inquiry
An inquiry letter is usually written to gain some information from the recipient. While
writing an inquiry letter it is important to provide some information about yourself so
that the recipient can authenticate the source of the inquiry.
Suggestions to write an inquiry letter:
· Give a brief introduction about yourself. Your organisation.
· Provide the details about the thing or subject of inquiry.
· If you require a detailed information, write your questions accordingly.
· If you require the information by a particular date, then do mention in the letter.
· Thank them for their time and consideration at the end.
Format:
Address Given
Or
Examination Hall
Date
The Principal
Name of Institute
Address
Dear Sir/Madam
Subject: Inquiry regarding …………………….. .
Body of letterPara I – After reading your advertisement regarding …………………….. . I wish
to make certain inquiries.Para II –
· Duration of course
● Fee structure
● No. of students per batch
● Transport facilities
Para III – I will be grateful if you could send me the brochure along with the enrolment form so
that I could register in the course at the earliest.
Complimentary Close

Question 1.
You are Anshul/Anshika, a student of class X and resident of 56 D, Ring Road, ITO, New
Delhi, and wants to be a choreographer. Write a letter to the director, National Institute
of Choreography, Noida, seeking information about their course, admission procedure,
eligibility criteria and other necessary details.

56 D, Ring Road, ITO


New Delhi 1100XX

1 July 20XX

The Director
National Institute of Choreography
Sector 16, Noida 102XXX
Dear Sir

Subject: Inquiry regarding course in choreography


56
Refer to your advertisement regarding the courses in choreography offered by your reputable
institute, I want to state that I am currently in X class and preparing for my final exam. I am
very much interested in dancing and want to take it as a career. I am also given to understand
that this institute is by far the best so far as choreography is concerned and I would very much
like to be part of it. Kindly send me the prospectus and the application form. I would be highly
grateful if you could provide me with the following information:
· The department and programme faculty
· Funding opportunities
· Scholarships available
· Admission procedure
· Eligibility criteria
· Hostel facilities
Kindly send me, the brochure along with the enrolment form at the earliest so that I could
register myself for the course.
Q2. Yours faithfully Anshika You are Rama/Ramesh of D–105, Lajpat Nagar, New
Delhi. You want information about German Language Courses at German Embassy,
Chankya Puri, New Delhi. Write a letter to the Director inquiring about the same.

D–105, Lajpat Nagar


New Delhi 1100XX

12 August 20XX

The Director
German Embassy
Chankya Puri
New Delhi 1100XX

Dear Sir/Madam

Subject: Inquiry regarding German courses


I wish to make certain enquiries about the German language courses offered by your institution.
I have just completed class X and want to pursue my career in German. I would like to know
the duration of the course, the fee structure and the transport facilities available. I have always
had a flair for language and have wanted to be a multilingual.
Hence the desire to learn the German language. I will be grateful if you could send me the
brochure along with the enrolment form enabling me to register myself for the course at the
earliest. Please also find enclosed with the letter a draft of 200/– for the brochure. Any balance
money shall be paid on receipt of the same.
Yours faithfully
Rama

Q3. You are interested in a short–term course in computer graphics during the
holidays. Write a letter to the Director, Fortune Graphics Computers, inquiring about
their short–term courses and asking for all the necessary details. You are Anuj/Anju of 28
A, Uttam Nagar, New Delhi.

28 A, Uttam Nagar
57
New Delhi 1100XX

12 July 20XX

The Director
Fortune Graphics Computers
Laxmi Nagar
New Delhi 1100XX

Dear Sir/Madam
Subject: Inquiry about the short–term courses in computers
I came across your advertisement in the Times of India dated 11th July. I would like to know in
detail about the courses offered by your institute. The advertisement was a comprehensive one
and offered a variety of courses. Since it’s the age of technology, my interest is in line with it. I
am a class X student and wish to pursue a course in computer graphics during the summer
vacation after my board exams. Kindly send all necessary details, including the courses offered,
its duration, fee structure, etc. at the above address. The payment for the brochure shall be made
at the time of the delivery. Thanking you in anticipation.
Yours faithfully
Anuj
Letter of Complaint
1. You had bought a mobile phone from a reputable showroom in Mumbai but after a
month it developed a serious fault. Write a complaint letter to the manager asking for
immediate repair or replacement of the same. You are Sadiq/Razia, 123, TT Nagar,
Bhopal.
123, T.T. Nagar
Bhopal

30 September 20XX

The Manager
Shiv Ram & Sons
Mumbai

Dear Sir
Subject: Complaint against a defective mobile phone

I bought an LG Nexus 5 from your show–room vide receipt number 123 dated 15 September
20XX. The phone has a warranty of three years vide Guarantee Card number 123 valid upto
30th August 2021. I regret to inform you that the phone has developed a technical snag and
stopped working. In fact, just a few days after its purchase, it had started giving me trouble off
and on but I related it to the connectivity issue or some minor fault that I thought would settle
with time. Since it is still in warranty period, you are requested to replace it immediately.
Needless to say, that mobile phones are not a luxury but a necessity these days. An early and
appropriate action will be highly appreciated.
Yours faithfully
Razia
2. You live in a crowded area in Ghaziabad, Uttar Pradesh. Unauthorized parking of
vehicles in your area is causing a lot of inconvenience to the locals. Write a letter to the
58
police commissioner district complaining against this practice. You are Akash/Akansha of
Kavi Nagar, Ghaziabad.
24, Kavi Nagar
Ghaziabad, Uttar Pradesh 201 XXX

25 January 20XX

The Police Commissioner


Ghaziabad 201XXX

Dear Sir
Subject: Unauthorised parking in Kavi Nagar

I would like to draw your kind attention towards the problem of unauthorised parking of
vehicles in Kavi Nagar area. The locals have lodged various complaints to the local police
station but all in vain. This is one of the most crowded market places of the area where a proper
parking place is required. The unauthorised parking causes a lot of inconvenience to the
residents. Narrow streets are blocked causing accidents and frequent road rages. Not a day
passes when there are no physical fights, use of abusive language or occurrence of untoward
incidents. The cars could be towed away from the unauthorised lot and heavily fined. A stern
action on your part would truly be appreciated. You are, therefore, requested to direct the local
police to monitor the parking in the area and do the needful.
Yours faithfully
Akash
3. You are Raman/Reena of 240, Shahid Nagar, Agra. Write a letter to M/s Raju Sales
Corporation complaining about the poor performance of the washing machine purchased
from their showroom, mentioning clearly the specific problem you are facing and the
action you want. Give all the relevant details.

240, Shahid Nagar


Agra

25 January 20XX

The Manager
M/s Raju Sales Corporation
Agra

Dear Sir
Subject: Complaint about defective washing machine

I bought an LG washing machine from you Shahid Nagar Branch on 1 January 20XX vide cash
memo number 123. It has a warranty of two years. I regret to inform you that it has developed a
technical snag in the first month itself. Its water dispenser is not working properly. The dryer
and water outlet are choked and sometimes it gives electric shock too. It’s a matter of grave
concern that a brand-new machine should develop so many defects in the first month of its
purchase.

59
It is not only disappointing but shocking too as the electric shocks could put the user’s life at
risk. Since it is still under warranty, you are, therefore, requested to replace it at the earliest. I
sincerely hope that such cases are few and far between and do not get repeated.
Yours faithfully
Raman

Letter of Placing Order


Q.1 You are Sakshi/Saksham, Hostel Warden, Radha Krishna Public School, Vrindavan,
Uttar Pradesh. Write a letter to the Sales Manager, Bharat Electronics and Domestic
Appliances Ltd., New Delhi, placing an order for fans microwaves, ovens and geysers that
you wish to purchase for the hostel. Also ask for discount permissible on the purchase.

Radha Krishna Public School


Vrindavan
Uttar Pradesh XXXXX

18 May 20XX

The Sales Manager


Bharat Electronics and Domestic Appliances Ltd.
New Delhi 1100XX

Dear Sir
Subject: Placement of order for electronics

We are happy to place the order for the following items. Kindly send the following items at the
above. address through transport carefully.
Name of the items No. of items Brand

Fans 50 Bajaj

Microwaves 25 L.G

Ovens 30 Philips

Gysers 25 Nova

All the items should be in good condition, well bound and packed properly. The items must be
delivered by the end of the month of the issue of this letter. Any damage during transportation
would be your responsibility. As in the past also, we have never been given any opportunity to
complain and the goods have always reached as well on time, and in excellent condition as per

60
our specifications. We do expect the same delivery this time as well. Kindly offer us a suitable
discount as has been the practice all these years.
Yours faithfully
Saksham
Hostel Warden
Q.2 Write a letter to Delhi Sports, Daryaganj, New Delhi, placing an order for sports
articles like footballs, cricket balls, Tennis balls and cricket bats to be supplied to your
school. Sign as Ravi/Raveena, Sports Secretary.

Happy Time Public School


Bhajan pura
New Delhi 1100XX

19 January 20XX

The Manager
Delhi Sports, Darya Ganj
New Delhi 1100XX

Dear Sir
Subject: Placement of sports goods
This has reference to the quotation dated 19 December 20XX. Kindly send the following items
at the above address.
Name of the items No. of items Remarks

Football 10 Sparton

Cricket balls 10 Leather

Tennis balls 20 Vicky(brand)

Cricket bats 10 SS

All the items should be in good condition, well bound and packed properly and delivered within
a week. The payment will be made by cash once the items reach us. Any damage during
transportation would be your responsibility. Though in the past, you have never given us any
opportunity to complain and – the goods have always reached us well on time, and in excellent
condition as per our specifications, we do expect the same delivery this time as well.
Yours faithfully
Raveena
(Secretary)
Q.3 You are Rakesh/Sonia of Cambridge Senior Secondary School, Noida. Write a letter
to the Sales Manager, Orient Blackson, Noida, placing an order for some books for your
school library.
Cambridge Senior Secondary School

61
Noida

18 September 20XX

The Sales Manager


Orient Blackson
Noida

Dear Sir
Subject: Supply of books for the School Library

Please refer to your quotations and booklist dated 2 September 20XX. I am pleased to place
an order for the following books for the current session.

Name Class Quantity

1. First Flight (English) 10 30

2. Footprints Without Feet 10 30

3. The Story of My Life by Helen Keller 10 25

4. Diary of a Young Girl by Anne Frank 10 25

You are requested to send the above books as per the terms and conditions at the earliest. I
do hope and expect the books will reach us in good condition, well–bound and packed
properly, and the discounts offered in the past shall be applicable this year too.
Thanking you
Yours faithfully
Rakesh

Analytical Paragraph Writing Format


An analytical paragraph shall be divided into three parts-
1. Introduction
2. Body of the paragraph and;
3. Conclusion

The introduction
The introduction is the first paragraph that should describe in brief what the graph is
about. It should be like an opening paragraph that introduces the reader to the context
of the chart given. When writing the introductory paragraph, you need not go into the

62
details. You just need to mention what is clearly evident from the chart or the graph
given in the question. It is best to write the introduction in one or two lines.

2. The body of the paragraph


This part of the analytical paragraph should contain details of the graph/chart given in
the question. It should contain all the important information. It is extremely important
to choose the significant details that should be included in the paragraph.
The body can be broken into two or three sub-paragraphs depending on the
information extracted from the graph. Breaking the body into subparagraphs makes it
easy for the reader to understand.
i. Organizing information: You should look for the large differences that are very
prominent. You can mention the aspect which is the smallest or the largest in the data
given. Any aspects that are similar should be mentioned next. You should further
mention about the aspect that has remained unchanged or constant throughout.
ii. Comparing information: If the chart mentions two different parties, for example,
men and women, refer to both of them in the analytical paragraph. If there is any
contrast or comparison that could be drawn, you must present it effectively. If the
chart is about a particular time period or highlights trends of several years, mention
each year in a concise manner. If any comparison or contrast can be done for two or
more years, you should write about it too.
iii. Mentioning quantities: When including numbers and figures, their accuracy must
be ensured. There are a number of ways that can be used to describe quantities, for
example- percentage, fraction, ratios, etc.
iv. Connecting sentences: If different information or ideas are there, you should use
connectors or linking phrases to link them logically. The sentences of the paragraph
should be sequential and connected rationally.

3. The conclusion
The last paragraph should summarise the idea mentioned and the information in
general. It should be concluding in nature and act as the closing statement. One should
keep in mind that one must not include any personal opinions, conclusions, or
observations. You should simply stick to the facts.
You need not dispose of the information to write a good analytical paragraph. The key
is to choose wisely the important information, organize it well, state correct facts and
summarise it properly.
Useful tips for writing an analytical paragraph
1. For an introduction, you can start with the following phrases-
1. The chart given above describes
2. The table suggests
3. The line graph shows
4. The data given provides information about
5. The pie chart illustrates, etc.
2. For describing trends, use phrases and words like- a pattern of growth, rapidly
doubled, skyrocketed, striking increase, peaked, soaring rates, declined, plummeted,
levelled off, stagnated, fluctuate, starting to rise, starting to fall, drop down, slightly,
etc.
3. For describing quantities, use various styles like- 48% of, one-third of, nearly one-
fourth of, almost 80%, majority, on average, twice as much, almost equal, the highest,

63
the lowest, very close to 2%, roughly, approximately 5% of, just under three percent,
etc.
4. For establishing a relationship or contrast, use phrases and words like- relationship
between, similarly, in contrast with, in comparison to, but in the opposite case,
however, whereas, when it comes to, as opposed to, while, striking difference,
noticeable difference, etc.
5. For the conclusion and other connecting phrases use- overall, subsequently, in all, in
a nutshell, for the chart given, in short, striking changes, including, therefore, etc.
Question 1: Below is a graph given showing birth and death rates in a country from 1901
to 2101. Write an analytical paragraph (100-150 words).

The graph shows birth and death rates starting from 1901 till 2101.
Since 1901, the birth rate has remained more than the death rate until 2041. Birth rate was
20000 in 1901 and started increasing gradually. It peaked in 1961 to around 65000. From 1961,
birth rate has fluctuated multiple times between 50000 and 60000. It is expected to decline for
the coming years reaching approximately 42000 by 2101.On the other hand, the death rate
stood at around 10000 in 1901 and then has increased steadily. It is expected to rise strikingly
from 2021 before levelling off to approximately 60000 between 2061 and 2081. The graph
indicates a slight decline in deaths in the year 2101.The graph shows the huge gap between
birth rate and death during 1961 to 2001. However, this gap is expected to reduce in the later
years. Overall, as opposed to the prevailing trends, the death rate will be more than the birth
rate in the later half of the 21st century.

Question 2: Below given are three pie charts showing consumption habits of India,
China and World overall in 2008. Write an analytical paragraph describing the pie
charts (100-120 words)

64
The three given pie charts depicts types of food consumption in 2008 as compared in two
countries- India and China. In general, processed food is the major type of food eaten.
Processed food comprises as high as 41% of the global consumption. In China and India as
well, processed foods are consumed the most, the rate of consumption being 34% and 39%
respectively. On an average, vegetables and fruits constitute 29% of consumption. In China,
vegetables and fruits comprise 32% of food consumption which is 9% more than that of India.
For Indians, animal food accounts for 27% of their diet, exceeding Chinese food consumption
by a striking 12%. However, nuts and seeds comprise almost one-fifth of Chinese food habits.
This is way more than the global consumption of 4% and 11% of India.
Overall, world food consumption largely comprises processed food. Chinese consumption of
nuts and seeds is unbelievably higher than the average percentage.

Question 3: The following table shows details about the internet activities for six
categories for different age groups. Write an analytical paragraph for the table given in
around 150-200 words.

65
The given table suggests the internet activities of seven age groups ranging from teens to those
in their seventies for six different kinds of activities. The table shows that the younger
generation is more interested in online games and news, while the older generation spends
time on the internet to research and buy products.
It is evident from the table that teens mainly use the internet for games (as high as 81%), news
and downloads and are interested in searching for people or friends or doing any product
research. The middle-age group (people in 20s to 60s) is highly interested in getting news,
doing product research and buying products, the percentage ranging from 70-80%. The
internet activity which gets the least time is searching for people. All the age groups spend less
than 30% of their internet time on the same. The amount of time spent on downloads decreases
with age and gets as low as 6% (for people in 70s).
Overall, the table suggests that teenagers are most likely to spend time playing games and
doing downloads. On the other side, older people are interested in researching and buying
products. People spend the least amount of time searching for other people online.

Question 4: The pie chart shows the proportion of people from different households
living in poverty in the UK in 2002. Write an analytical paragraph to describe the
information in 100-120 words.

The given pie chart illustrates seven different categories of households living in poverty in the
UK in 2002.
It is clearly evident from the pie chart that 26% of the total poverty-stricken households are
those of sole parents. Single people without children account for the second highest proportion
with 24%. In contrast to couples without children that accounts for just 9%, couples with
children account for 15% of the poor households. Single aged persons and aged couples’
proportion for 12% together for poor households.
Overall, 14% of all households in the UK were living under poverty. The younger generation
had a greater poor percentage than their aged counterparts. Couples without children had better
economic conditions than those with children.

LITERATURE SECTION
CLASS 10 (2022-2023)
Literature (prose) = The first flight

66
A Letter to God
Extra Questions and Answers
Read the following passages and answer the questions that follow:

Question 1.
The house—the only one in the entire valley—sat on the crest of a low hill. From this height
one could see the river and the field of ripe corn dotted with the flowers that always promised
a good harvest. The only thing the earth needed was, a downpour or at least a shower.
Throughout the morning Lencho— who knew his fields intimately—had done nothing else
but see the sky towards the north-east. “Now we’re really going to get some water, woman.”
The woman who was preparing supper, replied, “Yes, God willing”.
(i) Where was the house located?
(ii) What did the field of corn dotted with flowers promise?
(iii) What did the earth need?
(iv) Pick out the words from the passage which mean the same as
(a) Peak (b) Rain (c) Food
Answer:
(i) The house was located on the crest of a low hill.
(ii) The field of corn dotted with flowers promises a good harvest.
(iii) The earth needed a downpour or at least a shower.
(iv) (a) crest (b) downpour (c) supper

Question 2. With a satisfied expression he regarded the field of ripe corn with its flowers,
draped in a curtain of rain. But suddenly a strong wind began to blow and along with the rain
very large hailstones began to fall. These truly did resemble new silver coins. The boys,
exposing themselves to the rain, ran out to collect the frozen pearls.
(i) Who is ‘he’ in the passage?
(ii) What did it suddenly change?
(iii) What resembled new silver coins?
(iv) Why did the children run out?
Answer:
(i) Lencho is ‘he’ in the passage.
(ii) Suddenly, a strong wind began to blow and along with the rain very large hailstones began
to fall.
(iii) Hailstones resembled the new silver coins.
(iv) The children ran out to collect the frozen pearls
Question 3. It’s really getting bad now,” exclaimed the man. “I hope it passes quickly. “It did
not pass quickly. For an hour the hail rained on the house, the garden, the hillside, the cornfield,
on the whole valley. The field was white, as if covered with salt. Not a leaf remained on the
trees. The com was totally destroyed. The flowers were gone from the plants. Lencho’s soul
was filled with sadness. When the storm had passed, he stood in the middle of the field and
said to his sons, “A plague of locusts would have left more than this. The hail has left nothing.
This year we will have no com.”
(i) What was really getting bad?
(ii) Why did he wish for it to pass quickly?
(iii) Why will they have no corn that year?
(iv) Why was Lencho’s soul filled with sadness?
Answer:
(i) The continuous falling of hailstones was getting really bad.
67
(ii) He wished it to pass quickly because it was not good for his crops.
(iii) They would have no com that year because the com in the field was totally destroyed by
the hailstones.
(iv) Lencho’s soul was filled with sadness because his total com was destroyed.

Short Answer Type Questions


1. What were trenches feelings when he found the letter with money in it? What did he do after
opening the letter?

Answer: Lencho was not at all surprised to see the money in the letter as he was fully confident
about it. He knew that God would send money to him. On opening the letter, he found seventy
pesos instead of a hundred. So, he wrote another letter to God to send and rest of the amount
but now not through the mail as the post office employees were crooks.

2. Whom did Lencho write a letter? What did he ask for?


Answer: When Lencho’s annual crop was completely destroyed due to heavy rains and
hailstorms and there was no one to help hint during the crisis. At that time Lencho wrote a
letter to God.
He wrote the letter requesting him to send 100 pesos so that he could sow his fields again till
the next crop.

3. Why does the postmaster send money to Lencho? Why does he sign the letter ‘God’?
Answer: The postmaster was a kind, generous, helpful and God-fearing man. When he
received the letter written to God asking for 100 pesos, he felt sympathetic towards Lencho.
Hence, he decided to help Lencho. He gave a part of his salary and asked other employees to
help. He signed the letter ‘God’ in order to preserve the man’s faith in God.

Long Answer Type Questions


1. Lencho received the money and a letter from God but still, he was not happy. Was it not in
his attitude to be happy with what he achieved? Did he not know the pleasure of being grateful
to others? Reflecting on the values required to lead a happy life, explain in about 100-120
words.
Answer: Lencho had full faith in God and hence, had written a letter to God asking for 100
pesos. But when he received the money and a letter from God, he was not happy. It was not
his attitude(behaviour) to be happy with what he received, nor was he ungrateful to God. He
was angry because he had received 70 pesos instead of 100. He did not doubt God for it. He
wrote another letter to God asking him to send the rest of the money but not through the post-
office as they were a bunch of crooks. This shows that Lencho had full faith in God and was
confident(assured) that his desire would be fulfilled. It was Lencho’s innocent
optimism(hopeful) that eventually(finally) led to his getting seventy pesos. Optimism and faith
can lead to a happy life.

2. The postmaster represents such people who still believe in helping others mention those
values of the postmaster which you would like to emulate in yourself. Write in 100-120 words.
Answer: The postmaster was a kind, generous (readiness to give something), helpful,
amiable(friendly) and God-fearing man. He received a letter from Lencho which was written
to God asking for 100 pesos so that he could sow his fields again. The postman felt
sympathetic(liking) towards Lencho. He decided to help Lencho. He gave up a part of his
salary and asked all other employees to help. Even friends were made to contribute to the cause
68
of charity (help in the form of money) and he managed to collect 70 pesos. He sent the money
to Lencho along with a reply. He did so in order to preserve the man’s faith in God. This shows
the postmaster was a nice man at heart.

3. “If you don’t help me, my family and I will go hungry this year.” Lencho had faith in God
but he didn’t manage to solve the problem by himself. Did he lack the courage to resolve his
matter himself? What values did he lack? Explain it in 100-120 words.
Answer: Lencho was a hardworking farmer. He was completely dependent upon the crops of
his fields. His annual crop was completely destroyed due to heavy rains and hailstorms. In the
entire village, there was no one to help him during the times of financial crisis. He had a firm
belief in God and hence wrote a letter requesting Him to send 100 pesos so that he could sow
his fields again till the next crop. This act did not prove that he lacked the courage to resolve
his problem but shows that he did not have any option left for himself. He had a firm belief in
God. He was an ox of a man but lacked money to stand again.

CHAPTER 2
NELSON MANDELA LONG WALK TO FREEDOM
READ THE EXTRACTS GIVEN BELOW AND ANSWER THE QUESTIONS THAT
FOLLOW.
A. But the decades of oppression and brutality had another, unintended, effect, and that was
that it produced the Oliver Tambos, the Walter Sisulus, the Chief Luthulis, the Yusuf Dadoos,
the Bram Fischers, the Robert Sobukwes of our time* — men of such extraordinary courage,
wisdom and generosity that their like may never be known again. Perhaps it requires such
depths of oppression to create such heights of character. My country is rich in the minerals and
gems that lie beneath its soil, but I have always known that its greatest wealth is its people,
finer and truer than the purest diamonds. It is from these comrades in the struggle that I learned
the meaning of courage. Time and again, I have seen men and women risk and give their lives
for an idea.
(i) What was the unintended effect of the long oppression?
(a) It made the people indifferent to injustice.
(b) It made the people reject oppression.
(c) It influenced a generation to fight against injustice.
(d) It made people accept their oppression.
(ii) Men of such extraordinary courage refers to the people who ………………………….
(a) liberated and abolished the Apartheid system.
(b) used their resources to spread awareness.
(c) fought for their country’s freedom.
(d) demonstrated utmost strength to oppose the system.
(iii) Fill in the blank with ONE WORD only.
It is from these ________in the struggle that I learned the meaning of courage.
(iv) When Nelson Mandela says, “I have seen men and women risk and give their lives for an
idea”, he means that they are ……………………………
(a) stubborn
(b) committed
(c) intelligent (d) proud
69
(v) Pick a suitable word from the extract to complete the following: depths: heights:
compassion: ………………………

B. It was only when I began to learn that my boyhood freedom was an illusion, when I
discovered as a young man that my freedom had already been taken from me, that I began to
hunger for it. At first as a student, I wanted freedom only for myself, the transitory freedoms
of being able to stay out at night, read what I pleased and go where I chose. Later, as a young
man in Johannesburg, I
yearned for the basic and honourable freedoms...”
(i) The title that best suits this extract is:
(a)Freedom for Everything
(b)Knowledge about Freedom
(c)Significance of Freedom
(d)Realisation of Freedom

(ii) Fill in the blank with ONE WORD only.


According to the speaker the freedoms are and keep changing with time.
(iii) Choose the option that best fits the usage of the word ‘illusion’ as used in the extract.
(a)He was never able to get past the illusion.
(b)The illusion I experienced was quite intriguing.
(c)A large mirror in the room creates an illusion.
(d)I was living under the illusion that this is possible.
Iv) The speaker says, ‘at first as a student I wanted freedom only for myself.’ Why do you
think he only thought about himself?

v) A part of the extract has been paraphrased. Choose the option that includes the most
appropriate solution to the blanks in the given paraphrase of the extract. The speaker’s belief
about freedom, since childhood proved false. It was not until the speaker grew up to be a young
man when it (i) on him that he was (ii) of freedom. Then he began (iii) it.
(a)(i) desired (ii) dawned (iii) depriving
(b)(i) dawned (ii) deprived (iii) desiring
(c)(i) dawned (ii) arrived (iii) desiring
(d)(i) arrived (ii) deprived (iii) dawned

ANSWERS: A
(i) (c) It influenced a generation to fight against injustice.
(ii) (d) demonstrated utmost strength to oppose the system.
(iii) countrymen / people
(iv) committed
(v) Oppression

ANSWERS: B
(i) (d) Realisation of Freedom
(ii) Momentary / transitory
(iii) (d) I was living under the illusion that this is possible.
(iv) He was too young to realise that freedom was denied to others as well.
(v) (b) (i) dawned (ii) deprived (iii) desiring

2.SHORT ANSWER TYPE 3 QUESTIONS

70
ANSWER THE FOLLOWING IN ABOUT 40-50 WORDS EACH:
Q.1. What, according to Mandela, is ‘true freedom’?
ANS: When Mandela was a boy, freedom for him meant to run freely in the fields and to swim
in
the streams. As a young man, he wanted basic and honourable freedoms, e.g., to earn his living,
too many and to have a family. According to Mandela, true freedom means freedom not to be
obstructed in leading a lawful life.

Q.2. Why does Nelson Mandela call himself simply the sum of those African patriots who
had?
gone before him?
ANS: Nelson Mandela gratefully acknowledges the sacrifices of thousands of his people who
fought against the apartheid regime. Their suffering and courage can never be cemented or
repaid.
Mandela humbly believes that he was simply the sum of all those African patriots who had
gone
before him.

Q.3. After achieving political emancipation what does Mandela want to do in South Africa?
What “twin obligations” does Mandela mention?
Answer:
According to Mandela every person has twin obligations. The first obligation is towards his
family, parents, wife and children. The second obligation is to work for his people, community
and the nation
South Africa and the blacks have achieved their political emancipation. Mandela pledges to
liberate his people from the bondage of poverty, want, suffering, gender and other
discriminations.
South Africa will never ever experience the oppression of one by another.
He wishes the reign of freedom will never die in South Africa.

LONG ANSWER TYPE QUESTION

3.ANSWER THE FOLLOWING IN ABOUT 100-120 WORDS:


VALUE BASED QUESTION
‘The oppressed and the oppressor alike are robbed of their humanity.’ Discuss in the context
of the essay ‘Long Walk to Freedom’.
ANSWER:
“To deny people their human rights are to challenge their very humanity.”
– Nelson Mandela
Nelson Mandela in his essay, writes that according to him, like the oppressed, the oppressor
too is
not free. He says that oppressed don’t have basic rights; there is no dignity of life, only
subjugation,
cruelty and slavery. However, even the oppressor is not free. He says that the oppressor is also
a
slave of his hatred.
“The greatness of humanity is not in being human, but in being humane.”
– Mahatma Gandhi
71
When a person does something, he himself does not want to do but is pressurised due to his
beliefs,
he is a slave to those beliefs. He is not free as well. A man who takes away another man’s
freedom is a prisoner of hatred; he is locked behind the bars of prejudice and narrow
mindedness. He says that one is not truly free if one takes away someone else’s freedom, just
as surely as one is not free
when one’s freedom is taken away.

COMPETENCY BASED QUESTION


2. Explain with reference to the lesson “Nelson Mandela”, As long walk to freedom the
following statement, “Hunger for freedom” is the greatest virtue which one enjoys. How did
Mandela fight for freedom?
ANSWER:
“For to be free is not merely to cast off one’s chains, but to live in a way that respects and
enhances the freedom of others.”
- Nelson Mandela
Freedom is the greatest virtue which one enjoys while being on this earth. No one wants to
live in captivity viz. be it animal or bird. Even if a bird is kept in golden cage, it wants to fly.
“The hunger for freedom can make you taste the best recipe of life.”
Since boyhood Mandela discovered that his freedom had been taken from himself. He realised
in his youth that it was not just his freedom that was being curtailed but the freedom of all
blacks. The hunger for his own freedom became the hunger for the freedom of his people. This
desire of a non – racial society transformed him into a virtuous and self – sacrificing man. He
left the comforts of settled family life to fight for a greater cause.
“The only real prison is fear, and the only real freedom is freedom from fear.”
He joined the African National Congress and this changed him from a fearful young man to a
fearless rebel who fought against racial prejudice. The hunger for freedom made him a
homeless, monk and criminal. He underwent many tortures but still he worked very hard. He
remained behind the bars and won freedom for his country. He became the first black President
of South Africa marking the end of centuries of apartheid, thereby giving the blacks the
freedom to live like respectable citizens in their own land. This quenched his thirst and hunger
for freedom.
3.Discuss the scene of the inauguration ceremony? Who took oath in the ceremony? Why is
the inauguration calling a historic occasion for South Africa? What did the playing of two
national anthems symbolise?

Answer:
It was the bright and clear day of 10th May, 1994. The inauguration ceremony took place in
the Union Buildings amphitheatre in Pretoria. The most famous world leaders and
representatives gathered there. The generals and police officers were also there. They had
medals and ribbons on their chests. South African jets, helicopters and troop carriers roared in
perfect formation over the Union Buildings. First of all Mr. de Klerk, then Thabo Mbeki and
then Nelson Mandela took the oath.
The inauguration can be called a historic occasion for South Africa as on this day the first
democratic government was installed. It was an end of more than three centuries of the white
rule.
Nelson Mandela became the first Black President of South Africa.
On the historic day of the inauguration ceremony of the Republic of South Africa, two

72
national anthems were played. The whites’ song `Nkosi Sikelel-i-Africa’ and the black sang
‘Die Stem’. Neither group knew the lyrics of the anthem they once despised. They would soon
know the words by heart.

CHAPTER- 3 TWO STORIES ABOUT FLYING-- I. HIS FIRST FLIGHT

Q1 Multiple Choice Questions based on an extract


(A) The day before, all day long, he had watched his parents flying about with his brothers
and sister, perfecting them in the art of flight, teaching them how to skim the waves and how
to dive for fish. He had, in fact, seen his older brother catch his first herring and devour it,
standing on a rock, while his parents circled around raising a proud cackle. And all the morning
the whole family had walked about on the big plateau midway down the opposite cliff taunting
him with his cowardice.
i Based on the given sentence, pick the option that corresponds to what human parents
would say.
‘while his parents circled around raising a proud cackle.’
a) Well done!
b) Oh no!
c) Ready?!
d) Really!?
ii Which option lists the image nearest to ‘skim the waves’?

a) image (i)
b) image (ii)
c) image (iii)
d) image (iv)
iii Imagine that the young gull attended a workshop on inspiration and confidence building
and received a couple of pieces of advice.

Choose the option that reflects these pieces of advice, most relevant to his situation.

a) (i) and (ii)


b) (ii) and (iii)
c) (iii) and (iv)
d) (i) and (iv)

73
(iv) Which of the following feelings did the young gull, NOT feel according to the given
context?

“…all day long, he had watched his parents flying about with his brothers and sister…”

(i) sad (ii) incompetent (iii) excluded (iv) ungrateful


(v) inspired (vi) jealous (vii) anxious

a) i, iii, vi
b) ii, v, vii
c) ii, iii, vii
d) iv, v, vi
(v) Select the most appropriate option for the following: devour: guzzle: nibble:
a) chew
b) savior
c) peck
d) gulp
(B) He just felt a bit dizzy. Then he flapped his wings once and he soared upwards. “Ga,
ga, ga, Ga, ga, ga, Gaw-col-ah,” his mother swooped past him, her wings making a loud noise.
He answered her with another scream. Then his father flew over him screaming. He saw his
two brothers and his sister flying around him curveting and banking and soaring and diving.
Then he completely forgot that he had not always been able to fly, and commended himself to
dive and soar and curve, shrieking shrilly.
i 1. Pick the most appropriate reason why the young gull felt dizzy.

a) He hadn’t eaten anything for a day.


b) He was dizzy with excitement.
c) He was wary of heights.
d) He was flying for the first time.
ii How would you describe the screams of the gulls in the given extract?

a) elation
b) bewilderment
c) shock
d) protection

iii The line “he completely forgot that he had not always been able to fly” implies the
a) great confidence the young gull had in his skills.
b) naturalness of the act of flying for the young gull.
c) satisfaction and joy of flying together as a family.
d) desire of the young gull to leave his fears behind.
iv the extract refers to the many movements of the young gull’s brothers and sister. Choose
the option that correctly sequences these movements.
a) The young gull’s brothers and sister flew by tilting their wings, rose high, made darting
movements and plunged headfirst.
b) The young gull’s brothers and sister flew by plunging headfirst, making darting
movements, titled their wings and rose high.
c) The young gull’s brothers and sister flew with darting movements, titled their wings,
rose high and plunged headfirst.
74
d) The young gull’s brothers and sister flew by rising high, plunging headfirst, making
darting movements and tilting their wings.
V Which of the following mirrors the use of the literary device in “shrieking shrilly”?
a) sparkling saga
b) singing soft
c) slippery sloppily
d) sneeze silently
Q10 Answer in 20-30 words
i Describe the young seagull’s emotions when he flew over the sea.
ii The Oxford Dictionary defines ‘hangry’ as ‘bad-tempered or irritable as a result of
hunger’.
To what extent can the young seagull’s motivation to fly be attributed to being hungry?
Support your stance with evidence from the text.
iii “The young seagull was alone on his ledge.” How far do you think this condition was
by his choice?
Q11 Answer in 40-50 words
i If you were the family member of the young seagull, would you also decide not go near
him? Elaborate with reason.
ii The young seagull “failed to muster up courage to take that plunge”. In what way can
the reference to the ‘plunge’ here be both literal and metaphorical? Explain.
iii The ‘fight or flight response’, that is, to stay and face a situation or run from it- is an
automatic reaction to an event perceived as stressful or harmful. How would you evaluate the
young gull’s response on finding himself off the ledge?
Q12 Answer in 100-120 words (beyond text and across texts)
i The young seagull’s parents wanted him to fly, like his brothers and sister. How could
the young seagull’s newly flying siblings (Brother 1, Brother 2 and Sister) have motivated
him?
Write a brief conversation among them.
ii A noted author, Richard Bach wrote – “For most gulls it was not flying that mattered,
but eating. For this gull, though, it was not eating that mattered, but flight.”

Evaluate this statement with respect to the young seagull.


ii Look at the image of Maslow’s hierarchy of needs.
Abraham Maslow devised a theory of motivation that shows the basic needs at the bottom, and
more advanced needs as you move up. It includes the theory that fundamental needs must be
met before an individual can be motivated to achieve higher order needs.

75
Where in this hierarchy of needs, would you locate the young seagull’s first flight? Justify
your response.
iii Look at the given image of different parenting styles and what they signify.

Which of the given parenting styles would you attribute to the young seagull’s parents?
Do you think there was a difference between the mother’s and father’s attitude? Support your
answer with reference to the text.

Answer key for MCQs


A. i. - a) ii - c) iii - c) iv - d) v - c
B. i. - d) ii - a) iii - b) iv - c) v

L- 3 TWO STORIES ABOUT FLYING-- II. BLACK AEROPLANE TEXT--FIRST


FLIGHT
Q1 Multiple Choice Questions based on an extract
(A) Everything was going well — it was an easy flight. Paris was about 150 kilometers
behind me when I saw the clouds. Storm clouds. They were huge. They looked like black
mountains standing in front of me across the sky. I knew I could not fly up and over them, and
I did not have enough fuel to fly around them to the north or south. “I ought to go back to
Paris,” I thought, but I wanted to get home. I wanted that breakfast. ‘I’ll take the risk,’ I
thought, and flew that old Dakota straight into the storm.
76
i Based on the given extract, choose the option that lists the meme which would be the
most appropriate response to “…it was an easy flight”?

a) Option (i)
b) Option (ii)
c) Option (iii)
d) Option (iv)

ii How would you describe the “risk” the narrator took?

a) calculated
b) impetuous
c) unavoidable
d) navigable
iii In what way might the reference to the Dakota as “old” be relevant?

a) Its antique value made it expensive and precious to the narrator.


b) It is employed by the narrator as a term of endearment.
c) It did not have enough fuel to fly around the storm clouds.
d) Its ability to negotiate the storm clouds might have been suspect.
iv. Read the statements given below, and then select the option that best
describes the given statements.
Statement I – The narrator’s desire to reach home and see his family made him complacent.
Statement II – The narrator was unaware of the threat that the adversarial storm clouds
presented.
Statement III – The narrator’s decision making was quick but irresponsible as well as
dangerous.
a) Statement I is False, Statement II is True, Statement III cannot be inferred
b) Statement I and III are True, Statement II cannot be inferred.
c) Statement I cannot be inferred, Statement II is False, Statement III is True.
d) Statement I and II are False, Statement III is True.
v. Select the correct option to fill in the blanks below: risk: risky:: :
a) danger: dangerously
b) hazard: hazardous
c) peril: imperiled
d) caution: precaution
(B) I was safe! I turned to look for my friend in the black aeroplane, but the sky was empty.
There was nothing there. The black aeroplane was gone. I could not see it anywhere. I landed
and was not sorry to walk away from the old Dakota near the control tower. I went and asked
a woman in the control center where I was and who the other pilot was. I wanted to say ‘Thank
you’. She looked at me very strangely, and then laughed. “Another aeroplane? Up there in this
77
storm? No other aeroplanes were flying tonight. Yours was the only one I could see on the
radar.” So, who helped me…
i Select the option that correctly tracks the progression of emotions experienced by the
narrator in the given extract.
a) excited – surprised – relieved – grateful – perplexed
b) relieved – confused – curious – dejected – panic-stricken
c) optimistic – lonely – calm – elated – appreciative
d) triumphant – reassured– inquisitive – thankful – uncertain
ii Why do you think the woman in the control center laughed?
a) She found the narrator funny.
b) She thought his question preposterous.
c) She thought he was teasing her.
d) She was relieved the narrator was safe.

iii Filled with questions, the narrator decides to place an advertisement in the local
newspaper to look for his “friend”.
Read the advertisement given below and select the option that includes the most appropriate
solutions for the blanks:
Looking for a pilot of a black aeroplane who (i) an old Dakota out of storm clouds late last
night, but (ii) before the Dakota pilot could express his gratitude after landing. Though
control center and radar did not
(iii) its presence, the Dakota pilot would really appreciate if his friend reached out. Please
contact the Dakota pilot at 5200100110.
In deep gratitude and eager (iv) , XXX
a) (i) guided;(ii) disappeared;(iii) register;(iv) anticipation
b) (i) took;(ii) landed;(iii) acknowledge;(iv) appreciation
c) (i) brought;(ii) went away;(iii) confirm;(iv) expectation
d) (i) helped;(ii) vanish;(iii) make note;(iv) excitement
iv The narrator exclaimed that he was “safe”. Which of the following represented the most
immediate threat to the narrator’s safety?
a) The black mountain-like storm cloud
b) The depletion of fuel in the last fuel tank
c) Being lost due to non-functioning equipment
d) The old rattling Dakota aeroplane
v Choose the option that correctly matches the idioms in Column A to the story’s events
in column B:
a) 1-(iv); 2-(iii); 3-(ii); 4-(i)
b) 1-(iii); 2-(i); 3-(iv); 4-(ii)
c) 1-(i); 2-(ii); 3-(iii); 4-(iv)
d) 1-(ii); 2-(iv); 3-(i); 4-(iii)
Column A – Idioms Column B – Story events

1. Every cloud has a (i) The narrator really wanted to have a hearty English
silver lining. breakfast, even though he really ought to have turned back.

78
2. To be on cloud (ii) The man in the other plane waved at the narrator and
nine. asked him to follow, closely drawing him out.

3. To have your head (iii) The compass and other instruments stopped
in the clouds. working. The radio was dead too.

4. gathering clouds. (iv) And there it was – the well-lit runway. An airport. The
narrator could safely land.

Q10 Answer in 20-30 words


i How would you describe the mood of the story, ‘Black Aeroplane’ as it begins?
ii Look at the given image. Briefly evaluate the narrator’s decision to not go back to Paris
with reference to the image.

iii Do you think that after his experience, the narrator would be able to truly enjoy the
English breakfast he had yearned for? Why/ Why not?
iv What do you think prompted the narrator to act like “an obedient child”?
Q11 Answer in 40-50 words
i As a reader, do you feel betrayed or let down in the way the story ends? Validate your
opinion.
ii Based on your reading, would you call the narrator a family man? Justify your stance.
iii James Lane Allen says, “Adversity reveals character”. What do you learn about the
narrator from the experience? Mention any two traits supported by the textual evidence.
Q12 Answer in 100-120 words (beyond text and across texts)
i Imagine that the narrator shared his story with friends and family once he got home.
One of them, a spiritual leader, decided to incorporate the story as part of his weekly sermon
to the congregation. Another friend, a psychologist, worked the story into his next lecture on
survival instinct and crisis management.
Do you think the two interpretations of the story would be different? If so, how?
What insights might the narrator get about his mysterious experience, if he were to attend both
the sessions?
ii In a parallel world, the pilot of the black aeroplane narrates his adventure in a story
titled ‘The Old Dakota’. You come across an excerpt from the story, which responds to the
ending questions of ‘Black Aeroplane’ and describes
the black aeroplane pilot’s decision to fly away after watching the Dakota land.

Compose the said excerpt of the story employing relevant details from ‘Black Aeroplane’.
79
iii A few days after the experience, the narrator sat with his wife and recalled a long
conversation he’d had with his young son the day before. The narrator’s son had spoken to
him about his close friend, Freddie, and the many ways Freddie helped him. When the narrator
suggested inviting Freddie home for lunch, his wife casually said, “Freddie is like your friend
in the black aeroplane!”

That night the narrator decided to record his thoughts in his diary to clear his head. As the
narrator, write the diary entry.

Answer key for MCQs


A)
i. - c) ii - b) iii - d) iv - c) v - b)

B)
i. - a) ii - b) iii - a)
iv - b)
v - d)

CHAPTER 4- FROM THE DIARY OF ANNE FRANK


(i) Extract based questions From the Diary of Anne Frank
Question 1. Read the following extracts carefully and answer the questions that follow:
The class roared. I had to laugh too, though I’d nearly exhausted my ingenuity on the topic of
chatterboxes. It was time to come up with something else, something original. My friend,
Sanne, who’s good at poetry, offered to help me write the essay from the beginning to end in
verse and I jumped for joy. Mr Keesing was trying to play a joke on me with these ridiculous
subjects, but I’d make sure the joke was on him.
a) Who is ‘he’? What did ‘he’ had enough of?
b) How was this essay different from the one written earlier?
c) What do you think, why was Anne punished?
d) Why did the whole class roar with laughter?

From the Diary of Anne Frank (short question)


Question1. What does Anne write in her first essay?
Question2. ‘Paper has more patience than people.’ Do you agree/disagree? Give reasons
Question 3. (i) Why did Mr Keesing punish her?
(ii) What was the punishment?
(iii) How did Anne finally stop Mr Keesing from punishing her?

CHAPTER 5 - GLIMPSES OF INDIA

PART – I: A Baker from Goa by Lucio Rodrigues


Extract Based Questions
Read the following extracts carefully and answer the questions that follow
Question 1 (Solved)

80
“During our childhood in Goa, the baker used to be our friend, companion and guide. He used
to come at least twice a day. Once, when he set out in the morning on his selling round, and
then again, when he returned after emptying his huge basket. The jingling thud of his bamboo
woke us up from sleep and we ran to meet and greet him. Why was it so? Was it for the love
of the loaf? Not at all. The loaves were bought by some Paskine or Bastine, the maid-servant
of the house! What we longed for were those bread-bangles which we chose carefully.
Sometimes it was sweet bread of special make.”
(Glimpses of India–A Baker from Goa)
(i) What according to the narrator was the reaction of the children on hearing the baker’s
bamboo thud?
1. They avoided the loud noise and turned around and sleep.
2. They would wake up from their sleep.
3. They would jump out of bed quickly.
4. They would run to meet and greet him.
5. They would go to buy loaves.
Choose the correct option from the following:
(a) (1) and (5)
(b) (2), (3) and (4)
(c) (2) and (3)
(d) (3) (4) and (5)
Ans. (b) (2), (3) and (4)
(ii) Select the option which displays an example of ‘jingling’
(a) The hawker pushed through the crowd in the market.
(b) The little boy ran across the road to fetch the ball.
(c) The ice-cream vendor began ringing a small bell attached to his cart on the beach.
(d) The two old women were strolling in the park
Ans. (c) The ice-cream vendor began ringing a small bell attached to his cart on the
beach.
(iii) From the options given below, identify the attitude of the children in the extract:
(a) Frightened
(b) Restless
(c) Excited
(d) Hesitant
Ans. (c) Excited
(iv) ‘Not at all’ in the above extract means…Choose one from the following to answer:
(a) Of course
(b) In every respect
(c) By no means
(d) Absolutely
Ans. (c) By no means
(v) Select the most appropriate option for (1) and (2).
(1) Paskine or Bastine were male servants of the house.
(2) The narrator ate only the sweet bread bangles.
(a) (1) is true and (2) is false.
(b) (2) is the opposite of (1).
(c) (1) furthers the meaning of (2).
(d) Both (1) and (2) cannot be inferred from the extract.
Ans. (d) Both (1) and (2) cannot be inferred from the extract.

81
Question-2(Unsolved)
“Marriage gifts are meaningless without the sweet bread known as the bol, just as a party or a
feast loses its charm without bread. Not enough can be said to show how important a baker
can be for a village. The lady of the house must prepare sandwiches on the occasion of her
daughter ‘s engagement. Cakes and bolinhas are a must for Christmas as well as other festivals.
Thus, the presence of the baker ‘s furnace in the village is absolutely essential.”
(a) What are compulsorily prepared during Christmas in Goa?
(b) Why is a baker necessary in a village?
(c) Find the word in the extract which means celebration meal.
(d) What is must to be prepared on a daughter ‘s engagement by a lady?
Short Answer Type Questions: (40-50 words)
1.Describe a baker in Goa with reference to the author's childhood memories.
2.Describe the attire and comment on the fortune of the paders or the bakers in the old days.
3.What is the importance of breads for the Goans?
4. How did the baker make his entry?
Long Answer Type Questions: [100-120 words]
1. Instead of enjoying their childhood, the children today are keen to enter adulthood. After
reading about all the joys that the author Lucio Rodrigues had in his childhood do you think
such a keenness on the part of children is desirable?
2. After reading the story ‘A Baker from Goa’ do you think our traditions, heritage, values and
practices are the roots that nourish us? Why/why not?
3.During our childhood in Goa, the baker used to be our friend, companion and guide. ‘What
does this statement imply in relation to the character of the baker?
PART II: GLIMPSES OF INDIA – COORG by Lokesh Abrol
Extract Based Questions
Read the following extracts carefully and answer the questions that follow.
Question 1 (Solved)
“The fiercely independent people of Coorg are possibly of Greek or Arabic descent. As one
story goes, a part of Alexander’s army moved South along the coast and settled here when
return became impractical. These people married amongst the locals and their culture is
apparent in the martial traditions, marriage and religious rites, which are distinct from the
Hindu mainstream… Coorgi homes have a tradition of hospitality, and they are more than
willing to recount numerous tales of valour related to their sons and fathers. The Coorg
Regiment is one of the most decorated in the Indian Army, and the first Chief of the Indian
Army, General Cariappa, was a Coorgi. Even now, Kodavus are the only people in India
permitted to carry firearms without a license.”
(Glimpses of India-Coorg)
(i) How is the narrator trying to describe the special quality of the people of Coorg?
1. ferocious
2. fiercely independent
3. Greek or Arabic descent
4. dependent
5. not independent
Choose the correct option from the following:
(a) (1) and (5)
(b) (3), (4) and (5)
(c) Only 2
(d) (2) and (3)
Ans. (d) (2) and (3)
82
(ii) Select the most appropriate option for (1) and (2).
(1) Alexander’s army married amongst the locals.
(2) Their culture is apparent in the martial traditions, marriage and religious rites, and are
indistinct from the Hindu mainstream.
(a) (1) is true and (2) is false.
(b) (2) is the opposite of (1).
(c) (2) furthers the meaning of (1).
(d) Both (1) and (2) cannot be inferred from the extract.
Ans. (a) (1) is true and (2) is false.
(iii) From the options given below, find a word from the extract that means the same as
relating to war.
(a) Hospitable
(b) Fiercely
(c) Martial
(d) Humility
Ans. (c) Martial
(iv) Select the option which displays an example of ‘valour’.
(a) Jane picked up a bag found on the street and handed it over to the police.
(b) Raj patiently helped the old woman cross the road.
(c) Sam jumped into the water and saved the little boy from drowning.
(d) The ragged woman shared her food with the street children.
Ans. (c) Sam jumped into the water and saved the little boy from drowning.
(v) Which group of people in India are allowed to carry firearms without licence?
Choose one from the following to answer:
(a) Gorkha
(b) Nagas
(c) Kodavus
(d) Toda
Ans. (c) Kodavus
Question 2(Unsolved)
“Coorgi homes have a tradition of hospitality, and they are more than willing to recount
numerous tales of valour related to their sons and fathers. The Coorg Regiment is one of the
most decorated in the Indian Army, and the first Chief of the Indian Army, General Cariappa,
was a Coorgi. Even now, Kodavus are the only people in India permitted to carry firearms
without a license.”
(a) What kind of stories are the Coorg people always ready to tell?
(b) What is the special favour granted only to them even now?
(c) Find a word in the extract which means the same as ‗courage and bravery, usually in war.
(d) Who is free to have a firearm without a license in India?
Short Answer Type Questions: (40-50 words)
1. What does the author say regarding the descent of the people of Coorg?
2. How has the Coorgi tradition of courage and bravery recognised in modern India?
3. Describe any two tourist places of Coorg.
4. Describe Coorg’s weather. When is it most pleasant for the tourists to visit Coorg?
Long Answer Type Questions: [100-120 words]
1 “... a piece of heaven that must have drifted from the kingdom of God.” Elucidate with
reference to context.

83
2. The Coorgis are the descendants of the Greeks or the Arabs and are still are able to maintain
their traditional practices. Do you agree that following these practices today is important? Why
or why not?
PART III: GLIMPSES OF INDIA – Tea from Assam by Arup Kumar Datta
Extract Based Questions
Read the following extracts carefully and answer the questions that follow.
Question 1 (Solved)
“Tell me another!” scoffed Pranjol. “We have an Indian legend too. Bodhidharma, an ancient
Buddhist ascetic, cut off his eyelids because he felt sleepy during meditations. Ten tea plants
grew out of the eyelids. The leaves of these plants when put in hot water and drunk banished
sleep. “Tea was first drunk in China,” Rajvir added, “as far back as 2700 B.C.! In fact, words
such as tea, ‘chai’ and ‘chini’ are from Chinese. Tea came to Europe only in the sixteenth
century and was drunk more as medicine than as beverage.”
(i) “Tell me another!” scoffed Pranjol. By this, it is meant that
Pranjol spoke ______________.
Choose the right option:
(a) excitingly
(b) surprisingly
(c) mockingly
(d) angrily
Ans. (c) mockingly
(ii) In the lesson Tea from Assam, why does Pranjol scoff at Rajvir?
1. Pranjol was ignorant about the legends.
2. Rajvir had read about Assam.
3. Rajvir knew all about the history of Tea.
4. Pranjol was sleepy.
5. Pranjol was excited.
Choose the correct option from the following.
(a) (1) and (5)
(b) (3) and (4)
(c) (2) and (4)
(d) (1), (2) and (3)
Ans. (d) (1), (2) and (3)
(iii) Select the most appropriate option for (1) and (2).
(1) A Buddhist ascetic Bodhidharma felt sleepy during meditations.
(2) A Buddhist ascetic Bodhidharma cut off his eyelids.
(a) (1) is true and (2) is false.
(b) (2) is the opposite of (1).
(c) (2) is the result of (1).
(d) Both (1) and (2) cannot be inferred from the extract.
Ans. (c) (2) is the result of (1).
(iv) Select the option which displays an example of ‘meditation’ in the extract—
(a) The teacher asked Ram to run an extra round as a punishment.
(b) The group of women were practicing Yoga in the park.
(c) The woman was tired and lay down to sleep.
(d) The boy ran after the ball to save a run.
Ans. (b) The group of women were practicing Yoga in the park.
(v) Synonym of the word ‘banished’ in the above extract is—
Choose one from the following to answer:
84
(a) take away (b) replace
(c) abandon (d) vanished
Ans. (d) Vanished

Question 2 (Unsolved)
“Pranjol‘s father slowed down to allow a tractor, pulling a trailer-load of tea leaves, to pass.
―This is the second-flush or sprouting period, isn‘t it, Mr. Barua? Rajvir asked. ―It lasts
from May to July and yields the best tea‖. ―You seem to have done your homework before
coming‖, Pranjol‘s father said in surprise. „ ―Yes, Mr. Barua‖, Rajvir admitted. ―But I hope
to learn much more while I‘m here.”
(a) Why did Mr. Barua feel surprised?
(b) How did Rajvir want to spend his stay there?
(c) Which word in the extract means ‘agreed’?
(d) What is the sprouting period of tea?
Short Answer Type Questions: (40-50 words)
1. What legends are associated with the origin of tea?
2. “You seem to have done your homework before coming,”. Who has done the homework
and why is it so impressive?
3. Where and when was tea drunk first? When did tea come to Europe?
4.What made it a " magnificent view"?
Long Answer Type Questions: [100-120 words]
1.According to the text, Assam is said to be a tea country ‘. Do you believe that Assam has
some of the best plantations in the world that makes it a unique country?
2.Pranjol invited Rajvir to his home in Assam during summer vacation. Express your views
on the deep bond that they both shared. Do you also believe in the compatibility of two
individuals? Why/Why not?

CHAPTER 6- MIJBIL THE OTTER

Extract 1:
For the first twenty-four hours Mijbil was neither hostile nor friendly; he was simply aloof and
indifferent, choosing to sleep on the floor as far from my bed as possible. The second night
Mijbil came on to my bed in the small hours and remained asleep in the crook of my knees
until the servant brought tea in the morning, and during the day he began to lose his apathy
and take a keen, much too keen, interest in his surroundings. I made a body-belt for him and
took him on a lead to the bathroom, where for half an hour he went wild with joy in the water,
plunging and rolling in it, shooting up and down the length of the bathtub underwater, and
making enough slosh and splash for a hippo.
Q1) “Mijbil was neither hostile nor friendly; he was simply aloof and indifferent…”
From the actions of the given set of friends, choose the person whose behaviour is likely to
resemble Mijbil’s.

a) a child rudely tells her best friend to stop calling.


b) a child was amicable and friendly since the day he walked into his new classroom.
c) a child became nasty as soon as he started earning money.
d) a child was distant for a while after losing her grandmother.

85
Correct Answer: Option (d)
Q2) The words “slosh and splash” are examples of the onomatopoeic words that go together
phonetically-resembling the sound that they describe.
Choose the option that DOES NOT fit the given description.
a) hip hop
b) bow wow
c) tick tock
d) pitter patter
Correct Answer: Option (a)

Q3) Select the option that correctly puts the following events in correct order.
1. Maxwell planned to take Mijbil to the bathroom.
2. Mijbil slept at a significant distance from Maxwell’s bed.
3. Mijbil immensely enjoyed playing in water.
4. Mijbil slept in much closer vicinity of the author as opposed to the previous night.

a) 1, 3, 4, 2
b) 3, 1, 2, 4
c) 2, 4, 1, 3
d) 4, 3, 1, 2

Correct Answer: Option (c)

Q4) Choose the option that correctly describes a body-belt in the given context as explained
by the four people given below.

a) A body-belt is a device especially made for swimmers and scuba divers, which helps them
breathe under water.
b) Body-belts basically resemble the waist belts and keep our body upright, in a standing
position.
c) Body-belts are used for restraining while doing certain activities, in order to prevent hurt.
d) A body-belt is an accessory used by body builders for weight training as a measure against
muscle and bone injuries.

Correct Answer: Option (c)

Q5) Select the option that best describes the final action of the otter, based on the given
passage.

Correct answer : 2
● EXTRACT 2
When I returned, there was an appalling spectacle. There was complete silence from the box,
but from its airholes and chinks around the lid, blood had trickled and dried. I whipped off the
86
lock and tore open the lid, and Mij, exhausted and blood-spattered, whimpered and caught at
my leg. He had torn the lining of the box to shreds; when I removed the last of it so that there
were no cutting edges left, it was just ten minutes until the time of the flight, and the airport
was five miles distant. I put the miserable Mij back into the box, holding down the lid with my
hand. I sat in the back of the car with the box beside me as the driver tore through the streets
of Basra like a ricocheting bullet.
Q.1) Choose the speaker that uses the phrase ‘appalling spectacle’ correctly.
Speaker 1 – We got back home to see that our pet dog had eaten kheer my mother had kept
out of the fridge. Indeed, an appalling spectacle.
Speaking 2 – We got back home and witnessed that our cousin had dressed our pet dog in his
t-shirt. Have to admit that it was an appalling spectacle.
Speaker 3 – We got back home to see that our pet dog sitting with a mangled rat. It was an
appalling spectacle.
Speaking 4 – We got back home from the airport and observed our pet dog sound asleep on
the rug. What an appalling spectacle!
a) Speaker 1
b) Speaker 2
c) Speaker 3
d) Speaker 4
Correct Answer: Option (c)
Q.2) Choose the option that is closest in resemblance to the box’s condition in which Mij was
kept.

a) Option (1) b) Option (2) c) Option


(3) d) Option (4)
Correct Answer: Option (d)
Q.3) Select the option that DOES NOT state the possible reason for Maxwell’s cutting the box
edges off.
a) Mij could have injured himself further.
b) It could have made the place untidy.
c) Mij could have accidently swallowed the torn pieces
d) It could have obstructed the shutting of the lid.
Correct Answer: Option (b)
Q4) “whipped off”, is an example of a phrasal verb i.e. a combination of a verb and a
preposition. Choose the option that DOES NOT fit the given definition.

87
Correct Answer: Option (b)

Q.5 Choose the option that correctly states Maxwell’s likely feelings, in the situation below.
“… I put the miserable Mij back into the box, holding down the lid with my hand.”
(1) anxious (2) disappointed (3) terrified
(4) guilty (5) insulted
a) 1 and 5 b) 2 and 3 c) 1 and 4 d) 3 and 5
Correct Answer: Option (c)
EXTRACT 3
It is not, I suppose, in any way strange that the average Londoner should not recognize an
otter, but the variety of guesses as to what kind of animal this might me came as a surprise to
me. Otters belong to a comparatively small group of animals called Mousselines, shared by
the badged, mongoose, weasel, stoat, mink and others. I faced a continuous barrage of
conjectural questions that sprayed all the Mousselines but the otter; more random guesses hit
on a ‘baby seal’ and ‘a squirrel’. ‘Is that a walrus, mister?’ reduced me to giggles and outside
a dog show I heard ‘a hippo’. A beaver, a bear cub, a leopard – one apparently that had changed
its spots – and ‘a brontosaur’. Mij was anything but an otter.
Q,1 It is not in any way strange that the average Londoner should not recognize an otter.”
This implies that
[A]. People in London knew about otters.
[B]. The author was surprised that people did not know about otters.
[C]. The author expected people not to recognize an otter.
[D]. People in London also kept otters as pet
CORRECT ANSWER : B
Q.2“I faced a continuous barrage of conjectural questions,” What did the author face?
[A]. Unusual questions
[B]. Ordinary questions
[C]. careful questions
[D]. Questions filled with surprise.
Answer: Option A
Q.3 The way people reacted on seeing Mijbil tells us that otters are:
[A]. Commonly known pets
[B]. Rare pets
[C]. Comical pets
[D]. Carefully chosen pets
ANSWER : B

88
Q.4 The information that we gather about otters from this passage is that otters share
their characteristics with
[A]. Baby seal and squirrel
[B]. Badger and weasel
[C]. Walrus and bear club
[D]. Mongoose and brontosaurs
ANSWER : A

Q.5 According to the passage, which of the following animals look similar to an otter?

ANSWER: A
QUESTION AND ANSWERS.
1. What are ‘compulsive habits’? What does Maxwell say are the compulsive habits of?
(i) school children
(ii) Mij ?
Answer:
Compulsive habits are those acts which result from an irresistible urge. School children on
their way to and from school must place their feet squarely on the centre of each paving
block; must touch every seventh upright of the iron railings, or pass to the outside of every
second lamp post.
Mij had also developed certain compulsive habits. There was a single-storied primary
school opposite to the author’s home. There was a two feet high wall also. On his way to
the home, Mij would tug the author to that wall, jump on to it, and gallop the full length of
its thirty yards.
2. What ‘experiment’ did Maxwell think Camusfearna would be suitable for?
Answer:
Maxwell was staying in a cottage in Camusfearna. There was a lot of water around the cottage.
He thought to have an otter as a pet instead of a dog. Though London is not suitable for
otters but the place – Camusferna , surrounded by water could be suitable. An otter likes
flowing water. So the author thought that Camusfearna would be suitable spot for that
experiment.
3. How did Maxwell get Mijbil, the otter?
Answer: Maxwell had changed his idea of keeping a dog as his pet. He decided to have
an otter. He had asked one of his friends to arrange for an otter. He went to his bedroom to
read his mail. Two Arabs were squatting on the floor. Beside them lay a sack. They handed
him a note which said, “Here is your otter …”. The sack was opened. In this way, Maxwell
got his new pet — an otter.

LONG QUESTIONS.
1. What were strange and funny wild guesses that Londoners made about Mijbil? Who
made the best remark about the otter?

89
Answer: It was rather difficult for an average Londoner to recognise such a strange
animal as an otter. Most of them had never seen such a unique animal. The thing that
surprised Maxwell that Londoners made such funny and wild guesses about Mijbil, the
otter. He faced a continuous barrage of questions from the anxious but ignorant people.
They made random guesses about the animal. Some of them guessed that he was ‘a
baby seal’ or ‘a squirrel’ or ‘a walrus’. Others also made fantastic guesses. One called
Mijbil ‘a beaver’. Another called it ‘a bear cub’. The third guessed that it was `a
leopard’. Mij was anything but an otter.
The question that was awarded the highest score came from a labourer. The surprised
labourer spat, glared and growled out, “Here, Mister—what is that supposed to be?”
2. The way Maxwell was worried about Mijbil when he had to take him to London shows
that he was very concerned about him. Do you think, owning a pet is a life-time
commitment?Elucidate.
Answer:
Maxwell was very concerned about his pet, Mijbil when he had to undertake the plane
journey. He wanted him to get used to the box in which he had to travel. When Mijbil
was injured in his attempt to come out of the box, the author was really concerned about
his well-being. He took his food on the flight.
The lesson definitely shows that when we decide to keep a pet, it is a commitment of
lifetime. This is so because we are responsible for the well-being of that life and in fact,
keeping a pet at home is like bringing up and nurturing a child.

3. What are ‘compulsive habits’? What does Maxwell say are the compulsive habits of
Answer: Compulsive habits are usually strange acts or behaviour which a person does
without a clear reason. For example, a cricket player may put on his right shoes first as
he believes it would bring him good luck or a kid jumping over a fence, instead of going
through the passageway. Usually, the compulsive behaviour of children is full of
childhood mischief and those of adults are of an incorrigible type.
As per this story, children place their feet squarely on the centre of each paving
block; touch every seventh upright of the iron railings, or pass to the outside of every
second lamp post just to have fun.
Mijbil while on its way to home would jump over the boundary wall railing and run at
full speed throughout its length.

LESSON-9 MADAM RIDES THE BUS


1- Reference to context.
Read the following extract and answer the questions that follow.
Her first journey—what careful, painstaking, elaborate plans she had to make for it! She
had thriftily saved whatever stray coins came her way, resisting every temptation to buy
peppermints, toys, balloons, and the like, and finally she had saved a total of sixty paise.
How difficult it had been, particularly that day at the village fair, but she had resolutely
stifled a strong desire to ride the merry-go-round, even though she had the money.
After she had enough money saved, her next problem was how to slip out of the house
without her mother’s knowledge.
(i) Why did Valli save every penny that came on her way?
a) She wanted to deposit in piggy bank
b) She wanted to buy a pretty doll in village fair.
c) She wanted to go town by bus
d) She wanted to buy a beautiful dress for her birthday.
90
(ii) What temptations did she have to resist?
(iii) What kind of desire did she have to curb at the village fare?
(iv) Pick out the word from given options that means the same as— spend money carefully’.
a) Stray
b) Resist
c) Stifled
d) Thriftily
(v) Why did Valli find difficult to save money for her bus ride?
2-The most fascinating thing of all was the bus that travelled between her village and the
nearest town. It passed through her street each hour, once going to the town and once
coming back. The sight of the bus, filled each time with a new set of passengers, was a
source of unending joy for Valli. Day after day she watched the bus, and gradually a tiny
wish crept into her head and grew there: she wanted to ride on that bus, even if just once.
This wish became stronger and stronger, until it was an overwhelming desire. Valli would
stare wistfully at the people who got on or off the bus when it stopped at the street comer.
Their faces would kindle in her longings, dreams, and hopes. If one of her friends
happened to ride the bus and tried to describe the sights of the town to her, Valli would
be too jealous to listen and would shout, in English: “Proud! Proud!”
(i) What did Valli watch daily?
(ii) What was her overwhelming desire?
a) to buy toys from town
b) to see children playing marbles
c) to have a bus ride
d) to talk to bus passengers
(iii) Was Valli’s wish to ride the bus stronger or weaker?
(iv) Pick out the word from the passage that means same as—longingly.
a) kindle
b) wistfully
c) overwhelming
d) fascinating
(v) What was the source of unending joy for Valli?
4-Answer any THREE of the following in about 40-50 words each.
a) Why did Valli smile and agreed with her mother after bus ride?
b) What information did Valli gather for her bus ride from neighbours and passengers
5- Answer any THREE of the following in about 100-120 words each.
a) Valli is merely eight-year old girl but she behaved like a mature person. Write character
sketch of Valli. Support your answer with example from the text.
b) Whenever we want to achieve something obstacles always come in our way. What did
Valli have to do to go and ride in a bus?
c) Valli was overcame by sadness in her return journey. What lessons did Valli learn about
life while travelling in the bus?
d) “The conductor was a jolly sort, fond of joking.” How did the conductor receive Valli
and create humour?
First Flight Chapter 10 The Sermon at Benares
Reference to Context
Extract 1
(A) Poor Kisa Gotami now went from house to house, and the people pitied her and said, “Here
is mustard-seed; take it!” But when she asked, “Did a son or daughter, a father or mother, die

91
in your family?” they answered her, “Alas! the living are few, but the dead are many. Do not
remind us of our deepest grief.” And there was no house but some beloved one had died in it.
i) Name the lesson from where the extract have been derived?
a) The Proposal
b) Glimpses of India
c) A Letter to God
d) The Sermon at Benares
Answer: (d)
ii) Which of the following options represent the correct understanding of the word “poor” in
the phrase “Poor Kisa Gotami”?
a) in need of money
b) weak
c) unfortunate
d) inferior
Answer: (c)
iii) “Do not remind us of our deepest grief.” The tone of the speaker(s) is____
a) disillusioned.
b) sceptical.
c) ironic.
d) sorrowful.
Answer: (d)
iv) Pick the option that explains — ‘…the living few, but the dead many.’
a) It shows the high death rate and low birth rate in the city of Benares.
b) It highlights the holy status of Benares where many Hindus go to die.
c) It throws light on the numerous loved ones the villagers had lost over time.
d) It reflects that many children who had died in the village for various reasons.
Answer: (c)
v) Why did Kisa Gotami go from house to house.
a) to get a handful of mustard seed
b) to bring oil2 and 3
c) to bring food
d) None of the above
Answer: (a)

Extract 2
At twelve, he was sent away for schooling in the Hindu sacred scriptures and years later he
returned home to marry a princess. They had a son and lived for ten years as befitted royalty.
At about the age of twenty-five, the prince heretofore shielded from the sufferings of the world,
while going out on hunting, chanced upon a sick man, then an aged man, then a funeral
procession, and finally a monk begging for alms. These sights so moved him that he at once
became a beggar and went out into the world to seek enlightenment concerning the sorrows he
had witnessed.
(i) Who does ‘he’ refer to here?
(a) Kisa Gotami
(b) Gautama Buddha
(c) Buddha’s Father
(d) Buddha’s Neighbour
Answer: (b)
(ii) What did the prince chance to see while out on hunting?
92
(a) A Sick man
(b) An Aged man
(c) A Funeral Procession
(d) All of the above
Answer: (d)
(iii) Find a word from the passage that means the same as ‘holy’.
(a) Monk
(b) Sacred
(c) Scriptures
(d) None of the above
Answer: (b)
(iv) When was Buddha sent for Schooling?
(a) Ten
(b) Twenty-Five
(c) Twelve
(d) Six
Answer: (c)
(v) At what age Gautama Buddha left the Palace
(a) Twelve
(b) Thirteen
(c) Twenty-Five
(d) None of the above
Answer: (c)
Extract 3
He wandered for seven years and finally sat down under a peepal tree, where he vowed to stay
until enlightenment came. Enlightened after seven days, he renamed the tree the Bodhi Tree
(Tree of Wisdom) and began to teach and to share his new understandings. At that point he
became known as the Buddha (The Awakened or the Enlightened). The Buddha preached his
first sermon at the city of Benares, most holy of the dipping places on the River Ganges; that
sermon has been preserved and is given here. It reflects the Buddha’s wisdom about one
inscrutable kind of suffering.
(i) The lines from the extract are a part of Buddha’s first______ at the city of Benaras.
a) Talk
b) Speech
c) Sermon
d) Lessoner
Answer: (c)
ii. Before attaining enlightenment, Gautama was known as .......
a) Buddha
b) Siddhartha Gautama
c) The Enlightened
d) Sakyamuni
Answer: (b)
iii) Find the antonym of ‘comprehensible’ from the extract.
a) Suffering
b) Awakened
c) Dipping
d) inscrutable
Answer: (d)
93
iv) Gautama ________ to stay under the tree until enlightenment came.
a) Promised
b) Vowed
c) Practiced
d) Came
Answer: (b)
v) Find the synonym of the word ‘roamed’ in the extract.
a) Drift
b) Hike
c) Wandered
d) Walk
Answer: (c)
Short Answer type
Question 1.
How did Gautama came to be known as the Buddha?
Answer:
Siddhartha Gautama wandered for seven years and finally sat down under a Peepal tree till he
got enlightenment. After seven days of enlightenment, he renamed the ‘Bodhi Tree’ and began
to teach and to share his new understanding and came to be known as the Buddha.
Question 2.
Why did Prince Siddhartha leave the palace and became a wanderer?
Answer:
While out hunting Prince Siddhartha chanced upon a sick man, an aged man and then a funeral
procession and he also saw a monk begging. He was moved by those sights and was filled with
sorrow and thus decided to leave the palace in search of
enlightenment.
Question 3.
What did Kisa Gotami do after the Buddha had asked her for a handful of mustard seed?
Answer:
Kisa Gotami went from house to house to get a handful of mustard seed. People gave her the
mustard seed. But when she asked if anyone had died in their family, they regretfully told her
that the livings were few, but the deads were many. Kisa Gotami found no house where
someone had not died.
Long Answer Type
Question 1.
What is the nature of the life of human beings according to the Buddha?
Answer:
The life of mortals in this world is troubled and brief. It is combined with pain. Nobody can
avoid dying. After reaching old age, there is death. Such is the nature of human beings. Just as
ripe fruits are in danger of falling; so mortals are in danger of death. As all earthen vessels
after a certain period of time break, so is the life of mortals. All have to die. Only he can get
peace of mind who does not lament, complain and grieve. He who has overcome sorrow will
be free from sorrow, and be blessed.
Question 2.
What lesson on death and suffering did the Buddha teach Kisa Gotami in the chapter ’The
Sermon at Benaras’?
Answer:

94
Kisa Gotami’s only son had died. In her grief she carried the dead child to all her neighbours,
asking them for medicine. Then a man suggested her to go to Sakyamuni, the Buddha. Kisa
Gotami requested Gautama Buddha to give her the medicine that would cure her son.
The Buddha replied that he wanted a handful of mustard-seed which must be taken from a
house where no one had lost a child, husband, parent or friend. Kisa Gotami went from house
to house. People pitied her but she could not find any house where near and dear one had not
died. She thought how selfish she had been in her grief. She realised that death is common to
all human beings.
Question 3.
“The World is afflicted with death and decay, therefore the wise do not grieve, knowing the.
terms of the world”. Explain with reference to the story “The Sermon at Benares”.
Answer:
According to Gautama Buddha, the world is affected by sufferings, disease or pain, death and
decay. Therefore, the wise persons do not mourn, knowing the terms of the world. Nobody
can get peace of mind from weeping or mourning. Rather his pain will be greater and his body
will suffer. He will make himself sick and pale, yet the dead cannot be saved from his
expression of sorrow. He who seeks peace should “draw out the arrow” of lamentation,
complaint and grief. In this manner, she who has become composed will obtain peace of mind.
He who has overcome all sorrow will become free from sorrow, and be blessed.

FIRST FLIGHT: CHAPTER 10- THE PROPOSAL


I. Reference to context questions.
Extract 1
We just get along somehow, my angel, thanks to your prayer, and so on. Sit down, please
do… Now, you know, you shouldn’t forget all about your neighbours, my darling. My dear
fellow, why are you so formal in your get-up! Evening dress, gloves and so on. Can you be
going anywhere, my treasure?
Questions:
1. Who is the speaker?
2. What does the speaker say to his neighbour Lomov?
3. What kind of dress is Lomov wearing?
4. Was Lomov going anywhere else?
Answers:
1. Chubukov, the father of Natalya, is the speaker of these lines.
2. The speaker says that Lomov should not forget all about his neighbours.
3. Lomov is very formal in his get up. He is wearing an evening dress, gloves, and so on.
4. No, Lomov had come only to see chubukov and his daughter Natalya.

Extract 2
Read the following extracts carefully and answer the questions that follow—
It's cold... I'm trembling all over, just as if I'd got an examination before me. The great thing
is, I must have my mind made up. If I give myself time to think, to hesitate, to talk a lot, to
look for an ideal, or for real love, then He will never get married.
Question 1: Who is 'I’ in the passage?
(i) Natalya
(ii) Lomov
(iii) Chubukov
(iv) Lomov's friend
95
Answer
(ii) Lomov
Question 2: The speaker trembling because:
(i) it was cold
(ii) as if he had an examination
(iii) both (i) and (ii)
(iv) he wasn't covered well
Answer
(iii) both (i) and (ii)
Question 4: He thought he would never got married if he:
(i) thought too much
(ii) hesitated
(iii) talked a lot
(iv) all of the above
Answer
(iv) all of the above
Question 4: What according to the speaker, was the great thing?
(i) to leave the house.
(ii) to make up his mind.
(iii) to argue unnecessarily
(iv) none of the above
Answer
(ii) to make up his mind.

Extract 3 The Proposal


"He is old, but I wouldn't take five Squeezers for him. Why, how can you? Guess is a dog, as
for Squeezer; well it's too funny to argue. Anybody you like has a dog as good as
Squeezer...You may find them under every bush almost. Twenty-five roubles would be a
handsome price to pay for him."
Question 1: Who does 'he' refer to?
(i) dog
(ii) cat
(iii) rabbit
(iv) squirrel
Answer
(i) dog
Question 2: Who does 'squeezer' belong to?
(i) Lomov
(ii) a neighbour
(iii) Chubukov
(iv) Lomov's uncle
Answer
(iii) Chubukov

Question 3: Who is the speaker of the above lines?


(i) Natalya
(ii) Chubukov
(iii) Lomov
(iv) none of the above
96
Answer
(iii) Lomov

Question 4: What price was the speaker quoting for 'him'?


(i) Twenty roubles
(ii) Twenty-five roubles
(iii)Twenty-two roubles
(iv) Twenty-three roubles
Answer
(iv) Twenty-three roubles

II Short answer questions (40- 50) words


1. Why does lomov wish to propose to Natalya?
2. What does Lomov think when he is alone?
3. What do you learn about Natalya from the play, ‘The Proposal’?
Answers:
1. Lomov wishes to propose to Natalya because he feels she is an excellent housekeeper, not
bad looking, and is also well- educated. Moreover, he feels that he is already thirty-five, which
is a critical age and he ought to lead a regular and settled life.
2. Lomov thinks that Natalaya is an excellent housekeeper, not bad looking, well educated. It
is impossible for him not to marry. In the first place, he is already thirty-five. He ought to lead
a quiet and regular life. He suffers from palpitations. He is excitable and always getting upset.
There are some other problems too which trouble him.

3.Natalya is the only daughter of the landowner Chubukov. She was very possessive about her
land and was very determined not to part with it. She was a short-tempered lady who used to
quarrel with anyone very often. She always pinpointed the amount of help she had offered to
her neighbours. Though well educated, she did not use her education wisely and thoughtfully.
III Long answer questions
1.Neighbours must have a cordial relationship that Lomov and Nathalya do not have. Describe/
justify/explain the first fight between them.
2. Justify the title of the lesson, ‘The Proposal’.
3. The principle ‘forgive and forget’, helps a lot in maintaining cordial relations with our
neighbours. Do you think Anton Chekov conveys this message in the play ‘The Proposal’?
Answers
1. Neighbours must have a cordial relationship but in the case of Lomov and Natalaya, it was
different. Lomov and Natalaya were rich people. They were wealthy people and had a
competition between them on the subject of wealth.
Both were neighbours. Lomov, at a critical age of thirty-five, thought that he had
to marry and found Natalaya not bad looking, an excellent housekeeper. So, he
decided to propose to her.
Lomov, in a well-dressed manner, wearing coat and gloves, one evening went to Chubukov’s
house. Chubukov was Natalaya’s father. When he came there and said that he wanted a help,
the help which could only be done by him. Chubukov, for first time suspected that he had
come for asking money. But after a long struggling and shivering, Lomov uncovers his need.
He had come to propose to Natalaya. Chubukov was mistaken and he was filled with extreme
joy and called Natalaya. Lomov asked then, whether Natalaya would agree for it or not. Then,
Chubukov said that, she would be ready to accept it because she was a lovesick cat.

97
When Natalaya arrived, she actually did not know that Lomov had come to propose to her.
They started talking and when they were talking about land, Lomov stated “Our Oxen
Meadows”. Then Natalaya said, “Our Oxen Meadows”, it is not your Oxen Meadows’, it
is ours. Lomov did not agree. But Natalaya initiated a heated argument. By arguing with
each other, they did not solve any problem. They got trapped in a long argument just
because of their lack of tolerance towards each other and their desire to keep their pride.
Neighbours must have a cordial relationship which Lomov and Natalaya do not have.
2. Anton Chekov has aptly and logically titled his one-act-play ‘The Proposal’. The tile
justifies the theme and the subject of the play. The whole story revolves round Lomov’s
marriage proposal to Chubukov’s daughter Natalya. The play ends with Chubukov asking
Natalya to hurry up and get married immediately. Natalya seals the proposal by kissing
and embracing Lomov. Chubukov willingly and happily blesses the couple.
The play begins with the visit of Lomov to Chubukov’s house. The moment has come
with a proposal of marriage, he becomes extremely happy. Lomov is genuinely
interested in marrying Natalya. He thinks that she is a good housekeeper, educated and
not bad looking. He sees in Natalya an ideal wife who can help him in leading a peaceful
and regular life. Natalya herself being a love-sick cat sees in Lomov a suitable life-
partner for her. The moment she comes to know that Lomov has come with a marriage
proposal, she blames her father for not telling it before. She threatens to die if Lomov
is not brought back immediately.
No doubt, there are quarrels and disputes over the ownership of Oxen Meadows and on
the superiority of their dogs. Abuses and insults are freely traded. But this farce can’t
hide the real intentions of the main characters. The play ends with the main characters.
The play ends with Natalya kissing and embracing Lomov and Chubukov blessing the
newly married couple.

3. Anton Chekov does convey the message that the principle of ‘forgive and forget’ helps a lot
in -maintaining cordial relationship with one’s neighbours. Initially, the neighbours in the play,
Lomov, Natalya and Chubukov were fighting over petty issues. They were putting important
issues behind them and wasting their time by arguing over small things. These arguments
slowly developed into fights and they started hurling abuses at one another. This damaged
their relationship with each other.

It was when Natalya came to know that Lomov had come to propose her that she left the topic
of argument (Oxen Meadows) behind. She also asked for forgiveness from Lomov and
requested him to come to the point. Hence it is mandatory to ‘forgive and forget’ if one wants
to have cordial relationship with others.

LITERATURE (POEM) = THE FIRST FLIGHT


P-1 Dust of Snow
Q3 Multiple Choice Questions based on Extract
(A) The way a crow Shook down on me The dust of snow From a hemlock tree Has given my heart
A change of mood And saved some part Of a day I had rued.

98
I Of the many symbols the hemlock tree represents, choose the one that Frost drew upon in all
likelihood, for this poem.
Symbol of

a) longevity.
b) togetherness.
c) healing.
d) protection.

ii Choose the option that lists the possible feelings of the poet prior to the experience shared in
the poem.

1) reassured
2) disappointed
3) curious
4) demotivated
5) thankful
6) disheartened
7) impulsive

a) 1, 3 &7
b) 2, 4 & 6
c) 5 & 7
d) 1 & 3
iii Identify the option that DOES NOT use the word ‘rue’ correctly.
a) The film was a disaster and he rued his decision to act in it.
b) I am sure she rued the day she listened to a fortune-teller.
c) It wasn’t long before I rued my disobedience and my deceit.
d) Others finally rue the one who is dishonest and heartless.
iv Synecdoche is a poetic device that uses a part to represent the whole. E.g. That’s a great set of
wheels! (Set of wheels has been used for car.)
Pick an example of synecdoche from the poem.
a) Has given my heart / A change of mood
b) The way a crow/ Shook down on me
c) The dust of snow / From a hemlock tree
d) And saved some part / Of a day I had rued
v Choose the option showing the reason NOT corresponding with “… a crow / Shook down on
me / The dust of snow”.
a) The crow’s landing on the branch of the tree.
b) The shivering of the crow, due to the cold.
c) The readjustment of position of the crow on the branch.
d) The cawing of the crow hidden in the foliage.
vi Rationalise what you think the poet was probably doing when the dust of snow fell on him?
99
vii What does the presence of a hemlock tree tell you about the setting of the poem?
viii The dusting of snow on account of the crow helped make the poet’s day better. How do you
think the crow’s droppings might have impacted the poet?
Q11 Answer in 40-50 words
i ‘Dust of Snow’ is one of Frost’s most loved poems. Elaborate why you think this is so.
ii The poem evokes a sense of black and white. Justify.
iii The crow and hemlock are usually used as negative references in literature. How is this
different in this poem?
Q12 Answer in 100-120 words (beyond text and across texts)
i As a crow you feel highly injured by the incorrect perception humans have about your species.
Imagine yourself to be a crow and write your opinion about this prejudice. Make a case for breaking
stereotypes.
ii Imagine that Frost is of present day and has been invited to write for an e- zine - “Teen Talk”.
He chooses to draw upon his experience with the crow and the dusting of snow to share his thoughts
on ‘Every cloud has a silver lining’.
As Frost, express your thoughts for the e-zine

P-2 Fire and Ice


Some say the world will end in fire, Some say in ice.
From what I’ve tasted of desire I hold with those who favor fire. But if it had to perish twice,
I think I know enough of hate To say that for destruction ice Is also great
And would suffice.
i Choose the CORRECT statement about the given poem.
a) Fire and ice are images—they help the readers visualise the power of nature over man.
b) Fire and ice are symbols—not of natural disasters, but of humanity’s ability to create disasters
of its own.
c) Fire and ice are elements—not of Nature but man-made and possess the ability to create havoc
for mankind.
d) Fire and ice are agents—they change the thinking of mankind from negative to positive and
bring harmony.
ii Select the option that correctly classifies the connotations for fire and ice, as suggested in the
poem.
(1) rage (2) violence (3) indifference (4) hate (5) greed a) Fire- 3,4; Ice- 1,2,5
b) Fire- 2, 5; Ice-1,3,4
c) Fire-1,3,5; Ice- 2, 4
d) Fire- 1,2,4; Ice- 3,5
iii The poem is a , put across by the poet.
a) powerful warning
b) heart-felt apology
c) earnest appeal
d) vengeful threat
iv The poet uses the phrasal verb -hold with.
Choose the option that DOES NOT indicate a valid phrasal verb.

a) option 1
b) option 2
100
c) option 3
d) option 4
v Pick the option that is NOT TRUE about the poet according to the extract.
The poet
a) is inclined to believe that the world would most likely end with fire.
b) has heard divided opinions about the way the world would end in all likelihood.
c) preaches love and kindness to combat the spread of hate among all.
d) declares the power of ice to be as destructive as that of fire.
vi Extra Q:
Identify the most likely tone of the poet in the lines- ‘To say that for destruction ice/Is also great’.

a) sarcastic
b) serious
c) amused
d) celebratory

Q10 Answer in 20-30 words


I "Fire and Ice" was first published in 1920. Briefly explain how the WW I might
have influenced the theme of the poem.
ii Create a tag line / slogan / catchphrase for the poem.
i The tone of the speaker contrasts with the seriousness of the subject matter. Justify.
ii Fire and Ice projects a pessimistic outlook. Comment.
iii Evaluate the line- Some say the world will end in fire/Some say in ice- in the context of
volcanic eruptions, forest fires, meteor collisions, melting ice caps etc.
Q12 Answer in 100-120 words (beyond text and across texts)
i Imagine that Nelson Mandela read ‘Fire and Ice’.
As Mandela, write a diary entry associating the impact of the troubled history of South Africa with
the idea expressed in the poem.
You may begin like this:
10 May 1994, Tuesday 10 p.m.
Today, when I am the President of South Africa, I can’t help recalling Robert Frost’s ‘Fire and
Ice’………………...(continue)………………….
ii Most of your classmates, like Frost, feel that man’s greed and rage or hate, would lead the
world to its destruction.
You feel that the ‘fire and ice’ in Mankind can be addressed to create a harmonious world.
Write a speech, for your classmates, expressing your belief/s with respect to the poem.
Answer key for MCQs
Dust of Snow Fire and Ice
Q3 (A) Q3 (A)
i) c i) b
ii) b ii) d
iii) d iii) a
iv) a iv) d
v) d v) c vi) a

101
A TIGER IN THE ZOO

Reference to the context.


Attempt extracts based questions.
He should be lurking in shadow
Sliding through long grass
Near the water hole
Where plump deer pass.

(i) Who does ‘He’ refer to?


(ii) Where is the tiger at present?
(iii) Give a word opposite in meaning from these lines to ‘light’.
(iv) Pick out the word from the extract that means the same as— “wait in hiding”.

SHORT ANSWER QUESTIONS (30-40 WORDS)

Answer the following questions in 30-40 words each.

Q.1. Which words does the poet use to show the tiger’s style of attack?
Q2. What would the tiger do in a forest?
Q3. How does the tiger act in the cage?

LONG ANSWER QUESTIONS (100-120 WORDS)


Answer the following questions in 100-120 words.
Q1. What message does the poet want to convey through the poem “a tiger in a zoo”?
Q2. How the poet has made use of the literary devices to differentiate between a tiger in the
zoo and in the forest?
Q3. “Freedom is a birth right. It is the most beautiful gift of god. It is valuable for all human
beings as well as for creatures,” Discuss with reference to the poem “A Tiger in the Zoo.”

Reference to the context.


Attempt questions based on extract.
He hears the last voice at night,
The patrolling cars,
And stares with his brilliant eyes
At the brilliant stars.
(i) Who does “he’ refer to?
(ii) What effect has been created by the use of repetition of the word brilliant?
(iii) Why does the poet use the word voice instead of sound of patrolling cars?
(iv) Pick out the word from the stanza that means the same as ‘gleaming’.

SHORT ANSWER QUESTIONS (30-40 WORDS)


Answer the following questions in 30-40 words
Q1. Why does the tiger express his rage quietly?
Q2. How does the tiger make his presence felt in the village?
Q3. What emotion does the poet want to highlight of the tiger by putting stress on “Quiet
rage”?
LONG ANSWER QUESTIONS (100-120 WORDS)

102
Answer the following questions in 100-120 words.
Q1. Comment on the repetition of “Brilliant” in the last stanza?
Q2. What images does the poet use in both cases as he describes the in a cage and in a jungle
and what effect does it have on the readers?
Q.3. A tiger in the poem is feeling miserable in concrete cell. Does it not amount to cruelty?
Express your opinion about keeping wild animals in zoos. Is there any lesson for humans?

Short Answer Questions (40-50 words: 3 Marks each)

1.Why do you think the tiger was stalking in the cage? What does it show?
Ans. I think the tiger was feeling restless and uneasy in that small cage. He seems to be frustrated and
helpless as he could not come out of the cage in the open and enjoy his freedom. He did not want to
be a mere showpiece and a source of entertainment for human beings.
2.Describe the movement of the tiger in the cage and in the wild.
Ans. In the zoo, in his small cage and devoid of freedom, the tiger feels unhappy rather frustrated,
restless and angry. In the forest, he enjoys moving majestically, wherever he wants, terrorises the
villagers by displaying his sharp teeth and claws. He is happy in the forest, enjoying his liberty and
surroundings, but not in the zoo.
3. What does the tiger do at night? What does he feel when he stares at the brilliant stars in the
sky?
Ans. The patrolling cars move around and the tiger hears their sound late at night. He is locked in a
cage but stares at the brilliant stars shining in the vast and open sky. The vast open sky and the brilliant
stars only intensify the feeling of helplessness that he feels inside the cage.
4. Freedom can't be bargained at any cost. What message does the poet give to the readers in
'A Tiger in the Zoo?
Ans. Not only humans but animals too cherish freedom. Freedom can't be bargained. In this case, a
tiger may be well-fed and protected. However, the curtailment of his freedom keeps him in 'quiet
rage'. He resents being behind the bars. He is a different animal when he is in his natural habitat, the
jungle. He roams around the water hole and ambushes his favourite plump deer.
5. Do you think the tiger in the poem had lost its natural instinct due to captivity? Support your
response with evidence from the poem.
Ans. Tiger is known for its sudden attack on other animals and humans. But in the zoo, even though
he is surrounded by people, he is not attacking anybody. He is full of anger but his rage has turned
quiet. Thus, we can say that the tiger in the poem had lost its natural instinct due to captivity.

Long Answer Questions (100-120 words: 6 Marks each)

1. Some animals are becoming extinct; the tiger is one such animal. What do you think could be
the reason? Should this decrease in number be stopped? Why?
Ans. The primary threats to the survival of tigers are poaching and habitat loss due to intensive
development. The tigers are getting extinct. They are on the list of endangered species and the reason
behind it is human interference. Humans hunt tigers for their commercial benefits. The loss of habitat
also leads to a reduction in prey animals, so the area can support fewer tigers. Moreover, tigers are
poached for trophies, such as teeth and fur, and for traditional medicines. The growing population
clears the forest areas to make them agricultural land for food. This creates a problem for the tiger's
habitat Tigers are the backbone of food web and forest, hence should be protected.

103
2. The tiger in the poem is feeling miserable in a concrete cell. Does it not amount to cruelty?
Express your opinion about keeping wild animals in zoos. Is there any lesson for humans?
Ans. The tiger in the poem is wretched in its cage. It longs for freedom. It may be well looked after,
but the fact of the matter is that unless one is free, one is not alive. Confinement brings bondage, and
bondage is cruelty. One may argue that at least this way they all will not be killed and become extinct.
However, taking away one's freedom to keep one alive kills the desire to live anyhow. Even humans
throughout the world oppose the chains of slavery and oppression. How are other living creatures any
different? Humans have encroached on their space, and sheltering them in zoos is truly inhuman.
Humans must learn to respect nature, for humans exist only due to nature.
3. Imagine that man has been accused by the tiger of cruelty in an animal court. What would
be the tiger's two major accusations and how would man defend them?
Ans. Tiger's accusations:
1. Humans have been killing us for our velvety skin and have been doing illegal trade of our body
parts.
2. They have encroached on our natural habitat.
Man's accusations:
1. Our ancestors had hunted them to show off their manhood. We have now prohibited all sorts of
hunting and killing of animals.
2. We are making zoos and national parks to safeguard all species of animals.

HOW TO TELL WILD ANIMALS?

Extract based question


A. Read the following extract and answer the questions that
follow. (1x5=5)
If ever you should go by chance
To jungles in the east;
And if there should to you advance
A large and tawny beast,
If he roars at you as you’re dyin’
You’ll know it is the Asian Lion ….
(a) What are the features of an Asian Lion?
(b) What happens to you when the lion roars?
(c) What do you mean by ‘tawny’?
(d) Where is the Asian Lion found?
(e) Name the poem and its poet.
B.Read the following extract and answer the questions that
follow. (1x5=5)

If strolling forth, a beast you view,


Whose hide with spots is peppered,
As soon as he has lept on you,
You’ll know it is the Leopard.
Twill do no good to roar with pain,
He’ll only lep and lep again.

(i) How can you recognise the Leopard?


(ii) How is it different from the Bengal Tiger?
(iii) Pick out the word from the stanza that means the same as—‘a large animal of the cat
family
104
(iv) Name the poem and poet.
(v) Write synonym of 'strolling'.

Short Question (40-50 words -3 MARKS)


A. How does the poet distinguish the hyena from crocodile?
B. How does the poet suggest that you identify the lion and the tiger? When can you do
so, according to him?

Long Question (100-120 words -6 MARKS)


A. The poet has used some special qualities to describe the animals and create humour in
the poem. Each of us also has some special qualities which make us quite different from
others. Do you agree with this statement? Comment with reference to the poem ‘How
to Tell Wild Animals’.

B. Much of the humour in the poem arises from the way language is used, although the ideas
are funny as well. If there are particular lines in the poem that you especially like, share these
with the class, speaking briefly about what it is about the ideas or the language that you like
or find funny?

Short Answer Questions (40-50 words: 3 Marks each)


1. What is the theme of the poem, 'How to Tell Wild Animals?
Ans. The theme of the poem is to create humour. The poet creates humour by suggesting dangerous
ways of identifying wild animals. You can identify most of the beasts while they are attacking you.
Instead of shouting for help or trying to protect yourself, you are busy identifying the attacker-the
idea creates humour.
2. Why does the poet say that a bear's 'hug' may confuse a novice?
Ans. It is the tendency of a bear to kill his victim by embracing him. He hugs the victim so tight that
he usually dies. But a novice who does not know the ways of bears may misunderstand his hug. He
may think that it is a loving embrace.
3. What does the poet say about the crocodile and the hyena?
Ans. The poet describes the crocodile and the hyena humorously. The hyena seems to be laughing. In
fact, it does not laugh. Only its face appears so. On the other hand, the crocodile has tears in his eyes
when it eats its victim. It appears as if he is shedding tears at the death of his victim. But this is not
so.
4. Which genre is most appropriate for the poem? Substantiate your choice with reference to
the poem "How to Tell Wild Animals".
Ans. It's humour which I think the entire poem is replete with. The incorrect lines, utter surprises
waiting for its readers and the narration of wild beasts in a funny way make it humorous. The way the
poet establishes the difference between a hyena and a crocodile distinctly demonstrates its humorous
part.
5."All knowledge is useful. But not all knowledge is worth the cost." Elaborate on the quote in
the context of the poem "How to Tell Wild Animals".
Ans. The poet has humorously cautioned against the wild beasts found in the jungle. She emphasises
the lethal attacks of different animals and says we need to understand the ways how they prove to be
fatal. It's important to note their treacherous ways and the knowledge of the same could keep us safe
from the impending danger.
Long Answer Questions (100-120 words: 6 Marks each)

105
1. Many animals can be identified according to the poet's suggestion. Name the animals, Which
ones would you like to identify? Are there any lessons for us from this poem?
Ans. The poet has listed down the ways of identifying seven animals in the poem, viz., the Asian lion,
the Bengal tiger, the leopard, the bear, the hyena, crocodiles and chameleons As for me, I would like
to identify the bear, the Bengal tiger and the leopard. However, putting my life on the line just to
identify the tiger and the leopard doesn't seem worth the risk! So, if I see a noble beast with black
stripes on yellow. I'll know the tiger, and the peppered spots will reveal the leopard. The bear hug, I
wouldn't mind! The poet, using humour, seems to warn readers against venturing too close to these
animals.
2. The poet has successfully used humour to be able to tell or identify 'wild animals. How do
you think we need lots of it in our daily life?
Ans. Humour is the fuel of life. Without it, it would become difficult to continue living. Today, the
life of a common man is typically mired in stress, tension, problems and sadness It is amazing how
the smallest problems may take on epic proportions for a common person. In such a situation, it
becomes necessary to have a little humour in life. It gives us the much-needed respite from our
monotonous routine and helps us face life more positively. Thus, the poet's successful use of humour
helps liven our minds and brings a smile on our faces.
3. Imagine the poet meets Mijbil, the otter. Write a detailed account of the characteristics the
poet could use for her poem, if she were to write about telling an otter.
Ans. Let me acquaint you with the most adorable animal Mijbil who was neither hostile nor friendly.
Mijbil plunged and rolled making enough slosh and splash for a hippo. Mijbil's actions were similar
to that of human-like characteristics. I resemble Mijbil who is almost akin to a description of a cute
and playful child. A child in his initial age is believed to be restless, curious and aspires to get
everything new he encounters. He loves new associations and shows his/her likings or disliking.
Mijbil too had developed certain habits and remained playful with me. He liked my company and
loved running and touching thins in the street. He was just energetic and naughty who brought a lot
of love in my life.
THE BALL POEM

John Berryman

Read the following extracts carefully and answer the questions that follow.
Question 1.
What is the boy now, who has lost his ball,
What, what is he to do? I saw it go
Merrily bouncing, down the street, and then
Merrily over – there it is in the water!
No use to say – ‘O there are other balls’

(a) What has happened to the boy?


(b) Why does the poet say ‘No use to say – ‘O there are other balls’?
(c) Which word means ‘happily’?
(d) Where did the ball go?
Answer:
(a) The boy was very sad as he has lost his ball.
(b) The poet says so as the loss of the ball is of a major consequence to the boy.
(c) Merrily means happily.
(d) The ball went to the water.

Question 2.
106
An ultimate shaking grief fixes the boy
As he stands rigid, trembling, staring down
All his young days into the harbour where
His ball went. I would not intrude on him,
A dime, another ball, is worthless.

(a) Where had the boy’s ball gone?


(b) How has the loss affected the boy?
(c) Why was ‘he’ trembling, staring down?
(d) What does the poet mean by ‘first responsibility’?
Answer:
(a) The boy’s ball had gone into the dark waters of the harbour.
(b) The boy is very much troubled at the loss of his ball. He stands still, trembling and staring
at his ball.
(c) He was trembling, staring down as he had lost his ball.
(d) By ‘First responsibility’, he, means to look after his things properly.

Question 3.
……………Now
He senses first responsibility
In a world of possessions. People will take balls,
Balls will be lost always, little boy.
And no one buys a ball back. Money is external.

(a) What does the boy understand?


(b) What does the word ‘balls’ signify?
(c) What is meant by the word ‘possessions’?
(d) What does “In a world of possessions mean?”
Answer:
(a) The boy learns to grow up and understands his first responsibility.
(b) The word ‘balls’ signifies that the boy’s innocence and happy young days.
(c) ‘Possessions’ means something that is owned or possessed.
(d) It means that everybody loves to possess things or materials in their names.

Question 4.
He is learning, well behind his desperate eyes,
The epistemology of loss, how to stand up
Knowing what every man must one day know
And most know many days, how to stand up.

(a) What is the boy learning?


(b) Why are boy’s eyes desperate?
(c) What do you mean by ‘epistemology of loss’?
(d) What every man needs to know one day?
Answer:
(a) The boy is learning to cope up with the loss.
(b) Boy’s eyes look desperate as he is sad to see his ball gone forever.
(c) ‘Epistemology of loss’ means to understand the nature of loss.

107
(d) Every man needs to know one day that loss may occur to him and he would have to bear
it up.

Short Answer Type Questions


Question 1.
What does John Berryman want to convey through this poem?
Answer:
Poet, John Berryman wants to convey the importance of loss and responsibility in life. We all
should learn our responsibility and how to cope up with the loss.
Question 2.
How does the boy feel at the loss of his ball?

Answer:
The boy is very much troubled at the loss of his ball. He experiences grief at the loss of his
much-loved possession. Like a statue, he keeps staring at the ball with his desperate eyes.
Question 3.
Write the sum and substance of the poem “The Ball Poem”.
Answer:
In “The Ball Poem”, Berryman tells us about how our childhood can quickly fly by, as quickly
as a ball is lost and how we sometimes unsuspectingly must grow up and face hardships, like
loss.
Question 4.
“Money is external”. What does the poet mean by this expression?
Answer:
The poet makes the boy understand about his responsibility as the loss is immaterial. Money
is external as it cannot buy memories, nor can it replace the things that we love, the things that
really matter.
Question 5.
Why does the poet think that it is useless to give the following suggestion to the boy?
‘No use to say- ‘O there are other balls’:
Answer:
According to the poet, it is useless to console the boy by saying that he can get another ball in
place of the lost one. The boy had a long association with the ball. It was, thus, useless to give
him such a suggestion because he wanted to get back the ball that he had lost.
Question 6.
Why did the poet not offer the boy money to buy another ball?
Answer:
The poet watched the boy who had plunged in grief at the loss of his ball. He did not offer the
boy money to buy another ball. He felt that another ball could not console the boy. It seemed
that the boy had the ball for a long time. The poet also wanted the boy to realise the
epistemology of loss.

Long Answer (Value Based) Type Question


Question 1.
Why is it important for everyone to experience loss and to stand up after it?
Answer:
It is important for everyone to experience loss and to stand up after it in order to be strong and to get
on with life. One needs to stay strong no matter how much it hurts inside. Staying strong is the only
way to survive. Moreover, one needs to learn to accept and let go and not cling to something that they

108
can never have. One should understand that the past is gone and it will never come back. Experiencing
loss sometimes helps us to grow up and face hardships, like loss. This helps us in breaking all the
boundaries into freedom.
Question 2.

How is the lost ball, the metaphor of the lost childhood of the boy? Why doesn’t the poet
want to ‘intrude on’ the boy by offering him money to buy another ball?
Answer: The boy has a ball. Perhaps he has been keeping it for a long time. He must have
developed a lot of attachment and love with the ball. Suddenly while he is playing, the ball bounces
down the street. And after a few bounces, it falls down into the harbour. It is lost forever. The boy
stands there shocked and fixed to the ground. He constantly goes on staring at the spot where his
ball fell down into the water. Outwardly, the loss seems to be quite small. The boy seems to be
making a fuss over the loss. Many boys have lost such balls and will lose so in future. A new ball
can be easily bought in a dime.
The metaphor of the lost ball is beautifully linked to the loss of sweet childhood. No amount of money
can buy the ball back that has been lost forever. Similarly, no worldly wealth can buy back the lost
childhood. The poet doesn’t want to sermonize on this issue. The boy himself has to learn
epistemology or the nature of the loss. He has to move ahead in life forgetting all the losses he has
suffered in the past.
Question 3.

What is the epistemology of loss in this world of possessions? How has the child learned
to stand up in life?
Answer: Gain and loss are the two sides of the same coin. Getting, spending and losing things form a
natural cycle of life. The boy is inconsolable at the loss of his ball. Actually, it is not the ordinary ball
but his long association and attachment with it that makes the loss so unbearable. It is like the good
sweet days of childhood that the boy cherishes so much but are lost and gone forever. They will never
come back again.
So, what is the remedy? He can bear this loss by understanding the epistemology or nature of the loss.
In this world of material wealth and possessions, it seems that money can buy anything. However, it
is a false conception. Money has its own limitations. Its nature is external. It cannot compensate for
the losses that a person suffers emotionally or internally. No wealth can buy back the ball that has
been lost forever. Similarly, no wealth can buy back the lost childhood. The child will have to move
ahead and stand up in life. He has to stop weeping over his past losses and start living life as it should
be lived.

P-6 AMANDA! TEXT-FIRST FLIGHT


Q3 Multiple Choice Questions based on an extract
(A) (I am an orphan, roaming the street.
I pattern soft dust with my hushed, bare feet. The silence is golden, the freedom is sweet.)
i The tone of the given lines is
a) analytical.
b) despairing.
c) peaceful.
d) nervous.
109
ii Read the statements A and B given below, and choose the option that correctly evaluates these
statements.
Statement A – The figure ‘I’ imagines a less than realistic view of being an orphan.
Statement B – The figure ‘I’ does not like the speaker.
a) A is true, B is false, according to the extract
b) A is true, B cannot be clearly inferred from the extract.
c) A cannot be clearly inferred from the extract, B is false.
d) A is true and can be inferred from the poem, B is true too.
iii The golden silence is contrasted with the ___
a) chaos of the street.
b) constant instructions received.
c) sweetness of freedom.
d) hushed, bare feet.
iv The rhyme scheme ‘aaa’ in the above extract is followed in all other stanzas of the poem that
are written in parenthesis, i.e. (). Why?
Read the reasons given below, and choose the option that lists the most accurate reasoning:
(i) It shows the simplicity of the child’s thoughts.
(ii) It reflects the harmony and rhythm of the child’s inner world.
(iii) It mirrors a child’s expression.
(iv) It highlights the poet’s aesthetic sensibility.
a) (i) and (iv)
b) (i) and (ii)
c) (ii) and (iii)
d) (iii) and (iv)
v Pick the option that lists the usage of the word ‘pattern’, as in the extract above.
a) That is a lovely pattern for a wallpaper.
b) He decided to wear a patterned shirt to the party.
c) Poetry is a form of pattern making.
d) She patterned her hair after her favourite celebrity.
vi Extra Q.
Which option completes the popular adage given below?
silence is gold.
a) Precious are words for
b) Speech is silver
c) Silver is the tongue
d) Ideas may be precious but
vii Extra Q.
A transferred epithet is a literary device where the modifier or epithet is transferred from the noun it
is meant to describe to another noun in the sentence.
Based on the given definition of ‘transferred epithet’, choose the option that lists an example of
transferred epithet.
a) soft dust
b) hushed (bare) feet
c) freedom (is) sweet
d) silence (is) golden
viii Extra Q.
Which image corresponds to what Amanda, is doing in this extract?

110
a) option 1
b) option 2
c) option 3
d) option 4
(B) Don’t bite your nails, Amanda!
Don’t hunch your shoulders, Amanda! Stop that slouching and sit up straight, Amanda!
I The purpose of the speaker’s words in the given extract is to
a) show the speaker’s power over the listener.
b) make the listener a better human being.
c) advise the listener as an elder.
d) improve the listener’s posture and habits.
ii Pick the option that lists the image which correctly corresponds to the speaker’s mood in the
extract.

a) Image (i)
b) Image (ii)
c) Image (iii)
d) Image (iv)
iii Alliteration is a literary device used in the extract.
Which of the following options DOES NOT include examples of this literary device?
a) The moon and the shimmering stars watched over us
b) With that charming chat, Catherine chose comfort
c) Away ran the pathetic pooch pouting like a princess
d) Dee dee was driving down day after day
iv What does the repetition of “Amanda!” at the end of each line reflect?
a) It describes who the speaker is talking to.
b) It represents the absent-mindedness of the listener.
c) It shows the frustration of the speaker.
d) It helps create a rhyme scheme.
v Select the option that fits with the following: slouching: straight: :
a) transparent: translucent
b) lazy: agile
c) forgetful: lively
d) generous: liberal
Q10 Answer in 20-30 words
i Would you call Amanda a disrespectful child? Provide one reason to justify your opinion.
ii What kind of an image does “languid, emerald sea” evoke?
iii What does the line “never let down my bright hair” tell us about Amanda?
iv The reader sympathises with the speaker in the poem. Support this opinion with a reason.

111
v What do you think is the significance of the exclamation mark in the title of the poem
‘Amanda!’?
Q11 Answer in 40-50 words
i How would you characterise the speaker in the poem ‘Amanda!’? List any two qualities,
supporting the reason of your choice, with evidence from the poem.
ii ‘Amanda is alone but not lonely in the world she envisions.’ Justify the statement with
reference to any one image she pictures in her mind.
iii This is Calvin, a character from a comic strip created by Bill Watterson.
Do you think this might be something Amanda would say? Why/ why not? Give reasons.
Q12 Answer in 100-120 words (beyond text and across texts)

i Look at the given image of a book cover.

If the author, Jason Kotecki were to ask a grown-up Amanda to help with his book by writing one of
the 8 secrets based on her experiences, what might she share?
Elaborate based on the poem your understanding of

ii Read the quote given below carefully. Based on your understanding of the poem, explain how
and why this quote can be applied to the poem.
iii What advice might “chatterbox” Anne (Frank) have for the quietly imaginative Amanda?
Present this as a conversation between the two.
You may begin this way:
Anne: Gosh! You do run off into some strange worlds, don’t you?
Amanda: (nods) Well…
iv Colin Wilson, an English author, said, “Your imagination should be used not to escape reality
but to create it.”

Compare / contrast Wanda and Amanda in the light of this author’s opinion
Answer key for MCQs
Q3 (A) Q3 (B)
i) (c) i) (d)
ii) (b) ii). (c)
iii). (b) iii. (a)
iv). (c) iv. (c)
v). (d) v. (b)
112
vi) (b)
vii) (b)viii) (b)
POEM- Trees

Read the following extracts carefully and answer the questions that follow:
Question 1:
The trees inside are moving out into the forest, the forest that was empty all these days where
no bird could sit no insect hide no sun bury its feet in shadow the forest that was empty all
these nights will be full of trees by morning.
a. From where are the trees moving out into the forest?
b. Why can’t birds sit or insects hide in them?
c. How was the forest ‘all these nights’?
d. Are these trees useful for birds and insects?

(ii) Short Answer Type Questions


Question 1: What is the central idea of the poem, ‘The Trees’?
Question 2: Why is the description of the moon different in the beginning and at the end of the
third stanza?
(iii) Long Answer Type Questions
Question 1: ‘Departure is painful’. So is the departure of the trees for the poet. What will
happen after their departure?
Question 2: How does the poem ‘The Trees’ make a strong plea against deforestation?
Question 1: How does the poem ‘The Trees’ make a strong plea against deforestation?

FOG

i. Difficulties come but they are not to stay forever. They come and go. Comment referring
to the poem “Fog’.
Answer:
None can deny that fact that after every nightfall, there is sunshine. In other words, sorrows
follow happiness. Life has many ups and downs. We must all accept that problems as a part
of life. We should strengthen ourselves so much that we are able to face them and stand up
to solve these problems. We should not get carried away. We should learn that time and tide
wait for none.
Fog is also symbolic of darkness but it comes quietly like a cat and goes away. It symbolises
that if we are not disturbed by difficulties then life will itself move to lighter side and things
will be easy to confront. We should always remember that, we must face all the problems
boldly.
II. Write the central idea of the poem “Fog”.
Answer:
The poet Carl Sandburg in his poem ‘Fog’ describes fog as a cat. Fog is treated to be a living
creature. Fog comes quietly and stealthily like a cat. Fog sits looking over the harbour like a
cat does. Then it moves to settle somewhere else. Just as cat doesn’t settle at one place and in
the same way fog keeps on moving and finally vanishes.

EXTRACT 1
113
Extract 1:
The fog comes
on little cat feet.
It sits looking
over harbour and city
on silent haunches
and then moves on.
i) Choose the option with qualities of the cat that Carl Sandburg applies to the fog?
1) Cats are independent animals, they don’t follow rules, they slip and slide in and out of our
lives as they please.
2) Cats are distrustful of strangers and can be jealous and moody.
3) Cats are stealthy, moving in slow motion at times and they appear to be moving in a
mysterious fashion.
4) Cats often communicate with a combination of a distinctive sound and body language.
5) Cats like to move on at their own pace and before you know it, they’ve disappeared.
a) Only 1 b) 2, 3 and 4 c) 1,3 and 5 d) Only 4

Correct Answer: Option (c)

ii) Which one/s of the following applies to the given lines?

a) 1 and 4 b) 2 and 3 c) Only 1 d) Only 3


Correct Answer: Option (d)
iii) Pick the option that includes an image of the cat on its haunches.

a. Option (1) b. Option (2) c. Option (3) d. Option (4)


Correct Answer: Option (b)
114
iv) Choose the option that DOES NOT list the movement of the fog.

a. Option (1) b. Option (2) c. Option (3) d. Option (4)


Correct Answer: Option (c)
v) The poet has used short lines to compose the given poem. Choose the option that lists
the most appropriate explanation for the same.
a. By keeping the lines short, the poet keeps the liberty in stepping away from rule, traditional
form, logic, or fact, in order to produce a desired effect.
b. By keeping the lines short, the poet wants the reader to omit some parts of a sentence, which
gives the reader a chance to fill the gaps while reading it out.
c. By keeping the lines short, the poet wants to introduce ambiguity or contradiction into an
otherwise straightforward sentence.
d. By keeping the lines short, the poet is controlling the pace to make the reader slow down
thereby reflecting the slow rolling in of the fog.
Correct Answer: Option (d)

Poem-The Tale of Custard The Dragon


1-Pistol in his left hand, pistol in his right,
And he held in his teeth a cutlass bright,
His beard was black, one leg was wood;
It was clear that the pirate meant no good.
(i) Who is ‘He’ in this stanza?
(ii) What was the emotional condition of all members of house to see a man with pistol?
a) happy
b) sad
c) fearful
d) overjoyed
(iii) What was his intention?
(iv) Name the poem and poet.
(v) The poet has used a poetic device in the given line/s. Name the name poetic device from
given options.
‘Pistol in his left hand, pistol in his right,
And he held in his teeth a cutlass bright,’
a) alliteration
115
b) simile
c) metaphor
d) personification
4-Answer any THREE of the following in about 40-50 words each.
1.Why did Belinda and others flee away to see pirate except Custard?

2. How is ‘The tale of Custard the Dragon’ a ballad?

First Flight Poem For Ann Gregory


Reference to Context
Extract 1
“Never shall a young man,
Thrown into despair
By those great honey-coloured
Ramparts at your ear,
Love you for yourself alone
And not your yellow hair.”
(i) Who is the speaker in this stanza?
(a) Anne Gregory
(b) Poet
(c) An Old man
(d) A young man
Answer: (b)
(ii) Why the young man will not be in despair?
(a) Because of her Honey coloured hair
(b) Because of her Black coloured hair
(c ) Because of her Brown coloured hair
(d) None of the above
Answer: (a)
(iii) The word Rampart means ___________
(a) ears
(b) eyes
(c ) Wide Walls
(d) hill
Answer: (c)
(iv) Name the poet.
(a) Anne Gregory
(b) W B Yeats
(c ) John Keats
(d) Ogden Nash
Answer: (b)
(v) The poet is addressing ____________ in the above lines.
(a) Himself
(b) Old Lady
(c ) The Young Man
(d) Anne Gregory
Answer: (d)
116
Extract 2
“But I can get a hair-dye
And set such colour there,
Brown, or black, or carrot,
That young men in despair
May love me for myself alone
And not my yellow hair.”

(i) Who is the speaker in these lines?


(a) Himself
(b) Old man
(c ) The Young Man
(d) Anne Gregory
Answer: (d)
(ii) What colour would she give her hair?
(a) Brown
(b) Black
(c ) Carrot
(d) All of the above
Answer: (d)
(iii) Why is she ready to do so?
(a) wants others to love her for who she really is
(b) for a change
(c ) for look different
(d) to make others happy
Answer: (a)
(iv) Name the poem
(a) Daffodils
(b) The Fog
(c ) Amanda
(d) For Anne Gregory
Answer: (d)
(v) Despair means ______
(a) desperate
(b) happy
(c ) disappointed
(d) careful
Answer: (c)
Extract 3
“I heard an old religious man
But yesternight declare
That he had found a text to prove
That only God, my dear
Could love you for yourself alone
And not your yellow hair.”
(i) Who is ‘I’ in the first line?
(a) Poet
(b) Anne Gregory
(c ) Old man
117
(d) Young man
Answer: (a)
(ii) What did he find yesternight?
(a) everyone can love us
(b) only our relatives can love us
(c ) only God could love us for ourselves alone
(d) man looks at the inner beauty
Answer: (c)
(iii) What does the poet convey in the last three lines?
(a) They love someone only for the external beauty
(b) The poet conveys that human beings are incapable of seeing inner beauty
(c ) Only God can love someone for the inner beauty.
(d) All of the above
Answer: (c)
(iv) Name the poem and poet.
(a) Daffodils , William Wordsworth
(b) The Fog, Carl Sandburg
(c ) Amanda, Robin Klein
(d) For Anne Gregory, W B Yeats
Answer: (d)
(v) Yesternight means __________
(a) Today night
(b) Tomorrow night
(c ) Yesterday night
(d) None of the above
Answer: (c)
Short Answer type
Question 1.
What does the young woman wish?
Answer:
The young woman wishes that she should be loved for herself alone and not for her yellow
hair. She says that she could dye her hair in brown or black or carrot coloured.

Question 2.
Would the young man love her for herself alone?
Answer:
The young man will never love her for herself alone, though he might love her yellow hair.

Question 3.
What did the religious man tell the poet?
Answer:
The religious man told the poet that he found a verse to prove that only God could love her for
her inner beauty and not for her physical beauty.

Long Answer Type


Question 1.
The poet in the poem ‘For Anne Gregory’ conveys that We should give importance to the inner
beauty and not to the physical appearance. Explain with reference to the poem.
Answer:
118
It is an accepted fact that external beauty is short lived but inner beauty remains forever. Inner
beauty gives satisfaction and joy. We should thus learn not to accept things at their face value.
We should try to. discover the inner beauty and strength of a person, though it is not easy to
do so. We should accept the person with positive and negative values.
As the time passes relations are strengthened. In the poem ‘For Anne Gregory’ she wants that
her lover should love her for herself not the colour of the hair. She wants to strengthen this
fact that inner beauty is more important than external beauty.
Question 2.
“All that Glitters is not Gold”. Explain with reference to the poem “For Anne Gregory” written
by W. B. Yeats.
Answer:
What we see through the naked eye might not always be the naked truth. Like shining pieces
of glass that shine and glitter like diamonds, the way people appear on the outside can be very
misleading. We must never judge a book by its cover as there is a possibility that we might be
wrong.
Appearances should never be trusted. It can lead us to wrong conclusions. A person may be
very handsome or beautiful outside but inside if the character is not good that person’s
appearance is of no value. So always try to look beneath the skin because beauty is skin deep.
The real beauty lies in the personality of the person, not in external appearances.
Question 3.
What does the young man mean by “great honey-coloured / Ramparts at your ear?” Why does
he say that young men are “thrown into despair” by them? ”
Answer:
The “great honey-colored / Ramparts at your ear” refers to the beautiful yellow coloured hair
that falls at the woman’s ear and cover it like a wall around a fort. He says that the young men
are “thrown into despair” by them because they look so beautiful on the women that her beauty
gets thoroughly enhanced. The young men fall in love with her and feel despair. He says that
it is not possible that someone would love her alone and not her yellow hair.

FOOTPRINTS WITHOUT FEET


Footprints without Feet: Chapter 1- A Triumph of surgery
.Reference to context questions.
EXTRACT 1
I tried to sound severe: “Now! really mean this. If you don’t cut his food right down and
give him more exercise he is going to be really ill. You must harden your heart and keep
him on a very strict diet”.
Question:
(a) Why did the speaker try to sound severe?
(b) For whom was the advice given and why?
(c) Find the word in the extract which is a synonym of the word ‘serious’.
(d) What is the antonym of ‘hardened’?

Answer:
(a) The speaker tried to sound severe to make Mrs Pumphrey take his advice seriously and
act on it.

119
(b) The advice was given for Tricki because he had become obese and listless. He was
unwell and the speaker wanted him to be on a strict diet.
(c) The word is ‘severe’.
(d) Its antonym is ‘softened’.
Extract 2
As I moved off, Mrs Pumphrey, with a despairing cry, threw an armful of the little coats
through the window. I looked in the mirror before I turned the corner of the drive;
everybody was in tears.
Questions:
(a) Where was the speaker going and with whom?
(b) Why was everybody in tears? ‘
(c) Find the word in the extract which is an antonym of the word ‘cheerful’.
(d) What is the opposite of ‘before’?

Answers:
(a) The speaker was going to the hospital with Tricki.
(b) Everybody was in tears because Tricki was being hospitalised as he was ill. Everyone
was worried about his health.
(c) The word is ‘despairing’.
(d) ‘After’ is its opposite.
Extract 3
1. Mrs Pumphrey hastened to explain. “He was so listless, Mr Herriot. He seemed to have
no energy. I thought he must be suffering from malnutrition, so I have been giving him
some little extras between meals to build him up, some malt and cod-liver oil and a bowl
of Horlicks at night to make him sleep nothing much really.”
(a) ‘He’ in the given extract is:
(i) Guess
(ii) Tricki
(iii) Squeezer
(iv) Hodgkin
Answer: (ii) Tricki
(b) He was supposed to be suffering from:
(i) depression
(ii) fever
(iii) malnutrition
(iv) All of these
Answer: (iii) malnutrition
(c) Mr Herriot was:
(i) a veterinary surgeon
(ii) a physician
(iii) a philosopher
(iv) a psychologist
Answer: (i) a veterinary surgeon
(d) Who is the author?
(i) Ruskin Bond
(ii) H.G. Wells
(iii) K.A. Abbas
(iv) James Herriot
Answer: (iv) James Herriot

120
(e) The word ‘hastened’ means:
(i) disliked
(ii) hurried
(iii) worried
(iv) agreed
Answer: (ii) hurried
II Short answer questions (40- 50) words
1. Why does Mrs Pumphrey worried about Tricky?
2. What special diet did Mrs Pumphrey give to build Tricky up?
3. What kind of a person do you think the narrator, a veterinary surgeon, is ? Would you
say he is tactful as well as full of common sense?
Answers:
1. Mrs Pumphrey, the mistress of Tricki, is really worried about her dog. He has grown
quite fat and listless. He has lost his appetite. He often vomits and lies down motionless
on the carpet. His eyes have became blood-red and rheumy Naturally Tricki’s condition
is a cause of deep concern for Mrs Pumphrey.

Mrs Pumprey found that Tricki had became listless. He seemed to have no energy. She
thought he must be suffering from malnutrition. She decided to give some little special diet
between meals to build him up. She gave him some malt, cod-liver oil and a bowl of Horlicks
at night. She couldn’t refuse cakes and chocolates to Tricki even after being advised to cut
down on the sweet things.
The narrator, a veterinary surgeon, is a capable and wise doctor. He handles Tricki properly
which shows that he is sincere about his profession. He does not operate upon the dog
unnecessarily and cures him by altering his diet and activities. He is a tactful person as he
knew exactly how to convince Mrs Pumphrey to hospitalise Tricki for a surgery. His decision
to inform Mrs Pumphrey to collect Tricki supports his sensibilities even though he was
tempted to keep Tricki as a permanent guest.

III Long answer questions


1. Herriot enjoyed the lavish meal during Tricki's stay but still, he felt compelled to inform
Mrs. Pumphrey about the recovery of Tricky. Why did he not keep Tricki for a longer
time? Did he feel concerned for Mrs. Pumphrey?
2. Give a character sketch of Dr.James Harriot highlighting his common sense, practical
nature and competence as a veterinary surgeon.
3. Herriot is a dutiful doctor who cared for others 'emotions' more than his personal
interests. Prove the above statement.

Answers
1. Tricki was a fat dog, pet of Mrs. Pumphrey. He became listless and seemed to have no energy. It
fell ill. So, Mrs. Pumphrey called the veterinary surgeon, Dr Herriot for the help. Dr Herriot took
Tricki to his surgery. He kept an eye on him for two days and gave him no food but plenty of
water. At the end of the second day, he started showing some interest in his surroundings and on
the third day, began to whimper on the sound of dogs. While his stay at the surgery, Mrs.
Pumphrey started to bring round fresh eggs to a dozen at a time. She also sent bottles of wine and
brandy for Tricki's health. But that was consumed by Mr. Herriot and his colleagues. Hence,
Herriot enjoyed the stay of Tricki. He did not want to keep Tricki for a longer time because he felt
concerned for Mrs. Pumphrey. He did not want to separate the pet from his owner. He could
understand Mrs. Pumphrey's concern for the little dog. So he returned him back.

121
2. Dr.James Herriot, no doubt, was a competent veterinary surgeon. He was really worried
about Tricki. He understood that the real fault of the dog was his greed for food. He never
refused food. The dog had become hugely fat and listless. Dr Herriot instructed Mrs
Pumphrey to keep Tricki was on a very strict diet. He also asked her to give him a lot of
exercises. Dr Herriot also suggested to cut down sweets to him. He advised her that Tricki
must be hospitalised for a fortnight under his observation. Dr. James Herriot was practical
and pragmatic. He did not give any medical treatment to the dog. The dog was not given
food any but lots of water. His method worked. Tricki’s recovery was surprisingly rapid.
Tricki was transformed into a flexible and hard-muscled dog. He was not only out of danger
but was soon handed over to his mistress. The grateful mistress thanked Dr Herriot and called
his feat “a triumph of surgery!” Dr James was clever enough to enjoy the best of both the
worlds. He was tempted to keep Tricki on as a permanent guest. It was a happy period for
Dr Herriot and his friends. He enjoyed eggs for breakfast, and wine and brandy for lunch.

3. Herriot was a duty-bound veterinary doctor. He used to visit Mrs. Pumphrey's house
regularly for the regular check-up of her dog Tricky. Mrs. Pumphrey was a very rich lady
who pampered her dog a lot. Dr Herriot was quite worried about Tricki's health. He knew
that the dog was overfed by his owner. He advised Mrs. Pumphrey to keep him go on diet
and give him a lot of exercises. But she didn't bother. Hence, in order to improve Tricki's
health, he decided to take him away from Mrs. Pumphrey. He told her that Tricki needed a
surgery. He made Tricki cut down his diet and made him do certain exercises which
improved his health. Although he knew Mrs. Pumphrey was very rich and much attached to
the dog, he did not take any advantage of it. He just performed his duties and we should also
stick to the same values in our life. He enjoyed the food and wine sent by Mrs. Pumphrey
but he still didn't keep Tricki at the surgery for long. He knew that Mrs. Pumphrey would be
missing Tricki so he allowed the dog to go home as soon as he was fine.

**

THE THIEF’S STORY


Extract -1
1. Anil was watching a wrestling match when I approached him. He was about 25 - a tall,
lean fellow - and he looked easy-going, kind and simple enough for my purpose. I hadn't
had much luck of late and thought I might be able to get into the young man's confidence.
122
"You look a bit of a wrestler yourself," I said. A little flattery helps in making friends.
i. According to the extract, the young boy was watching the wrestling match because he:
a. had been invited there by the wrestlers.
b. was supposed to meet someone there.
c. was looking for simple people to dupe.
d. loved wrestling and followed it very closely.
ii. 'I hadn't had much luck of late' means that the boy hadn't:
a. ever conned people successfully.
b. been successful in duping people lately.
c. understood the consequences of thievery till date.
d. considered the role of fate in deceiving others.
iii. 'I might be able to get into the young man's confidence.' Choose the option that DOES
NOT display what the statement means.
a. He wanted to win his trust.
b. He wanted him to share his thoughts without caution.
c. He wanted him to feel comfortable revealing more details about himself.
d. He wanted to be able to spend quality time with him.
iv. Anil looked easy-going, kind and simple to the narrator. Which of characteristics
would NOT fit in this description?
a. Compassionate
b. Suave
c. Uncomplicated
d. Carefree
v. Based on the line, "A little flattery helps in making friends", choose the option that
displays the quote closest in meaning.
a. Imitation is the best form of flattery; people generally understand that my comedy is not
intended to hurt anybody.
b. I know imitation is the highest form of flattery but stealing one's identity is totally different.
c. Nothing is so great an example of bad manners as flattery. If you flatter all the company,
you please none; If you flatter only one or two, you offend the rest.
d. One may define flattery as a base companionship which is most advantageous to the
flatterer.
ANSWER
i. c. was looking for simple people to dupe.
ii. b. been successful in duping people lately.
iii. d. He wanted to be able to spend quality time with him.
iv. b. Suave
v. d. One may define flattery as a base companionship which is most advantageous to the
flatterer.

Extract-2
2. I think he knew I made a little money this way, but he did not seem to mind. Anil made
money by fits and starts. He would borrow one week, lend the next. He kept worrying about
his next cheque, but as soon as it arrived, he would go out and celebrate. It seems he wrote for
magazines - a queer way to make a living.
i. Anil made money 'by fits and starts' means that he:
a. deemed it fit to start investing money.
b. started earning money in the recent past.
c. received money intermittently.
123
d. put his money to use frequently.
ii. The information in the extract suggests that Anil could be a:
a. salaried professional
b. freelancer
c. businessman
d. volunteer
iii. If borrow :: lend, then pick the ODD pair from the options below.
a. give:: take
b. lose :: find
c. hop: : skip
d. buy :: sell
iv. The reference to making a little money 'this way' refers to a way that is viewed by
most people as:
a. sensible
b. inappropriate
c. charitable
d. aggressive
v. Based on your understanding of Anil in the extract, choose the option that synchronises
with his thinking.
a. So, what if I don’t have much money? Giving it to that person is important as they
could do with a helping hand.
b. I better learn how to protect my money. I think I am being looted.
c. I earn money with such tremendous effort. Where does it all go?
d. When I become rich, I can begin to help friends then. Right now, I will spend only on my
myself.
ANSWER
i. c. received money intermittently.
ii. b. freelancer
iii. c. hop: : skip
iv. c. charitable
v. a. So, what if I don’t have much money? Giving it to that person is important as they
could do with a helping hand.

Extract -3
3.My spirit rose. But when I took the note, I saw it was still wet from the night's rain. He
knew. But neither his lips nor his eyes showed anything. I smiled at Anil in my most
appealing way. And the smile came by itself, without any effort."

i. What is suggested by the following evidence from the text?


a. The narrator is happier being honest with Anil rather than stealing from him.
b. The narrator is pleased that he is 600 rupees richer.
c. Anil is a funny guy that can make the narrator laugh.
d. The narrator did not want to leave the innocent man alone to be cheated by
someone else.
ii. What promise did Anil make to the Thief Boy?

a. giving him a Good Job


b. giving him money
c. teaching him reading and writing
124
d. giving him a room on rent
iii. The thief boy was grateful to Anil for:
a. putting his efforts to educate him
b. giving him money
c. watching match with him
d. playing with him
iv. How, according to the narrator, would Anil feel upon finding out?
a. angry
b. fear
c. acceptance
d. sad
v. Why does Hari return the money?
a. he wanted to learn to write
b. he was worried about how Anil would feel
c. he thought that he would not get food and shelter anywhere else.
d. none of the above
ANSWER
i. a. The narrator is happier being honest with Anil rather than stealing from him.
ii.b. giving him money
iii.a. putting his efforts to educate him
iv.d. sad
v.a. he wanted to learn to write

Short Answer Type Questions


1.Did Hari like working for Anil? Give reasons in support of your answer.
Ans.Yes, Hari liked working for Anil. He was happy to carry on the chores for him and was
grateful for the education he was receiving. He used to make profit of about a rupee a day as
well, which was a decent amount besides being fed.
2. Hari Singh did not catch the train deliberately while he could catch it easily? Why?
Ans: Hari Singh did not catch the train deliberately because his inner conscience held him
back. He thought that Anil would feel sad not for loss of money but for the loss of trust he had
reposed in him. Moreover, he did not want to lose the chance of learning. His inner self was
transformed.
3.Had Anil really forgiven Hari Singh? Support your answer with evidence
Ans. Yes, Anil had forgiven Hari Singh. It is evident because Anil handed over to Hari a fifty
rupee note as soon as Hari woke up. Though he knew that Hari had robbed the money at first
but his subsequent actions gave him hope of change in Hari’s character

Long Answer Type Questions (6 Marks)

Q1. Money can’t make a man as much as education can. Elucidate the statement.
Answer: The statement stands true in almost all the aspects of life. Money may buy us all the
luxuries and fulfil our needs but it cannot buy us knowledge, civilized thinking, skills and
abilities to achieve our dreams. Education lays the platform for all to act upon our goals
according to our abilities. Education enables us to keep up with the fast- moving world. It
opens the door to opportunities we do not know even exist. Money, on the other hand, can
assist us to a certain level. It can buy us a plan but education gives us the knowledge of its
execution.

125
Hari Singh befriends the struggling writer, author, ‘Anil’, in the plans of executing a robbery.
Anil; the author is a good fellow and wants to teach Hari Singh how to read and write.
Gradually, he develops trust in Hari Singh. But Hari Singh has been waiting for the right
moment to steal the valuables of Anil. After stealing money from Anil, Hari Singh tries to go
away forever. But his conscience stops him from doing so. He returns to Anil because he wants
to live a life of respect. This shows that Hari Singh prioritized the chance of being literate over
a few hundred rupees, we must understand that education can help us to achieve whatever we
desire.
Q2). Hari Singh didn’t board the express and returned to Anil. Why did he return? On
what values does this incident put light on?
Answer: Hari Singh was a thief and he had stolen Anil’s money. After the theft, he realised
that he had robbed not only Anil but also himself of the chance of being literate and having a
bright future. His conscience pricked him to think what all he could have got had he not done
this. It was difficult for him to rob Anil but it was tougher for him not to back. He realised that
he could not make tea, buy daily supplies and learn how to read and write then. His inner self
did not agree to bypass this and forced him to return.
Hari’s return to Anil shows that despite indulging in criminal acts, he still had a practical and
positive attitude towards life. It is the awakening of Hari’s conscience and Anil’s love and care
that reformed Hari’s character. It teaches us that love alone can change a person. Anil’s
understanding nature and care changed Hari’s thinking to mend his ways for good.
Q3). Love is a powerful force that can transform a criminal into a responsible citizen.
Justify the statement from your study of “The Thief’s Story”.
Love is a potent force that can transform even a habitual criminal into a responsible and law-
abiding citizen. It can make ineffective even the most venomous elements. “The Thief’s Story”
is a salient example of an experienced and ‘fairly successful thief’ turned into sensible person
driven to lead life as an educated man. Hari Singh knows only the unethical and antisocial
ways of surviving. He very successfully and efficiently robs Anil. Although he could
conveniently board the train to Lucknow and decamp the loot, he cannot. The love, kindness
and sympathy that Anil had shown earlier chains him and forces him to return to his
compassionate employer. The trust Anil puts in Hari Singh is enough to create a sense of guilt
in him. Thus, love comes out to be a winner in the long run and Hari Singh turn to the path of
honesty, sincerity and loyalty.

THE MIDNIGHT VISITOR


Extract 1
1. Max was slender, a little less than tall, with features that suggested slightly the crafty,
pointed countenance of a fox. There was about him — aside from the gun — nothing
especially menacing. “The report,” he murmured. “The report that is being brought to
you tonight concerning some new missiles. I thought I would take it from you. It will be
safer in my hands than in yours.” Ausable moved to an armchair and sat down heavily.
“I’m going to raise the devil with the management this time, and you can bet on it,” he
said grimly. “This is the second time in month that a somebody has got into my room
through that nuisance of a balcony!”
i Owning a gun gave a menacing impression to Max’s character because
a) a gun is only used by police officials.
126
b) he was carrying it without any official license.
c) a gun has a threatening implication.
d) the gun was particularly huge.
ii Choose the option that displays the correct analogy.
a) crafty : fox :: menacing : gun
b) report : missile :: management : devil
c) crafty : fox :: management : balcony
d) armchair : grim :: balcony : nuisance
iii Choose the part of the house through which Ausable thought Maxwell had entered his
room.

a) Option 1
b) Option 2
c) Option 3
d) Option 4

iv. Why did Max enter Ausables room?


v. Which word replaces ‘sat down heavily’ in the sentence—’Ausable moved to an
armchair and sat down heavily’?
a) dumped.
b) flumped.
c) pumped.
d) jumped.
ANSWER
i. c) a gun has a threatening implication.
ii. a) crafty : fox :: menacing : gun
iii. b) Option 2
iv. Max entered Ausable’s hotel room to get an important report by force
v. b) flumped

Extract 2
Ausable did not fit any description of a secret agent Fowler had ever read. Following him
down the musty corridor of the gloomy French hotel where Ausable had a room, Fowler
felt let down. It was a small room, on the sixth and top floor, and scarcely a setting for a
romantic adventure. Ausable was, for one thing, fat. Very fat. And then there was his
accent. Though he spoke French and German passably, he had never altogether lost the
American accent he had brought to Paris from Boston twenty years ago. “You are
disappointed,” Ausable said wheezily over his shoulder.
i “Following him down the musty corridor of the gloomy French hotel where Ausable
had a room…” Choose the correct option to replace the underlined words.
a) expansive; abnormal
b) lighted; formal
c) damp; dingy
127
d) uncanny; smart
ii What is special about Ausable’s language?
iii It can be inferred from the extract that a secret agent is expected to be
1) fit. 2) compassionate.
3) apologetic 4) suave. 5) violent.
a) Option 1, 3 & 5
b) Option 1 & 4
c) Option 2 ,4 & 5
d) Option 2 & 3
iv. The reason Fowler was disappointed by Ausable when he met him was that Ausable
a) lived in a shady French hotel.
b) was related to the cops.
c) didn’t have the personality of a spy.
d) had a very strange accent.
v. Choose the option that lists the INCORRECT statements about the given passage.
a. Fowler had no expectations at all from Ausable.
b. Fowler was elated when he met Ausable.
c. Ausable was living in France as a spy.
d. Ausable had acquired the French accent over twenty years ago.
ANSWER
i. c) damp; dingy
ii. Ausable spoke French and German passably with an American accent.
iii. b) Option 1 & 4
iv. c) didn’t have the personality of a spy.
v. d. Ausable had acquired the French accent over twenty years ago.

Extract 3
"You are disappointed," Ausable said wheezily over his shoulder. "You were told that I
was a secret agent, a spy, dealing in espionage and danger. You wished to meet me
because you are a writer, young and romantic. You envisioned mysterious figures in the
night, the crack of pistols, drugs in the wine." "Instead, you have spent a dull evening in
a French music hall with a sloppy fat man who, instead of having messages slipped into
his hand by dark-eyed beauties, gets only a prosaic telephone call making an
appointment in his room. You have been bored!" The fat chuckled to himself as he
unlocked the door of his room and stood aside to let his frustrated guest enter.
i "You were told that I was a secret agent, a spy, dealing in espionage and danger."
Who is 'you' referred to here?
a. Ausable
b. Fowler
c. Max
d. Henry
ii. What is the profession of 'you' in the given extract?
a. He was a spy.
b. He was a waiter.
c. He was a writer.
d. He was a fat and hopeless man with no business.
iii. How did the man envision his meeting with Ausable?
a. The man was actually scared of meeting a spy and smelt risks associated with it.
b. The man had expected of Ausable to be a down to earth man.
128
c. The man anticipated that Ausable must be a fat and lethargic person.
d. The man envisioned that he would be meeting an agent dealing with danger, crime scenes
and drugs.
iv. Which one of the characteristics given below doesn't describe Ausable?
1. Muscular 2. Clever 3. Meticulous
4. Linguist 5. Extravagant 6. Poorly dressed 7. Hefty
a. Options (1), (3) and (5)
b. Options (2), (3) and (5)
c. Options (1), (6) and (7)
d. Options (4), (3) and (5)
v. Select the option that makes the correct use of 'chuckled', as used in the extract. to fill
in the blank space.
a. The burglars ---------------------- at a thought while breaking into a house.
b. The couple ----------------------- at the child's efforts to walk.
c. The priest ---------------------- while addressing the evening prayer.
d. The judge ----------------------- while pronouncing a verdict.

ANSWER
i. b. Fowler
ii. c. He was a writer.
iii. d. The man envisioned that he would be meeting an agent dealing with danger,
crime scenes and drugs.
iv. b. Options (2), (3) and (5)
v. b. The couple chuckled at the child's efforts to walk.

Short Answer Type Questions


1.Why was Fowler disappointed after meeting Ausable?
Ans: - Fowler felt disappointed with Ausable because he did not look like a secret agent of
his imagination. He found him quiet boring. In Fowler’s concept secret agents typically are
physically fit, well-versed in many languages, can handle different firearms and are very
witty and humorous. In contrast, Ausable is not fit, is very fat and has a prominent accent.
He could speak French and German passably. Moreover, he lives in a small room in the
musty corridor of gloomy French hotel. It was on the sixth and top floor and it was scarcely
the setting for a romantic adventure.
2. How does Ausable manage to make believe that there is a balcony attached to his
room? Look back at his detailed description of it? What makes it a convincing story?
Ans: - When Ausable sees Max in his room, demanding the report at gun point, he creates
an impression of frustration about anybody entering his room via a balcony under his
window. By connecting it to a door visible in the room, he creates an authentic-sounding
explanation for the balcony to exist. Max believes the story and tries to use the same
balcony for his escape on hearing about the police.
3.Why did Max’s face turn black with anger? What did he want Ausable to do?
Ans. Max’s face turned black with anger because he felt that his plan of extracting
the important document from Ausable was in danger of failing due to the knocking on the
door, which Ausable identified as the police. He wanted Ausable to send the police away.

Long Answer Type Questions (6 Marks)

129
Q1). Presence of mind and intelligence is more powerful than a gun.’ How far is it true
in case of Ausable, the secret agent?

Ans. A good spy need not be handsome, attractive and smart. Physical strength can be
additional advantage but certainly not the essential one. Secret agents have to face critical
situations at every step. One night, Ausable found Max in his room. There was an automatic
pistol in his hand. He had come there to take the report from Ausable. Max was also a secret
agent of another organisation. Ausable was not the least afraid of Max’s presence. He thought
of a clever trick. He convinced Max about a balcony outside the window. Just then there was
a loud knock at the door. Ausable told Max that it was police to provide him security. Max
had no time to think. Max rushed towards the window and dropped to the non-existent balcony
and met his tragic end. Ausable did not use physical strength. He won over the critical situation
by his sheer presence of mind.

Q2). Describe how Max was duped by Ausable?

Ausable created a detailed description of how his room was a part of a bigger apartment and
how the next had a direct connection with a balcony. His statement that somebody else had
also broken into his office through that balcony made it a convincing story. Ausable did this
to fool Max into thinking that a balcony existed just outside the window, as he knew that soon
the waiter bringing the drinks he had ordered would be knocking at the door, which may
prompt Max to jump into the imaginary balcony to hide and when the waiter knocked at the
door, Ausable again be fooled Max by saying that it was the police who wanted to come inside
to provide him protection. Max wanted to run away to avoid the police and jumps to his death
into that non-existent balcony.

Q3). Ausable is a successful secret agent. What qualities can be attributed to him? Give
some examples.

Ausable is unusual kind of secret agent. Generally secret agents who are described in detective
books are very smart and gun-friendly. Wine and dark-eyed beauties are associated with them.
However, Ausable doesn't present such a romantic picture. He is very fat and sloppy. He lives
on the 6 floor. Fowler, the writer is disappointed after meeting him. They say that appearances
th

can be deceptive. This is quite true in case of Ausable. Physically, he may be fat but mentally
he is agile and quick-witted. He knows all the tricks of the trade. He convinces even the crafty
Max in believing that a balcony existed outside the window. This is a well-thought-out trap.
Max unwittingly Falls in to it he then cooks up a story of the police. Once again success in
convincing Max. He tells Max that he has sought police protection as he has some reports on
new missiles with him. In this way, he gets rid of his rival and enemy who jumps to his death.

A QUESTION OF TRUST
1] Extract Based Questions (RTC)
(A) How foolish people are when they own valuable things, Horace thought. A magazine
article had described this house, giving a plan of all the rooms and a picture of this room. The
writer had even mentioned that the painting hid a safe! But Horace found that the flowers were
hindering him in his work. He buried his face in his handkerchief. Then he heard a voice say
from the doorway, “What is it? A cold or hay fever?” Before he could think, Horace said, “Hay
130
fever,” and found himself sneezing again. The voice went on, “You can cure it with a special
treatment, you know, if you find out just what plant gives you the disease. I think you’d better
see a doctor, if you’re serious about your work. I heard you from the top of the house just
now.”

1) According to the extract, Horace was sneezing due to a


a) sudden allergy
b) prior infection
c) long-standing disease
d) nasal pain

2) Which magazine could have published the article mentioned in the given extract?
a) Paws & Claws
b) Outdoors & Landscaping
c) Interiors
d) The Investor

3) Choose the image that correctly describes the location of the safe, based on the given
extract.

a) Option (1) b) Option (2) c) Option (3) d) Option (4)

4) In the line – Before he could think, Horace said, “Hay fever,” –the response was
a) rehearsed
b) spontaneous and natural.
c) hesitant.
d) rapid and hostile.

5) The line from the extract that can be considered an example of sarcasm is
a) How foolish people are when they own valuable things.
b) Horace found that the flowers were hindering him in his work.
c) You can cure it with a special treatment.
d) I think you’d better see a doctor, if you’re serious about your work.

(B) But he never got the chance to begin his plan. By noon a policeman had arrested him for
the jewel robbery at Shotover Grange. His fingerprints, for he had opened the safe without
gloves, were all over the room, and no one believed him when he said that the wife of the
owner of the house had asked him to open the safe for her. The wife herself, a gray-haired,
sharp-tongued woman of sixty, said that the story was nonsense. Horace is now the assistant
librarian in the prison. He often thinks of the charming, clever young lady who was in the same
profession as he was, and who tricked him. He gets very angry when anyone talks about
‘honour among thieves’.
131
1) The plan devised by Horace was to
a) tell about the safe to owner of the house.
b) blackmail the wife of the owner later.
c) look for another safe for stealing books.
d) tell police the truth before the lady could deceive him.

2) Based on the extract, choose what you think are the main feelings Horace probably
has, for the young lady who tricked him, when he thinks about her?
a) anger and blame
b) admiration and respect
c) respect and gratitude
d) anger and vengeance

3) Choose the option that lists evidence that the police might have used against Horace,
to arrest him.

a) Option 1 b) Option 2 c) Option 3 d) Option 4

4. Given below are four situations in Mr. Verma’s house. Choose the situation that
depicts Mr. Verma being sharp-tongued with his family members.
a) Mr. Verma’s wife shares a life problem with him and he advices her honestly.
b) Mr. Verma’s daughter scores poor marks in his exams and Mr. Verma remains silent.
c) Mr. Verma’s son cooks food for the first time and he is criticized by his father immediately.
d) Mr. Verma’s sister buys a new car to surprise him and he is overjoyed with the news.

5. Honour among thieves is an example of a/n


a) idiom
b) slogan
c) proverb
d) lyric

(C) The voice went on, “You can cure it with a special treatment, you know, if you find out
just what plant gives you the disease. I think you’d better see a doctor, if you’re serious about
your work.” I heard you from the top of the house just now”.

1)Whose voice is referred to in the above extract?


2) What disease was the voice referring to?
3) Find a word from the passage that means the same as ‘ailment’.
4) What part Of speech is the word ‘it’ in the extract?
5) What is the House mentioned?

132
D] Answer the following questions in 40-50 words.

1. Who is the real culprit in the story, the young lady in red or Horace Danby?
Comment.

2. Do you think that the young woman in red proved far smarter than Horace
Danby? Give reasons.

3. When and why did the woman in red say, “Society must be protected from men
like you”? Doesn’t it sound ironical?

E] Answer the following questions in 100-120 words.

1. Horace Danby represents such people who adopt the wrong ways to fulfill their wishes.
What values would you like such people to imbibe to reform themselves? Write in about
100-120 words.

2. Horace stated that he robbed only those who had a lot of money. Was he really a threat
to society? Did he lack the qualities of a good citizen? Discuss the values he needs to
imbibe to lead a path of righteousness in 100-120 words.

3 ‘The lady in the red was a more professional thief than Horace Danby’. Give a reasoned
answer.

ANSWERS (RTC)
(A)
1. (b) prior infection 2. (c) Interiors 3. (c) Option (3) 4. (b) spontaneous and natural.
5. (d) I think you’d better see a doctor, if you’re serious about your work.

(B)

1. (c) look for another safe for stealing books. 2. (b) admiration and respect
3. (d) Option 4
4. (c) Mr. Verma’s son cooks food for the first time and he is criticized by his father
immediately.
5. (a) idiom

(C)
1. The voice referred to in the above extract is of the young lady who pretended to be a
member of the household.
2. The disease the voice was referring to was hay fever, with which Horace Danby was
afflicted.
3. The word is ‘disease’.
4. ‘It’ is a pronoun.
5. Shotover Grange.
(D) ANSWERS OF SHORT QUESTIONS ( 40-50 WORDS)
133
1. Answer: Law of any land works on evidence. Horace Danby opened the safe without
wearing his gloves. He didn’t think it necessary to wear them as he thought he was working
for ‘the lady of the house’. So the police matched his fingerprints and he was arrested. The
lady in the red was actually the real culprit. She made Horace open the safe for her. She was
also a thief and decamped with the jewels. But in the eyes of the law, Horace was the real
culprit because the evidence was against him.
2. Answer: Definitely, the young woman in red proved far smarter than Horace Danby. She played
her part of being the lady of the house with complete perfection. She was an example of self-
confidence. She didn’t show any hesitation. She spoke and acted with an air of complete confidence
and authority. Poor Horace became a puppet who couldn’t take himself out of her spell. She
threatened to call the police and made him open the safe, while Horace was later caught and arrested.
She decamped with the jewels.
3. Answer: Horace assured the woman in red that he would not hurt her. She must forget she ever
met him. It made her angry. She couldn’t let him go, otherwise, he would rob someone else. Society
must be protected from people like him. It sounds ironic because she herself was a thief and had
come for the same purpose for which Horace was there in the house.

(E) ANSWER OF LONG QUESTIONS (100-120 W0RDS)


1. Answer: Horace Danby was a successful businessman. He was about fifty years respectable
citizen but had a habit that led him to do robberies. Danby loved rare, expensive books and to get
them he used to rob a safe every year. Every year he planned carefully just to get what he wanted.
Danby adopted wrong ways to fulfill his wishes and hence, later got trapped in a plot by a thief like
him. People should imbibe good values. They should learn that hard work is the only key to success
One should work hard to fulfill his desires. I think such people should imbibe the values of honesty,
responsibility, hard work, dedication, self-confidence in order to reform themselves.
2. Answer: I don’t think Horace was a threat to society. He never threatened anyone and rather stole
for a very good reason. He used to rob a safe every year to pursue his hobby of getting rare and
expensive books to read. He was not a professional thief. Moreover, he never harmed anyone. He
was considered a good and honest citizen by everyone. He was about fifty years old and unmarried.
He had all the qualities of a good citizen. He was not a criminal or a typical thief. He did so only to
fulfill his need. He was an honest and good man. He believed in the lady’s story and gave away all
the jewels to her. If he had been a criminal, he would have killed the lady or would have never handed
the jewels to her.
3. Answer: Horace Danby was going to rob the safe at Shotover Grange. He had cut the wires of the
burglar alarm. But the flowers on the table made a tickle in his nose and he was sneezing repeatedly.
Just then a young lady dressed in red came in. She spoke friendly to Danby but her sound was firm.
She said that she was the owner’s wife. She told him that she had come there to take the jewels from
the safe. She had to wear them that night at a party. She made an excuse that she had forgotten the
number to open the safe. She told the thief that she would let him go if he opened the safe for her.
Danby was taken in. He opened the safe without gloves. He gave all the jewels to the young lady.
She went away safely with the jewels but Danby was arrested for the jewels robbery and sent to
prison.

LESSON 5. FOOTPRINTS WITHOUT FEET


1. Multiple Choice Questions based on an extract.

A. As she and her husband turned away in terror, the extraordinary chair pushed them both
out of the room and then appeared to slam and lock the door after them.

134
Mrs Hall almost fell down the stairs in hysterics. She was convinced that the room was
haunted by spirits, and that the stranger had somehow
caused these to enter into her furniture.
“My poor mother used to sit in that chair,” she moaned! To think it should rise up against
me now!
The feeling among the neighbours was that the trouble was caused by
‘’Witch craft”
i. Mrs Hall felt that the room was haunted by spirits because
a) she could see evil spirits.
b) she heard strange noise.
c) uncanny things happened there.
d) the door slammed shut.
ii. Pick the option that best describes how Mrs Hall must be feeling at the
moment described in the extract.
a) stunned and furious
b) shocked and outraged
c) outraged and nervous
d) stunned and agitated
iii) Pick the sentence that brings out the meaning of ‘hysterics’ as used in the
extract.
a) My friend and I were in splits when we saw the clown’s antics.
b) I don’t know why I suddenly felt worried about flying
home.
c) The sight of blood put the old man in a frenzy.
d) The people who had witnessed the accident were spellbound.
iv) Pick the option that displays a cause -> effect relationship.
a) pushed and locked out -> hysterical
b) rising of the chair -> moaning
c) troubled neighbours -> witchcraft
d) stranger -> haunted spirits
v) The neighbours thought it was ‘witchcraft’. This tells us that neighbours were
a) suspicious.
b) superstitious.
c) nervous wrecks.
d) gossip-mongers.
vi. Pick the option that includes the correct matches of Column A with Column B.

Column A Column B

I. The stranger was i) eccentric, lonely and callous

II. He had escaped ii) Eccentric, callous and short-


tempered

135
III. He had an uncommon iii) From Iping to London
appearance

iv) as he wore bandages round his


forehead

a) I-ii; II-iv; III-iii


b) I-i; II-iii; III-iv
c) I-iii; II-ii; III-i
d) I-ii; II-iii; III-iv

B.The two boys started in surprise at the fresh muddy imprints of a pair of
bare feet. What was a barefooted man doing on the steps of a house in the
middle of London? And where was the man?
As they gazed, a remarkable sight met their eyes. A fresh footmark
appeared from nowhere!
Further footprints followed, one after another, descending the steps and
progressing down the street. The boys followed, fascinated, until the
muddy impressions became fainter and fainter, and at last disappeared
altogether.

i. Why were the boys surprised to see a barefooted man in London?


The boys were surprised as

i) it was an unusual sight to see someone this way.


j) everybody in London moved around in shoes.
k) it was pretty cold to move around bare feet.
l) only a person who is homeless and wandering does so.

ii. Pick out the option that is NOT related to ‘started’ as used in the extract

a) option (i)
b) option (ii)
c) option (iii)
d) option (iv)

iii. Pick the option that best describes how the boys are feeling based on the
extract.
a) enchanted, curious, puzzled
136
b) captivated, curious, puzzled
c) repulsed, curious, captivated
d) enchanted, repulsed, curious

iv. The boys felt that the footprints were

a) seen due to some magic trick.


b) a figment of imagination.
c) of a man who was invisible.
d) those of a mysterious man.

v. Pick the option that best matches synonyms of the word ‘gazed’.
a) i) gaped ii) gawked
b) i) admired ii) disbelieved
c) i) overlooked ii) stared
d) i) surveyed ii) overlooked

vi. Pick the option that lists the correct direction of the footprints on the stairs, as
noticed by the boys.

a) option (i)
b) option (ii)
c) option (iii)
d) option (iv)

ANSWERS FOR MCQ


(A.)
1. c) uncanny things happened there.
2. d) stunned and agitated
3. c) The sight of blood put the old man in a frenzy.
4. a) pushed and locked out -> hysterical
5. b) superstitious
6. d) I-ii; II-iii; III-iv

(B)
1. c) it was pretty cold to move around bare feet.
2. d) option (iv)
3. b) captivated, curious, puzzled
4 d) those of a mysterious man.
5. a) i) gaped ii) gawked
6. b) option (ii)

(C) They naturally gave chase. In the end he was able to escape only by quickly taking off his
newly found clothes. So once more he found himself invisible but naked in the chill January
air.
(a) Who are ‘they’ and ‘he’ referred to in the above extract?
137
(b) Why has the author used the phrase ‘once more’?
(c) Find a word from the extract that means the same as ‘run away’.
(d) What is the opposite of ‘invisible’?

(e) Choose the Most appropriate word for the picture.


i) Following ii) Ascending iii) Descending iv) Chasing

Answer:
(a) ‘They’ are the shop assistants and ‘he’ is the invisible scientist, Griffin.
(b) The author has used the phrase ‘once more’ because Griffin was naked before he had
entered the large store, from where he had taken the ‘newly found’ clothes.
(c) The word is ‘escape’.
(d) Its opposite is ‘visible’. (e) Chasing.
2. Answer the following in 20 to 30 words:

Q1. How was Griffin a lawless man?


Ans. Griffin was lawless because he had committed many anti-social deeds for which he never
felt guilty. He set fire to his landlord’s house and ran away, he stole ‘ foodstuffs and clothes
from a London store, stole other goods from a theatrical company and also stole money from
a clergyman’s house to pay his bills.

Q2. What did Griffin do in the shop of a theatrical company?


Ans. The second time Griffin tried the stock of a theatrical company in the hope of finding not
only clothes but also something like side whiskers, glasses, false nose, bandages, hat etc that
would hide the empty space above his shoulders.

Q3. How did the visible man become invisible? What did he do then?
Ans. The invisible man got angry with Mrs Hall for asking an explanation for the mysterious
happenings and threw off all he was wearing on his head so that he became a man without a
head. Soon the constable Jaffer’s arrived to arrest him, but the invisible man threw off all his
clothes to become invisible and, in the ensuing scuffle, knocked Jaffer’s unconscious and
escaped.
3.Answer the following questions in 100 to 120 words
1. “Discoveries of science can be used for welfare as well as for destruction.” How is this
applicable to Griffin’s scientific discovery? How did Griffin bring a bad name to his invention?
How was his character?
Ans. A true scientist works for the good of humanity. He wants to make man’s life easier,
more comfortable and enjoyable. He does not misuse his discoveries for personal gains or
selfish ends. But Griffin, though a brilliant scientist, misuses his discovery. By his
experiments, he has been able to make his body transparent and invisible.
He uses this discovery to puzzle other people, enters stores and shops unseen, robs people of
their money or things and escapes. He sets fire to the house of his landlord who tried to eject

138
him. He steals food, sweets and wine. Griffin brought a bad name to science by misusing his
invention. He was a selfish man with a criminal bent of mind.
2.Griffin was a brilliant scientist but not a true one. What made him a bad scientist? What does
it show about his character? How is the value of Welfare for all’ important in the life of a
scientist?
Ans. Griffin was a brilliant scientist but not a true one. He discovered how human body could
be made transparent. It was an amazing discovery that could be used the welfare of the society.
But Griffin misused it. He used it to puzzle others. He disturbed the peace of others. He robbed
the innocent people. He took revenge upon his landlady.
It shows he was not a man of good character. He lacked the value of kindness, cooperation and
humility’. He was an eccentrics dishonest and lawless scientist who brought bad name to
science. He did not understand the value of welfare for all. If he had used his invention for a
good cause, he would have become immortal and remembered forever.
3.Are there forces around us that are invisible, for example, magnetism? Are there aspects of
matter that are ‘invisible’ or not visible to the naked eye? What would the world be like if you
could see such forces or such aspects of matter?

Ans.Yes, there are forces around us that are invisible such as magnetism, gravitation, air and
electric current. These aspects of matter are not visible to the naked eye because they cannot
be perceived by the sense of sight but these forces can definitely be felt.
The world would have been more adventurous and interesting because we have seen such
forces only on paper or in books and never with the eye as of now. The understanding of the
components of matter would have been easier and there would probably be no gap and empty
spaces in the world. Above all, Nature would have never been as mysterious and mighty as it
is now, if we could see some hidden aspects of nature with a naked eye.

THE MAKING OF A SCIENTIST


(A) RTC Extract Based MCQ (a),(b)+(c). (5 marks)
(a) “I didn’t get any real results,” he said. “But I went ahead and showed that I had tried
the experiment. This time I won.” The next year his science fair project was testing the
theory that viceroy butterflies copy monarchs. The theory was that viceroys look like
monarchs because monarchs don’t taste good to birds. Viceroys, on the other hand, do
taste good to birds. So, the more they look like monarchs, the less likely they are to
become a bird’s dinner. Ebright’s project was to see whether, in fact, birds would eat
monarchs. He found that a starling would not eat ordinary bird food. It would eat all the
monarchs it could get.

1) Choose the option listing Ebright’s qualities as depicted by the above extract.
1. persevering
2. visionary
3. determined
4. liberal
5. conceited
Choose the correct option among the following:
a) 1, 2 b) 3, 5 c) 1, 3 d) 4, 5
Answer: c) 1, 3

139
2. According to the dictionary, ‘fair’ as a noun, shows the following meanings. Choose
the option that lists the meaning similar to the usage to that in the extract.

a) A gathering of stalls and amusements for public entertainment.


b) A competitive exhibition showcasing products or ideas.
c) A periodic gathering for the sale of goods.
d) An annual exhibition of livestock, agricultural products, etc., held by a town, county, or
state.
Answer: b) A competitive exhibition showcasing products or ideas.

3. Choose the option that is true for the two statements given about the information in
the extract.

Statement 1- Starling feeds on viceroys.


Statement 2- Starling does not eat seeds and insects.

a) Both statements are clearly mentioned in the extract.


b) Statement 1 cannot be clearly inferred from the text and statement 2 is true.
c) Statement 1 is false and statement 2 cannot be clearly inferred from the extract.
d) Bothe statements need to be inferred from the given extract.
Answer: c) Statement 1 is false and statement 2 cannot be clearly inferred from the
extract.

4. Choose the statements that are TRUE for the given extract contextually.
1. Ebright didn’t get any results for the experiment he conducted on butterflies.
2. Monarchs tasted awfully to the birds.
3. Ebright wanted to explore the possibility of monarchs getting eaten by birds.
4. He wanted to prove that viceroys are lookalikes of monarchs.
Choose the correct option among the following:
a) 1, 2 b) 2, 3 c) 1, 3 d) 2, 4
Answer: b) 2, 3

5. Four friends bring their pets to a pet show. Choose the option that mentions the friend
with a starling as a pet.

Friend 1 has a turtle named Missy.

Friend 2 has a dragonfly named Majesty.

Friend 3 has a rabbit named Molly.

Friend 4 has a bird named Mitch.

140
a) Friend 1 b) Friend 2 c) Friend 3 d) Friend 4
Answer: d) Friend 4

(b) When he saw those photos, Ebright didn’t shout, ‘Eureka!’ or even, ‘I’ve got it!’ But
he believed that, along with his findings about insect hormones, the photos gave him the
answer to one of biology’s puzzles: how the cell can ‘read’ the blueprint of its DNA. DNA
is the substance in the nucleus of a cell that controls heredity. It determines the form and
function of the cell. Thus, DNA is the blueprint for life. Ebright and his college room-
mate, James R. Wong, worked all that night drawing pictures and constructing plastic
models of molecules to show how it could happen. Together they later wrote the paper
that explained the theory.

1. Choose the option that shows the picture of the type of task Ebright and Wong were
engaged in, as per the extract.
a) Option (1)
b) Option (2)
c) Option (3)
d) Option (4)
Answer: c) Option (3)
2. Ebright was perhaps expected to shout ‘Eureka!’ because he had
a) realised that he needed a partner to work with to finalise his findings.
b) discovered something new and ‘Eureka!’ was a cry to announce it.
c) worked hard and was relieved at nearing the end of his project.
d) given shape to the teachings of his teachers by choosing this field of science.
Answer: b) discovered something new and ‘Eureka!’ was a cry to announce it.
3. “Thus, DNA is the blueprint for life”, is another way of saying that the DNA contains
a genetic __________.
a) experiment
b) ultimatum
c) takeaway
d) plan
Answer: d) plan

4. Four newspapers published a headline about Ebright and Wong. Choose the option
that published a factually correct headline, as per the extract.
Newspaper 1 Newspaper 2 Newspaper 3 Newspape
r4

WONG DENIES EBRIGHT WONG AND EBRIGHT


CONTRIBUTIN COLLABORATE EBRIGHT AND
G TO S WITH ROOM- EXAGGERAT WONG’S
EBRIGHT’S MATE WONG E THEIR THEORY
THEORY THEORYDEF PROVED
Y LOGIC WRONG

a) Newspaper 1
b) Newspaper 2
141
c) Newspaper 3
d) Newspaper 4
Answer: b) Newspaper 2

5. Compound words are those words which are formed by joining two separate words to
create a new word with an entirely different meaning. Choose the option that lists the
compound words from the above extract.
1. determines
2. blueprint
3. nucleus
4. heredity
5. room-mate
Choose the correct option among the following:
a) 1, 3 b) 2, 4 c) 1, 4 d) 2, 5
Answer: d) 2, 5
(c) From the first he had a driving curiosity along with a bright mind. He also had a
mother who encouraged his interest in learning. She took him on trips, bought him
telescope, microscope, cameras, mounting materials, and other equipment and helped
him in many other ways.
Questions:
(a) What was the role of his mother in his life?
(b) How did she perform her duties?
(c) What does the word ‘bright’ mean here?
(d) What did Ebright desire?

(e) It refers to……………………………….


Answer:
(a) She observed her child, identified his interests and inclinations and channelized his energies
in right direction.
(b) To help him, she bought him books, took him out on trips, also bought him a telescope, a
microscope, cameras, mounting materials and other equipments.
(c) ‘curious and intelligent’.
(d) Ebright had a driving curiousity.
(e)Bright mind
(B) Short Questions - Answers. (3 marks each)
Q.1. What rare achievement did Richard manage at the age of 22?
Answer. Richard had a rare honour at the age of 22. He wrote an article with his friend about
its theory of how cells work. The article was published in the scientific journal ‘Proceedings
of the National Academy of Science. ‘No one had this achievement at such a young age before
him.
Q.2. How did ‘The Travels of Monarch X’ prove a turning point in Richard’s life?
Answer. Richard was bored with collecting butterflies. At this time his mother got the book
‘The Travels of Monarch X’ for him. After reading the book, he studied the migration of
butterflies which told how monarch butterflies migrated to Central America and it opened the
world of science to him.
Q.3. Why do viceroy butterflies copy the monarch butterflies?

142
Answer. Birds eat viceroy butterflies because they taste good to them, whereas monarch
butterflies do not taste good to the birds. So, the viceroys try to copy the monarchs to protect
themselves from the birds. Ebright has placed this project and it was the first in Zoology
division.
(C) Long Questions - Answers (6 marks each)
Q.1. Richard Ebright displayed a well-rounded personality. Do you agree? Elucidate in
the context of the given text.
Answer: Richard’s genius was obvious by the time he was in his second grade. He managed
to collect all twenty-five species of butterflies around his hometown and classify them. He also
loved to collect coins, fossils and rocks. Science was not his only passion. He was an active
member of his school’s oratory club and model United Nations club and was an effective
debater and a public speaker. He loved photography as well. He was an enthusiastic canoeist
and an all-around outdoors person. Learning was easy for him. So he found it simple to devote
time and energy to many other interests. He became a champion in whatever he did. He
believed in the spirit of competition to win. But, he did not wish to defeat others just to win.
He wanted to win to do his best. Thus, he displayed a well-rounded personality.
Q.2. Although Richard does not win anything at the science fair but it was a stepping
stone for his success. With reference to the story ‘The Making of a Scientist’ of the above
statement, give your comments whether competitions are for winning sake or to give your
best at work.
Answer: It is true, no one can deny the fact that every person wants to be a winner. Each has
basic wish to reach at the top. For that many competitions are organised at various levels. But
still we must accept that all cannot be winners. Participation is more important than winning.
The participant should work hard to reach their level best. Failures should not make us
disheartened and best way to overcome failure is to learn through our mistakes. We cannot
deny that experimentation and learning are stepping stones to our success. So we should try to
give our best.
Q.3. Besides curiosity a number of other values are required to become a successful
scientist. Explain with reference to the chapter, ‘The Making of a Scientist’.
Answer: From very young age Richard Ebright was competitive and put in extra effort
with curiosity for the right reason to win. But his mother was always very dedicated and
made his spirits rise high. He did not lose heart even after losing when he was in seventh
grade. To him people around were very encouraging. His mother’s encouragement was
really an eye-opener. She took him on trips, bought him telescopes, microscopes, cameras,
mounting materials and other equipment. Thus constant support of each other opened a
new world. This helps us to conclude that hard work, parental guidance and keen
observation are the qualities which help one to excel.

CHAPTER 7- THE NECKLACE

Q3 Multiple Choice Questions based on an extract.


“She suffered incessantly, feeling herself born for all delicacies and luxuries. She suffered from the
poverty of her apartment, the shabby walls and the worn chairs. All these things tortured and
angered her.
When she seated herself for dinner opposite her husband who uncovered the tureen with a delighted
air, saying, “Oh! the good potpie! I know nothing better than that...,” she would think of elegant
dinners of shining silver; she thought of the exquisite food served in marvellous dishes. She had
neither frocks nor jewels, nothing. And she loved only those things.

143
She had a rich friend, a schoolmate at the convent, who she did not like to visit- she suffered so
much when she returned. She wept for whole days from despair and disappointment.”
(A)
i Choose the option that list the set of statements that are NOT TRUE according to the given
extract.
1. Matilda was very pleased with her life.
2. Matilda envied her friend for being well-off.
3. M Loisel didn’t appreciate what Matilda cooked.
4. Matilda despised the fact that she lived a life of poverty.
5. Matilda never felt troubled, though she desired a luxurious life.
6. Matilda thought of grand dinners and silverware sitting at the dinner table.
7. Matilda felt depressed after visiting her friend.
a) 1, 3, 6
b) 3, 5, 7
c) 1, 3, 5
d) 2, 4, 7
ii Which word does ‘delicacies’ NOT correspond to?
a) etherealness
b) elegance
c) exquisiteness
d) robustness

iii Choose the answer that lists the correct option of what a ‘tureen’ is?

a) Option (i)
b) Option (ii)
c) Option (iii)
d) Option (iv)
iv Choose the characteristic displayed by M Loisel in the extract.
a) conceited
b) contended
c) appeased
d) subdued
V The extract uses the phrase ‘elegant dinners’. Which of the following expressions is incorrect
with respect to the word ‘elegant’?

144
a) option i)
b) option ii)
c) option iii)
d) option iv)
“He was silent, stupefied, in dismay, at the sight of his wife weeping. He stammered,
“What is the matter? What is the matter?”
By a violent effort, she had controlled her vexation and responded in a calm voice, wiping her moist
cheeks, “Nothing. Only I have no dress and consequently I cannot go to this affair. Give your card
to some colleague whose wife is better fitted out than I.’’
He was grieved, but answered, “Let us see, Matilda. How much would a suitable costume cost,
something that would serve for other occasions, something very simple?”

She reflected for some seconds thinking of a sum that she could ask for without bringing with it an
immediate refusal and a frightened exclamation from the economical clerk.”
i. What does ‘economical clerk’ indicate?
a) M Loisel was a spendthrift even though he earned a lot.
b) M Loisel was thrifty as he had a meagre income.
c) M Loisel calculated money all the time as he was a clerk.
d) M Loisel was stingy about money and didn’t spend it.
ii Pick the option that correctly classifies fact/s (F) and opinion/s (O) of the people below:

i) F-1, 2 and O-3, 4


j) F-3, O-1,2, 4
k) F-2, 4, O-1,3
l) F-2, 3, 4, O-1

iii Pick the correct set that matches with the feelings of the highlighted words related to the
characters:
Matilda: By a violent effort, she had controlled... M Loisel: He was grieved....
i) Matilda felt aggressive; M Loisel was troubled
ii) Matilda was irritated; M Loisel was upset and cried
145
iii) Matilda tried extremely hard; M Loisel felt intense sorrow
iv) Matilda was quite calm; M Loisel’s heart ached for love
a) Option i)
b) Option ii)
c) Option iii)
d) Option iv)
iv M Loisel was astonished seeing his wife’s reaction. He writes a diary entry that night.
Complete the entry by with the missing words, by choosing the correct option.
11 January, Monday 9:00 pm
I thought Matilda would be (i) _ ___ seeing the invitation in my hand.
However, her reaction has left me (ii)____ ____ ____ I don’t know how I
would be able to (iii)___ ___ a new dress for her.
a) i) vexed ii) disturbed iii) bring
b) i) elated ii) disturbed iii) afford
c) i) keen ii) depressed iii) bring
d) i) elated ii) distressed iii) afford
v Choose the option that gives the most appropriate response to the statement made by the
speaker.

a) I think Matilda was being unreasonable and unrealistic.


b) I feel that M Loisel loved Matilda and wanted her to be happy.
c) In my opinion M Loisel was being too harsh with Matilda.
d) I feel that M Loisel should not have brought the invite home.
Q10 Answer in 20-30 words
i Mme Loisel now knew the horrible life of necessity. Do you think Mme Loisel accepted this
change willingly? Give two reasons in support of your answer.
ii Mention two things you would have done, other than what M Loisel did, to help resolve the
problem of the lost necklace.
iii Mention one of the most pleasurable things, according to you, that money can’t buy and
support your choice with an appropriate reason.
iv Imagine Matilda got to know that M Loisel had given her four hundred francs, which he had
saved to buy a gun. How do you think she would have reacted?
Q1
1 Answer in 40-50 words
i Mme Forestier writes a diary entry after having met Matilda, knowing the truth and seeing
the transformation in her friend. Write a short diary entry as Mme Forestier about your encounter
and how you felt. You may begin like this.

20th July, Monday 9:00 pm


I couldn’t recognise Matilda at all. I wish she had told me the truth about the necklace.
Ii Imagine you’re one of the columnists in a magazine. People write to you about their issues,
seeking your advice.

146
Matilda writes to you about the guilt she feels for having lost a necklace and how it changed the
course of her life and her husband’s. What would be your advice to her?
iii Matilda goes home after meeting Mme Forestier after ten years and shares the fact with M
Loisel that the necklace was a false one. Write the dialogue between Matilda and M Loisel
Matilda: I don’t know how to tell you this but I met Mme Forestier today and she told me that her
necklace was
M Loisel: What! I can’t believe my ears. That huge debt was for nothing!
Matilda: ........................................................................................................................
M Loisel: ......................................................................................................................

Q12 Answer in 100-120 words


i Your teacher conducted an Turn Tables in class, and asked you to put forth your views on
‘Frugality in Life is the Ideal Way to Live’.
Write the debate script with points to supplement your stand, both for and against the motion and
conclude with your own views.
ii One of your friends has to attend a class party at his / her school and wants to purchase the
most expensive clothes and shows. He/ She feels that this would make him/ her stand out in the
crowd.
You receive a call from your friend seeking advice on this matter. Write down that telephonic
conversation between your friend and yourself.

You may begin this way:


Friend: Hey, I want an honest opinion from you. Will you please help me?
You: ......................................................................................................................................
Friend: So, there’s this class party I have to attend. It’s a big deal!
You: .......................................................................................................................................
iii Read the following quote.

“We are too involved in materialistic things, and they don't satisfy us. The loving relationships we
have, the universe around us, we take these things for granted.” - Mitch Albom

Matilda was never satisfied with her life and desired more. The given quote reflects her character.
Justify.

BHOLI by K A Abbas
Extract Based Questions
Read the following extracts carefully and answer the questions that follow.
Question 1(Solved)
“Pitaji! Take back your money. I am not going to marry this man.”
Ramlal was thunderstruck. The guests began to whisper, “So shameless! So ugly and so shameless!”
“Bholi, are you crazy?” shouted Ramlal. “You want to disgrace your family? Have some regard for our
izzat!”
“For the sake of your izzat,” said Bholi, “I was willing to marry this lame old man. But I will not have such
a mean, greedy and contemptible coward as my husband. I won’t, I won’t, I won’t.”
“What a shameless girl! We all thought she was a harmless dumb cow.”

147
Bholi turned violently on the old woman, “Yes, Aunty, you are right. You all thought I was a dumb–driven
cow. That’s why you wanted to hand me over to this heartless creature. But now the dumb cow, the
stammering fool, is speaking. Do you want to hear more?”
(i) When Bholi declared , “……….. I am not going to marry this man.” Her decision portrays her
as______ girl.
(a) rude and shameless
(b) bold and assertive
(c) tongue-tied and mild
(d) sharp and sarcastic
Ans. (b) bold and assertive
(ii) Choose the statements which are true according to the given extract:
1. Bholi had become self-reliant and was able to take decisions at the end of the story.
2. Bholi’s parents considered Bishamber as an unworthy bridegroom.
3. Bishamber had demanded extra money from her father.
4. Bholi refused to marry Bishamber and she demanded money from him.
5. Bholi know that Bishamber was a greedy man of ill-intent.
6. Villagers appreciated Bholi’s step of refusing to marry Bishamber.
(a) 1, 2 and 4
(b) 2, 4 and 6
(c) 4, 5 and 6
(d) 1, 3 and 5
Ans. (d) 1, 3 and 5
(iii) Bholi had refused to get married because:
(a) her father couldn’t afford the dowry that was demanded.
(b) the bridegroom had been greedy.
(c) the bridegroom had insulted her father.
(d) her father was getting her married to a man older to her.
Ans. (c) the bridegroom had insulted her father.
(iv) From the options given below, identify villagers’ character traits in the extract.
1. orthodox
2. helpful
3. understanding
4. conservative
5. gossip monger
(a) 1, 4 and 5
(b) 2, 3, and 4
(c) 1, 3 and 5
(d) 3, 4 and 5
Ans. (a) 1, 4 and 5
(v) Which word does ‘contemptible’ not correspond to?
(a) worthless
(b) shameful
(c) praiseworthy
(d) despicable
Ans. (c) praiseworthy
Short Answer Type Questions: (40-50 words)
1.Why is Bholi’s father worried about her?
2.Does Bholi enjoy her first day at school?
3.Why do Bholi’s parents accept Bishamber’s marriage proposal?
148
4.Bholi had many apprehensions about going to school.
5. What made her feel that she was going to a better place than he How did Bholi’s teacher play an important
role in changing the course of her life?r home?
Long Answer Type Questions: [100-120 words]
1.Bholi’s real name is Sulekha. We are told this right at the beginning. But only in the last but one paragraph
of the story is Bholi called Sulekha again. Why do you think she is called Sulekha at that point in the story?
2.Should girls be aware of their rights and assert them? Should girls and boys have the same rights, duties
and privileges? What are some of the ways in which society treats them differently? When we speak of
‘human rights’, do we differentiate between girls’ rights and boys’ rights?
3.And in her smiling eyes was the light of a deep satisfaction that an artist feels when contemplating the
completion of her masterpiece. Describe the “masterpiece” that was earlier referred to as a “harmless dumb
cow” based on your reading of “Bholi”.
4.Education is the passport to a better life. Discuss on the basis of the story ‘Bholi’.

THE BOOK THAT SAVED THE EARTH


(SET-1)

Reference to the context.


Read the following extracts carefully and answer the questions that follow by choosing the correct
option.
1. OOP: I haven’t a clue. I’ve been to seven galaxies, but I’ve never seen anything like this. Maybe
they’re hats. (He opens a book and puts it on his head.) Say, maybe this is a haberdashery!
OMEGA: (bowing low) Perhaps the Great and Mighty Think-Tank will give us the benefit of his thought
on the matter.
THINK-TANK: Elementary, my dear Omega. Hold one of the items up so that I may view it closely. (Omega
holds a book on the palm of her hand.) Yes, yes, I understand now. Since Earth creatures are always eating,
the place in which you find yourselves is undoubtedly a crude refreshment stand.
OMEGA: (to lota and Ooop) He says we’re in a refreshment stand.
OOP: Well, the Earthlings certainly have a strange diet.
(a) Why did Omega bow low before Think-Tank?
i. As a mark of respect and recognition of supremacy
ii. To flatter and appease the character
iii. As a sign of submission and understanding of his strength
iv. To curry favour and goodwill from the character

(b) Choose the option that associates the person to a ‘haberdashery”.


i. Neetu is a primary school teacher who teaches English.
ii. Kamlesh is a tailor who makes garments for men only.
iii. Mayank is a chemist who manufactures her own medicine.
iv. Gaurav is an engineer who works on designing space stations.

(c) Choose the option that explains what Think-Tank meant by saying-“Elementary, my dear Omega.’
i. It’s simple general knowledge, Omega.”
ii. It’s something that a primary school person won’t understand, Omega.”
iii. “It’s quite obviously deducible, Omega.”
iv. “It’s a little more than complicated,Omega.”
149
(d) The quality of being crude has been allotted to the refreshment stand because:
i. Earthlings have bad eating habits according to Think-Tank.
ii. The temperature of the refreshment stand was too high.
iii. The refreshment stands are responsible for poor health of the Earthlings.
iv. It produces food that is unrefined and unprocessed in nature.
(e) Earth inhabitants are Earthlings and those from Mars are Martians, what are the inhabitants of Venus
addressed as in most sci-fi stories?
i. Venusites
ii. Venatians
iii. Venings
iv. Venusians

Ans (a) I (b) ii (c) iii (d) (iv) e (iv)

SHORT ANSWER QUESTIONS (30-40 WORDS)

Answer the following questions in 30-40 words each.

Q.1.Why did Think-Tank wish to contact the space probe?

Q2. What difficulty do the crew of the space probe face on earth?

Q3. How does Think-Tank explain the books to his probe crew?

LONG ANSWER QUESTIONS (100-120 WORDS)

Answer the following questions in 100-120 words.

Q1. Noodle hails Think-Tank as the most powerful and intelligent creature in the whole
universe. Do you agree? Write your opinion of Think-Tank citing instances from the text.

Q2. Compare and contrast the contrast between the characters of and Think-Thank.

Q3.Explain the guess about the book made by Think-Tank and other Martians.

THE BOOK THAT SAVED THE EARTH


( SET-II)
Reference to the context.
THINK-TANK: Mirror, mirror, in my hand. Who is the most fantastically intellectually gifted being
in the land?
OFFSTAGE VOICE: (after a pause) You, sir.
THINK -TANK: (smacking mirror) Quicker. Answer quicker next time. I hate a slow mirror. (He admires
himself in the mirror.) Ah, there I am. Are we Martians not a handsome race? So much more attractive than

150
those ugly Earthlings with their tiny heads. Noodle, you keep on exercising your mind, and someday you’ll
have a balloon brain just like mine.
NOODLE: Oh, I hope so, Mighty Think-Tank. I hope so.
THINK -TANK: Now, contact the space probe. I want to invade that primitive ball of mud called Earth
before lunch.
NOODLE: It shall be done, sir. (He adjusts levers on switchboard. Electronic buzzes and beeps are heard as
the curtains open.)
i. Choose the option that lists the most appropriate meaning of think-tank from those given below.
a) A tank that helps you think deeper than an average person, if you use its contents regularly.
b) Non-stop flow of ideas from a fixed source about one particular issue.
c) A pool of ideas and solutions to various problems via an individual, group or organisation.
d) A method by way of which people think collectively to identify stray thoughts among them.
Answer: c) A pool of ideas and solutions to various problems via an individual, group or organisation.
ii. “Mirror, mirror, in my hand. Who is the most fantastically intellectually gifted being in the land?”
Choose the option that lists the genre having the original reference to the above lines.
a) Biography
b) Science-fiction
c) Mystery
d) Fairy tale
Answer: d) Fairy tale

iii. Choose the option that correctly categorizes the phrase “(after a pause)” based on the dialogue given
above.
a) aside
b) stage direction
c) narration
d) setting
Answer: b) stage direction

iv. Choose the option that gives you fits the example of an irony from the ones given below.
a) “Who is the most fantastically intellectually gifted being in the land?”
b) “I hate a slow mirror.”
c) “Noodle, you keep on exercising your mind, and someday you’ll have a balloon brain just like mine.
d) “I want to invade that primitive ball of mud called Earth before lunch.”
Answer: c) “Noodle, you keep on exercising your mind, and someday you’ll have a balloon brain just like
mine.
v. Choose the option that includes the display of ‘smacking’ as used in the extract.

a) Option (1)
b) Option (2)
c) Option (3)
d) Option (4)

Answer: d) Option (4)

151
SHORT ANSWER QUESTIONS (30-40 WORDS)

Answer the following questions in 30-40 words

Q1.What did Sergeant Oop tell Think-Tank after he had bitten a corner of a book?

Q2.What thought had come to Thin-Tank’s mind about Vitamin?Why do the space probe crew take
vitamins?

Q3.What is Think-Tank’s interpretation of words like ‘shells’, ‘silver bells’, and ‘garden ’from the
nursery rhyme? What is its impact?
LONG ANSWER QUESTIONS (100-120 WORDS)

Answer the following questions in 100-120 words.

Q1. Explain how does a mere book Mother Goose, one old book of nursery rhymes saved the world from a
Martian invasion?

Q2. The Play, ‘The Book that saved the Earth’ conveys the message that misunderstanding of cultural
differences between various races can cause confusion and conflict. Based on your reading of the play, write
how such confusion and conflicts can be checked so that peace and harmony is maintained.

Sample Question Paper - 1


Class X Exam (2022-23)
English-Language and Literature (184)
Time Allowed: 3 Hrs. Maximum Marks: 80
_________________________________________________________________
General Instructions:
1. 15-minutes prior reading time allotted for Q-paper reading.
2. The Question Paper contains THREE Sections READING-20 Marks, GRAMMAR & WRITING -
20 Marks and LITERATURE – 40 Marks.
3. Attempt question based on specific instructions for each part.
_____________________________________________________________________________________

Section A: Reading Skills (Marks-20)


I. Read the passage given below. (10 Marks)
1. Mountains have always been held in great awe by mankind. They have been a challenge to humans.
Those brave among us have always wanted to conquer them. You see, the more incredible the
mountains, the greater the thrill – a challenge to the bravery of the human race. Climbing mountains
is an experience that is hard to put into words. You are in a beautiful environment and, when you reach
the top, you feel incredible. But you also have to climb down, which is when most accidents happen
– people are tired, it gets dark, and it’s harder. So, mountain climbing is undoubtedly one of the most
popular adventure sports along with being challenging and risky for the climber.
2 Without any perceived risk, there can’t be a feeling that any significant challenge has been
surmounted. Fair, but we have to bear in mind that mountaineering is not a sport that can be embraced
without preparation. The enthusiasts must develop in themselves the spirit of adventure, willingness
152
to undertake hardships and risks, extraordinary powers of perseverance, endurance, and keenness of
purpose before climbing a mountain. They should also know how to handle mountaineering
equipment. Then comes the penance of the rigorous training. This could very well be the lifeline up
there. It helps inculcate and hone survival instincts that allow the climber to negotiate perilous
situations. There are numerous institutes in India and abroad that offer such training.
3 Mountain climbers are unanimous in agreeing that the unpredictable weather is what they fear the
most. There may be sunshine one moment and a snowstorm the other. At higher altitudes, snow is a
regular feature, and being decisive about setting up camps or proceeding further is crucial. The icy
sheets after ice storms make walking treacherous, while the powdery snow makes a mountaineer sink
deep into the snow. Up there, where the intention is to embrace Nature’s wonder, one realizes that it
cannot be done without facing its formidable glory. A true mountaineer may challenge the mountain,
yet is always respectful of the powerful forces of nature.
4 Summiting mountains carries its own health risks such as oxygen and altitude sickness problems,
frost bites, swelling of hands and feet, fluid collection in the brain or lungs, and exhaustion. Yet, the
gratification mountaineers feel from mastering something that is so frightening, urges them to
undertake these endeavours. We may think that the mountaineers are fearless, but experts say, “Not at
all. Its fear that keeps them so intrigued with such arduous journeys.” Impulse and brazenness can be
deadly foes. In the words of the Indian mountaineer, Bachendri Pal, “The biggest risk ... is to not to
take the risk at all. Remember that.”
Based on your understanding of the passage, answer the questions given below.
i. Why does the writer say that mountains inspire ‘awe’ in humans? (Paragraph 1)
A. They present us with opportunities for exciting sports.
B. They evoke the wish in us, to master them.
C. They inspire in us, deeds of valor.
D. They represent peace and calm, to us.
ii. Select the option that corresponds to the following relation below:
The more incredible the mountains—the greater the thrill (Paragraph 1)
A. The higher the stamina—the lower the food intake
B. The more you laugh—the lesser your illness
C. The smaller the car—the bigger the advantage
D. The heavier the luggage—the higher the penalty
iii. Select the option that displays what the writer projects, with reference to the following:
So, mountain climbing is undoubtedly one of the most popular adventure sports (Paragraph 1)
A. doubt
B. caution
C. conviction
D. denial
iv. Complete the following with a phrase from paragraph 1.
Opinion Reason

…………………….. Best experienced rather than described

v. The writer compares training to penance in the line --Then comes the penance of the rigorous
training. (Paragraph 2) State 1 point of similarity between training and penance.
vi. Based on your reading of the text, list 2 reasons why the writer says that “Mountaineering
is not a sport that can be embraced without preparation”. (Paragraph 2)
1)………………….
2)…………………..
153
VII. What connection does the writer draw out between unpredictable weather and the setting
up of camps? (Paragraph 3)
VIII. The writer says, “A true mountaineer may challenge the mountain, yet is always
respectful to the powerful forces of nature.” (Paragraph 3) Select the reason the mountaineer
is respectful to the forces of nature, up in the mountains.
A. survival
B. experience
C. tradition
D. directive
IX. Supply 1 point to justify the following: While mountain-climbing, an impulsive
mountaineer is either disaster-prone or as good as dead
X. Evaluate the INAPPROPRIATE reason for the feeling of exhilaration on reaching a summit
that the mountain climbers experience.
A. Achievement of a seemingly impossible feat
B. Spectacular panoramic view
C. Application of the inculcated survival instincts
D. Opportunity to use sophisticated mountaineering equipment

II. Read the passage given below. (10 Marks)


1 The North-East of India is a melting pot of variegated cultural mosaic of people and races, an ethnic
tapestry of many hues and shades. Yet, these states are less explored as compared to the rest of the
country. The new generations of travellers who are ‘money rich and time poor’ are increasingly
looking for unique experiences --a phenomenon being called the emergence of the ‘experience
economy. For this new and growing breed of tourists, the Northeast with its variety and uniqueness
holds immense attraction.
2. A study conducted in 2020 by Dr. Sherap Bhutia, revealed that the foreign tourist arrival in the
North-East increased from 37,380 persons in 2005 to 118,552 in 2014. The overall growth rate of
tourists (both domestic and foreign) in the Northeast was as high as 26.44% during 2005-06. A Page
4 of 15 2 high and positive growth of 12.53% was registered in foreign tourist visits to the North-
East States of India during 2012 from 2011, which further rose to register a growth of 27.93% during
2013 from 2012. Foreign tourist arrivals in the Northeast witnessed a growth of 39.77% in 2014 from
2013, according to data provided by the Ministry of Tourism, Government of India.
3. The study recommendations for tourism planners included the need to concentrate on some key
areas like enhancement of tourist facilities, tourism financing, focus on community involvement and
others for the formulation of a sustainable tourism strategy in the North-East States of India.

II. Based on your understanding of the passage, answer the questions given below.
I. Infer one reason for the following, based on information in paragraph 1. The rate of tourism
in the North-East of India puzzles tourism officials.
ii. Select the appropriate options to fill in the blanks.
From paragraph 1 we can infer that the _____________ and _________________ of the North-
Eastern States aid attracting the ‘money rich and time poor’ tourists.
1. Distinctiveness
2. Conventionality
3. Diversity
4. Uniformity
5. Modernity
A. 1 & 3
B. 2 &4
154
C. 2 & 5
D. 1 & 4
iii. Complete the following analogy correctly with a word/ phrase from paragraph 1: aroma:
cooking:: _________: painting
iv.Select the correct option to complete the sentence.
Travellers advocating the ‘experience economy’ seek a holiday package with ______________
A. grand facilities, expensive hotels and excellent services to pamper them.
B. a wholesome experience within the budget they have planned for.
C. places and cities to buy things from and opportunities spend money.
D. minimal services, hostel accommodation and many days of touring.
V. Select the chart that appropriately represents the trend of foreign tourist travels in the North-East,
from 2011-2014, as per paragraph 2.
A. Option 1
B. Option 2
C. Option 3
D. Option 4
vi. Fill in the blank by selecting the correct option.
The study of tourist travel statistics in the North-East, from 2005 to 2014 showed ____________ results.
A. expected
B. encouraging
C. inconsistent
D. questionable
VII. Substitute the word ‘witnessed’ with ONE WORD similar in meaning, in the following
sentence
From paragraph 2: Foreign tourist arrivals in the Northeast witnessed a growth of...
VIII. List any 2 examples of ‘tourist facilities as referred to, in Paragraph 3.
IX. List one reason why the researchers recommend that the formulation of a tourism strategy in
the North Eastern States of India be sustainable.
Section B –Grammar (Marks 10)
Attempt ANY TEN of the following questions. (10 Marks)
i. Fill in the blank by choosing the correct option to complete an online update.
The climate control comment by an activist on social media yesterday.
A. blow up
B. blew up
C. is blown
D. will be blown
ii. Read the conversation between a doctor and his patient. Complete the sentence by reporting
the patient’s reply correctly.
Doctor: Do you feel down from time-to-time Mr. Gopalan?
Patient: Yes, I do not stay in a good mood.
The doctor, while trying to figure out his patient’s ailment, asked about his well-being,
to which, the patient affirmed……………..
iii. Select the correct option to fill in the blank for the given line, from a health magazine.
The advertisement read, ‘If you smoke, statistically your story end 15% before it should.
A. must
B. should
C. will
D. ought to

155
iv. Select the option that identifies the error and supplies the correction for the following line,
from a news report:
Last week a child was not allowed to board the plane at Ranchi airport.
Option no. error correction

A. Child Children

B. Last Previous

C. the a

D. at in

v. Complete the given narrative, by filling in the blank with the correct option:
As I was standing on the dock, looking out at the lake for the last time, a feeling of emptiness was over
me like darkness.
A. will wash
B. had washed
C. will have washed
D. washed
vi. Fill in the blank by using the correct form of the word in the bracket, for the given portion of
a letter:
Subject: Request for Approval
Dear Sir
This is to respectfully submit that I…………… (seek) approval for organizing a tree
plantation drive to be undertaken by the club.
vii. Report the dialogue between a grandson and his grandfather, by completing the sentence:
Grandson: Grandpa, who are your superheroes?
Grandpa: Anyone who shows kindness and compassion to others.
In response to the question about his superheroes, grandfather says that………….
viii. Identify the error in the given sentence, from a school magazine report and supply the
correction.
In order to balancing the sentiments of the Eagles and the Hawks, the Student Council suggested a
rematch between the teams.
Error Correction

ix. Sunil shared some information, with Tariq, about a holiday at sea.
Report Tariq’s question: Did you enjoy traveling by sea?
x. Fill in the blank by choosing the correct option, to complete the slogan by the ministry for Child
Welfare.
……………WE AFFORD TO NEGLECT CHILDREN? THINK TWICE!!
A. WILL
B. MAY
C. NEED
156
D. CAN
xi. Select the correct option to complete the narration of the dialogue between Latha and her
father.
Father: Why ask so many questions, Latha?
Latha: I believe that if you don’t know the answer, keep asking till you do!
Father asked Latha the reason for the many questions she was asking. Latha exclaimed
good-humoredly that in event of not knowing the answer one should………….……..
A. keep asking till one does.
B. kept asking till one does.
C. keep asking till one do.
D. kept on to ask till one do.
xii. Identify the error on a shop’s hoarding and supply the correction, for the following sales offer:
Gumnaam & Daughters Pvt. Ltd.
Bindapur, Jharkhand
Massive discount for all senior citizen vaccinated with the precautionary dose.

Error Correction

Section B –Creative Writing Skills (Marks-10)


1. Attempt ANY ONE from A and B given below. (05 Marks)
A. You are Sunidhi Prakash, the Vice Captain of Brilliant Vidyalaya, Barra, and Kanpur. You have
recently noticed several posters around your school premises conveying the hazardous message: Lose
weight in just a month! “A WONDER DIET comes to your rescue … A privilege available for only a
few!” Write a letter to the Editor of The DWA, in not more than 120 words, drawing attention to harm
caused by such advertising. Propose the implementation of “Wholesome Lunch Month’ in schools as
an idea to address such practices, mention the advantages and share suggestions to foster healthy eating
routines and develop positive body image among youngsters.
OR
B. You are Zac Skaria, a resident of D 412, Magna Greens Apartments, Gandhi Marg, Jonpara, Mumbai.
Three students of grade 10 from your residential complex have rescued and rehabilitated a few old
beggars from the neighborhood. You think that their work deserves appreciation and recognition. Write
a letter to the President of the RWA, seeking recommendations for these youth, to be nominated for the
‘Serving Citizens’ Award’. Suggest other ways such acts of kindness could be recognized and awarded
in the future.
2. Attempt ANY ONE from A and B given below. (05 Marks)
A. Gurmeet Kaur is an aspiring candidate for a public-funded engineering college in the suburbs.
She belongs to a nearby village, has minimal technical skills and exposure, has the required cut-off
percentage, and is looking for a complete or partial scholarship. Write a paragraph in about 100-120
words, analyzing her SWOT notes to support your stand on whether she should join/not join the college.

157
STRENGTH WEAKNESS
● Strong Curriculum ● Lack of diversity
● Quality faculty ● Students’ behavioural problems
● Vibrant Activity Clubs ● No hostel facility
● Green location ● Slow repair and maintenance work
● Close proximity to residential areas ● Underutilization of IT Services
● Lack of targeted advertisements to
● out-state students

OPPORTUNITIES THREAT
● Practice-based research ● Lack of publicity in areas of excellence
● Partnerships with professional ● Public perception towards funded colleges
organizations ● Declining students’ interest towards technical
● Strong alumni subjects
● Acclaimed Student Exchange ● Low employee morale due to budget cuts
● Program with European countries

B. Read the following excerpt from an online post of a website on educational practices.
Kids who appreciate how much effort, time and care goes into growing food will understand how
important farmers are, and why it's important to take care of our Earth. In the world of today, gardening
needs to be given more importance than sports, music and dance in all schools because it creates
environmental stewards and outdoor learning laboratories that help the child and community for years
to come. Write a paragraph in 100-120 words to analyze the given argument. You could think about
what alternative explanations might weaken the given conclusion and include rationale / evidence that
would strengthen / counter the given argument.
Section C - Literature (Marks- 40)
Attempt ANY ONE of the two extracts given. (05 Marks)
“Hey, a tea garden!” Rajvir cried excitedly. Pranjol, who had been born and brought up on a plantation,
didn’t share Rajvir’s excitement. “Oh, this is tea country now,” he said. “Assam has the largest
concentration of plantations in the world. You will see enough gardens to last you a lifetime!” “I have
been reading as much as I could about tea,” Rajvir said. “No one really knows who discovered tea but
there are many legends.”
I. Why was Pranjol not as excited as Rajvir about the tea gardens?
A. He disliked looking at tea gardens.
B. He had worked in tea gardens himself.
C. He had grown up in and around tea gardens.
D. He was bored with tea gardens.
II. What does Pranjol mean by saying that Assam has the largest concentration of plantations in
the world?
158
III. Fill in the blank with ONE WORD only.
Pranjol ……………. comes through clearly when he exclaims, “You will see enough gardens to last you a
lifetime!”
IV. How according to Rajvir does the world know about the discovery of tea?
A. Historical places
B. Traditional tales
C. Authentic anecdotes
D. Popular publications
V. Select the option that correctly captures the application of the word ‘cried’ as used in line 1 of the
extract.
A. Jaspreet cried a lot in spite of winning second place in a competition.
B. Jaspreet cried out loud when she saw a white tiger in the sanctuary.
C. Jaspreet cried for hours when the police were unable to find her lost pet.
D. Jaspreet has barely cried since she was three years of age.
OR
B. OMEGA: It shall be done, Sir. Remove vitamins. (Crew takes vitamins from boxes on their belts.) Present
vitamins.(They hold vitamins out in front of them, stiffly.) Swallow vitamins. (They pop the vitamins into
their mouths and gulp simultaneously. They open their eyes wide, their heads shake, and they put their hands
to their foreheads.)
THINK-TANK: Excellent. Now, decipher that code.
ALL: It shall be done, Sir. (They frown over the book, turning pages.)
OMEGA: (brightly) Aha!
IOTA: (brightly) Oho!
OOP: (bursting into laughter) Ha, ha, ha.
THINK-TANK: What does it say? Tell me this instant. Transcribe, Omega.
(The Book that Saved the Earth)
i. Select the option that correctly captures the usage of the word ‘present’ from line 1 of the extract.
A. Oops received a nice present from Think Tank.
B. Iota needs to present his opinion firmly.
C. Omega must focus on the present and leave the past behind.
D. Oops didn’t know anyone even though a crowd was present.
ii. Complete the analogy by selecting the suitable word from the text frown: smile::
gloomily:……………
Solution: ii. frown: smile: gloomily: brightly
iii. Select the option that displays the reason why all crew members were asked to have vitamins. In
order to -
A. boost their physical energies.
B. adapt to their circumstances.
C. quickly turns all the pages.
D. accomplish a specific task.
iv. According to the extract, what did THINK-TANK most likely want OMEGA to do when he said
‘Transcribe…’?
1. read aloud
2. translate
3. make notes
4. interpret
5. record reactions
Select the correct option.
A. 1 & 3
159
B. 2 & 4
C. Only 3
D. 1, 4 and 5
v. The playwright places certain words and sentences in brackets in the given extract.
List any ways these benefit both the director and actors.
i)…………………….
ii)……………….
2. Attempt ANY ONE of the two extracts given. (05 Marks)
A. The trees inside are moving out into the forest, the forest that was empty all these days where no bird
could sit no insect hide no sun bury its feet in shadow the forest that was empty all these nights will be full
of trees by morning. (The Trees)
i. Complete the sentence appropriately.
It is clear that Personification is the poetic device used for ‘No sun bury its feet….’ because. (Clue: explain
how personification applies here)
ii. The poet has used a poetic device in the given lines. What effect does she wish to create by its use?
…no bird could sit no insect hide no sun…
A. emphasis
B. comparison
C. rhyme
D. humor
iii. State whether the following statement is TRUE or FALSE:
The extract uses trees as a symbol for conservative people.
iv. Select the appropriate option to complete the sentence, according to the extract. The idea of
a forest that has been ‘empty all these days’ is.
A. unnatural
B. scary
C. magical
D. legendary
v. How does the use of enjambment impact this extract?
A. It forces frequent pauses.
B. It simplifies the meaning.
C. It builds momentum.
D. It makes the lines lyrical
OR
B.But I can get a hair dye and set such a color there, Brown or black, or carrot, that young men in
despair May love me for myself alone and not my yellow hair.”
i. What is the poet’s tone in the extract?
1. thoughtful
2. authoritative
3. agitated
4. insulting
5. argumentative
Select the appropriate option.
A. 1, 4
B. 3, 5
C. 2, 4
D. 1, 5
ii. What causes the young men to ‘despair’, according to the extract?
iii. Identify the reason for the speaker’s need to color her hair, as per the extract.
160
A. Her control over what makes her look beautiful.
B. Her desire to be loved for inner beauty
C. Her need to change people’s perception of beauty
D. Her conviction that she is beautiful inside
iv. Complete the analogy about the speaker’s hair.
yellow: blonde :: : carrot: ………
v. Select the sentence in which the word ‘set’ is used in a similar manner as line 2 of the extract.
A. I want to set him up and get my work done this time.
B. Do you have another set of the books that I can read?
C. The dessert needs to set for two hours before being served.
D. The set for the school play looked quite grand.
VI. Answer ANY FOUR of the following in about 40-50 words each. (3x4=12 Marks)
i. Validate the given statement with reference to baby seagull’s fear. ‘Fear doesn’t exist anywhere else
other than one’s mind.’ (His First Flight- Two Stories about Flying)
II. Explain why the poet personally holds the conviction that the world will primarily end in fire? (Fire
and Ice)
iii. Valli’s unique maiden bus ride experience could be possible because she belonged to a small
village. Do you agree? Why? /Why not? (2 reasons) (Madam Rides a Bus)
iv. Give one reason why ‘The Tale of Custard the Dragon’ is more a fable than a ballad.
v. How can we say that Natalya was continuously successful in maintaining an upper hand during her
arguments with Lomov? (Any one example) (The Proposal)

VII. Answer ANY TWO of the following in about 40-50 words each. (3x2=06 Marks)
i. Dr. Herriot knew his patients as well as their owners really well. Discuss. (The Triumph of Surgery)
ii. State one likely reason the writer of The Midnight Visitor chose to characterize Ausable as short
and fat.
iii. Validate the importance of small, fun learning tasks towards successful careers, in the context of
Richard Ebright in The Making of a Scientist.
VIII Answer ANY ONE of the following in about 100-120 words. (06 Marks)
i. Mijbil and the Tiger, both were looked after by humans. Assume they both meet each other in the
zoo and have a conversation about their lifestyle and feelings. Write this conversation as per your
understanding of Mijbil the Otter and A Tiger in the Zoo. You may begin like this Tiger: Thanks for
visiting me, though I don’t usually like visitors. Mijbil: Oh? I would love visitors, I think.
OR
ii. “Not from weeping nor from grieving will anyone obtain peace of mind’. If you had to use the
message of the given quote from the Buddha’s sermon (The Sermon at Benares) to help the boy cope
with the loss of his ball and what it signifies (The Ball Poem), what would you include in your advice?
Also, evaluate why it might be difficult for him to understand the notion.
IX Answer ANY ONE of the following in about 100-120 words. (06 Marks)
i. Fiction writers prefer creating grey characters rather than black and white. Analyze this in detail,
with reference to both the characters of The Thief’s Story
ii. ‘Honor among thieves’ is considered a popular code. Examine ‘A Question of Trust’ as a story
woven around this code.

Sample Question Paper - 2


Class X Exam (2022-23)
161
English-Language and Literature (184)
Time allowed: 3 Hrs. Maximum Marks: 80

Q1. Read the passage given below. 10 M


1. The choices we make on a daily basis-wearing a seat belt, lifting heavy objects correctly or
purposely staying out of any dangerous situation-can either ensure our safety or bring about potentially
harmful circumstances.
2. You and I need to make a decision that we are going to get our lives in order. Exercising self-
control, self-discipline and establishing boundaries and borders in our lives are some of the most
important things we can do. A life without discipline is one that's filled with carelessness.
3. We can think it's kind of exciting to live life on the edge. We like the image of "Yeah! That's me!
Living on the edge! Woo-hoo!" It's become a popular way to look at life. But if you see, even highways
have lines, which provide margins for our safety while we're driving. If we go over one side, we'll go
into the ditch. If we cross over the line in the middle, we could get killed. And we like those lines
because they help to keep us safe. Sometimes we don't even realize how lines help to keep us safe.
4. I'm not proud of this, but for the first 20 years of my life at work, I ignored my limits. I felt horrible,
physically, most of the time. I used to tell myself "I know I have limits and that I've reached them, but
I'm going to ignore them and see if or how long I can get by with it." I ran to doctors, trying to make
myself feel better through pills, vitamins, natural stuff and anything I could get my hands on. Some of
the doctors would tell me, "It's just stress." That just made me mad. I thought stress meant you don't
like what you do or can't handle life, and I love what I do. But I kept pushing myself, travelling, doing
speaking engagements and so on- simply exhausting myself.
5. Finally, I understood I was living an unsustainable life and needed to make some changes in my outlook
and lifestyle.
6. You and I don't have to be like everyone else or keep up with anyone else. Each of us needs to be exactly
the way we are, and we don't have to apologize for it. We're not all alike and we need to find a comfort zone
in which we can enjoy our lives instead of making ourselves sick with an overload of stress and pressure.

Based on your understanding of the passage, answer the questions given below:
i) Supply one point to justify the following: (paragraph 3)
‘Sometimes we don’t even realize how lines help to keep us safe’.
ii) Complete the following.
OPINION REASON

Life on edge has become popular

iii) What is the tone of the author in this write-up?


iv) Based on your understanding of the text, list 2 reasons why the writer says that
“We’re not all alike” (paragraph 6)
1) _________________________________________________________.
2) _________________________________________________________.
v) Choose the option that best captures the central idea of the passage from the given quotes.

162
It's all about quality of life and To go beyond is as wrong as to fall
finding a happy balance between Short.
Work and friends. Confucius
Philip Green (2)
(1)

Life is like riding a bicycle. To Balance is not something you


keep your balance, you must keep find, it's something you create.
moving.
Albert Einstein Jana Kingsford
(3) (4)

(A) Option 1 (B) Option 2 (C) Option 3 (D) Option 4


vi) What does the writer mean by the expression “potentially harmful circumstance”?
vii) The author uses colloquial words such as "yeah" and "Woo-hoo!” Which of the following is NOT a
colloquial word?
(A) hooked (B) guy (C) stuff (D) stress

viii) Which of the characteristics are apt about the writer in the following context
"I know I have limits and that I've reached them, but I'm going to ignore them and see if or how long I can
get by with it."?
1. Negligent 2. Indecisive 3. Spontaneous 4. Reckless 5. Purposeless 6. Patient

(A) 2and 5 (B) 3 and 6 (C) 1 and 4 (D) 2 and 3


ix) Select the option that makes the correct use of "unsustainable", as used in the passage, to fill in the blank
space.

(A) In the long run, the __________ officials followed emergency procedures.
(B) Emergency procedures were _____________ by the officials.
(C) Officials reported an _____________ set of events during the emergency.
(D) Officials admit that the emergency system is ______________ in the longer run.

x) Choose the option that correctly states the two meanings of 'outlook', as used in the passage.
1. A person's evaluation of life
2. A person's experiences in life
3. A person's point of view towards life
4. A person's regrets in life
5. A person's general attitude to life

(A) (1) and (4) (B) (2) and (3) (C) (3) and (5) (D) (4) and (5)

QII. Read the passage given below 10 M


1. Pain is probably the only emotion that founders of crypto currency exchanges and retail traders in crypto
coins share at this moment. Investors have been hurt by discomforting taxes and significant losses, which
threaten to make the future of crypto trading uncertain. The present puts into haze the story of India’s only
163
two crypto unicorns—Coin DCX and Coin Switch Kuber. But just a few years ago, these entrepreneurs had
muscled their way through adversities as these are new-world, volatile, unregulated, digital assets operating
in a complex ecosystem.
2. Most of India’s large crypto exchanges saw a boost to their total income and profitability from 2019
onwards—led by a surge in interest and the adoption of crypto trading in India.

3. Most of India’s crypto exchanges have a simple formula, where income is generated through
brokerage fees, matching buyers and sellers on an exchange’s platform. Investors also pay a flexible
withdrawal fee when coins in an exchange wallet are withdrawn to a personal wallet. Exchanges also
earn through listing fees, when crypto coins are listed at a particular exchange.
4. Coin Switch buys and sells crypto on behalf of users, allowing them to invest and divest through
Indian rupees. In return, it charges a trading fee and commission. The exchange, however, does not
reinvest users’ crypto in any form or manner, and has zero leverage on them. This simple and
transparent trading method has spelt success for the exchanges. “Crypto is a business that touches
money, forex, security and trust. The team at Coin Switch advocates a safe, calculated and trustworthy
approach to crypto investment, and is transparent.
(This story appeared in the 23 September, 2022 issue of Forbes India)
Based on your understanding of the passage, answer the questions given
below. (1x10=10 marks)

1. List two threats that Crypto currency might pose in the future. 1

164
2. How is the income generated through digital currency in Indian exchange? 1

3. Infer one reason for the following, based on the information in para 1. 1
The entrepreneurs had muscled their way through adversities.

4. Complete the following analogy correctly with a word/phrase from paragraph 3. 1


Investment : business :: _____________ : crypto currency

5. Select the appropriate option to fill in the blanks. 1


From para 4, we can infer that Coin Switch upholds________________ and ____________
in their business approach making it transparent.
1. Reliability
2. Forex
3. Reinvestment
4. Guarantee
5. Brokerage
a) 1&4 b) 2&5 c) 1&3 d) 4&5

6. Substitute the word ‘boost’ with one word similar in meaning, in the 1
following sentence from paragraph 2.
Crypto exchanges saw a boost in their total income.

7. Which unicorn currency kicked off their business with loss and in which 1
financial year?

165
8. Analyse the graph and find out in which financial year the two crypto 1
unicorns had the least difference in their total expenditure.
1. FY 2019
2. FY 2020
3. FY 2021
4. FY 2018
a) 1&2 b) 1 only c) 2 only d) 1, 3&4

9. What is the difference in the total income of the two unicorns in the 3 financial years put 1
together?
a) 59.57% b) 19.2% c) 47.15% d) 47.59%

10. Which image relates to UNICORN?

Section –B (Grammar)
QIII. Attempt ANY TEN of the following:
(i) Fill in the blanks by choosing the correct option to complete this piece of news.
Rupee _______ 5 pie to 79.47 against the US dollar amid rate hike concerns.
a) rises
b) rose
c) risen
d) risen

166
(ii) Read the conversation between a father and son. Complete the sentence by reporting the patient’s reply
correctly.
Father: Where are you going?
Son: I am going to my friend’s house.
Father asked his son where he was going. The son replied ______________ to his friend’s house.
(iii) Identify the error and supply the correction.
We, there in India have so many religions, eat so many different foods and wear so many different kinds of
dresses.

Error Correction

(iv)Complete the given narrative, by filling the blank with the correct option:
Piyush went to the field to _______ football match as a goalkeeper.
a) play
b) played
c) playing
d) has been playing
(v) Select the correct option to fill in the blank for the given line, about a rare musical instrument.
The Vichitra Veena as the name suggests is a rare instrument and there are not ______ artists who can
play this instrument.
a) much
b) many
c) more
d) most
(vi) Report the dialogue between Ravi and Peon, by completing the sentence:
Ram: I want to meet the principal.
Peon: Sir, he is not in the office.
Ram went to the Principal’s office and told the peon that he wanted to meet the Principal. Respectfully, the
peon told him that ____________.
(vii) Fill in the blank by using the correct form of the word in the bracket, for the given portion of a letter.
Subject: Permission to participate in Half-Marathon
Sir,
It is a matter of great pleasure that the authorities of our school has___________ (decide) to take part in
the half-marathon organized by your reputed NGO.
(viii) Report the dialogue between Ganesh and Dr Gautam, by completing the sentence.
Ganesh told Dr Gautam that his wife was unwell and asked if he might know what would be the total
expenses for her treatment. The doctor replied that _________
for her treatment.
(ix) Fill in the blank by choosing the correct option, to complete this piece of information about the ruler of
Kushanas.
Kanishka, the ruler of the Kushanas embraced Buddhism. It was during his time that Buddhism_________
to Sri Lanka, Myanmar and Java.
(x) Identify the error in the given sentence about electricity and supply the correction.
Electricity is express by the amount of charge flowing through a particular area in unit time.

Error Correction

(xi) Report the dialogue between Rajeev and his dad, by completing the sentence.
Rajeev: Dad, will you gift me a laptop?
Dad: Not now. Wait till your class 12 board results.
167
Rajeev asked his dad if he would gift him a laptop. Negating, Dad asked Rajeev______________.
(xii) Select the correct option to fill in the blank for the piece of information about Belgium.
Belgium is considered to be country of chocolates. Most Belgian chocolate makers ___________ their craft
at Vocational Schools.
a) are learning
b) have learnt
c) will learn
d) learn
Section-C: Creative Writing Skills (10 marks)
1. Attempt ANY ONE of the following in 100-120 words. (5 marks)
A. You are Neha/Nihal Singh, Computer In-charge of Army Public School. Your school wishes to buy 20
computers and computer accessories from Apple Inc, Nehru Place, New Delhi. Draft a letter placing an
order for the same.
OR
B. You noticed while traveling in the Metro that despite the strict vigilance, people try to deface the Metro
rains and stations. Write a letter to the editor of an evening newspaper suggesting DMRC, Delhi to start
some awareness programmes at the school level in coordination with the Directorate of Education
explaining how such programmes may help in bringing a positive change in people's attitude. You are
Sudheer/Sweety of 3-B, DDA Flats, Punjabi Bagh, Delhi.

2. Attempt ANY ONE of the following in 100-120 words. (5 marks)


A.The following graph shows the number of visitors (in millions) who visited the famous Salarjung Museum
in Hyderabad between 2000 and 2010 as well as the number of visitors expected to visit it in the next ten
years.

Write an analytical paragraph interpreting the given data.


OR
B. Look at the data given below which shows the changes that have occurred between 1995 to 2010 in
Bilaspur regarding women's education. Write a paragraph to record the changes by interpreting this data.

Female Education in Bilaspur 1995 2010


1 No. of girl's school 10 15
2 No. of women colleges 3 4
3 No. of students in schools 10,000 16,000
4 No. of students in colleges 6000 10,000
5 Literacy rate 30% 55%
6 Distance education centers Nil 03

168
Section- D: Literature (40 Marks)
V. Reference to context (10 marks)

1. Attempt ANY ONE of two extracts given. (5marks)


A. But, please, Stephen Stepanovitch, how can they be yours? Do be a reasonable man. My aunt’s
grandmother gave the meadows for the temporary and free use of your grandfather's peasants. The
peasants used the land for forty years and got accustomed to it as if it was their own, when it happened
that....
CHUBUKOV: Excuse me, my precious. You forget just this, that the peasants did not pay your grandmother
and all that because the Meadows were in dispute.
(i) Who is the speaker of above lines?
a) Stephen c) Ivan
b) Natalya d) Stranger
(ii) The meadows were given by my aunt's grandmother for the temporary and free use of your
grandfather's_______.
(A) labourers (C) servants
(B) Peasants (D) drivers
(iii) For how many years had peasants used the land?
(A) Forty (C) Fifty
(B) Thirty (D) Twenty
(iv) Find out the antonym of 'permanent' from the passage.
(A) reasonable (C) accustomed
(B) meadows (D) temporary
(v) Which land is being talked about in these lines?
(A) orchard (B) oxen meadows
(C) tea-garden (D) meadows
OR
B. The following Sunday, Lencho came a bit earlier than usual to ask if there was a letter for him. It was the
postman himself who handed the letter to him while the postmaster, experiencing the contentment of a man
who has performed a good deed, looked on from his office. Lencho showed not the slightest surprise on
seeing the money; such was his confidence - but he became angry when he counted the money. God could
not have made a mistake, nor could he have denied Lencho what he had requested.
(i) Was Lencho surprised on seeing the money?
(A) a little (B) not at all
(C) very much (D) a sort of
(ii) Lencho did not get 100 pesos from God, it was only.
(A) Eighty pesos (B) Fifty pesos
(C) Seventy pesos (D) Sixty pesos
(iii) Choose the characteristic displayed by Lencho when he opened the letter.
(A) Lencho's belief in God was strong.
(B) Lencho's belief in God was weak.
(C) Lencho's belief in God was somewhat strong.
(D) Lencho's belief in God was very weak.
(iv) Who handed the letter to Lencho?
(A) The Postmaster (B) The Clerk
(C) The Postman (D) Stranger
(v) What is the antonym of 'accepted' in the given extract?

2. Attempt ANY ONE of two extracts given. (5marks)


A. Pistol in his left hand, pistol in his right,
And he held in his teeth a cutlass bright,
His beard was black, one leg was wood;

169
It was clear that the pirate meant no good.
(i) Who possessed pistols?
(A) Dragon (C) Pirate
(B) Belinda (D) Dog
(ii) What was the colour of 'his' beard?
(A) Grey (C) Brown
(B) Black (D) White
(iii) What was his one leg made of?
(A) Steel (C) Wood
(B) Iron (D) Alloy
(iv) Find a word in the passage which means the same as 'thief.
(A) Pirate (C) Burglar
(B) Robber (D) Pick pocketer
(v) Name the poet.
(A) Robert frost (C) Ogden Nash
(B) Carl Sandburg (D) Robin Klein
OR
B.He should be lurking in shadow,
Sliding through long grass
Near the water hole
Where plump deer pass

(i) Tiger is passing through the


(A) grass (B) trees
(C) mountains (D) river
(ii) For whom is the tiger waiting near the water hole?
(A) Fat elephant (B) Fat hare
(C) Fat deer (D) Fat ox
(iii) Find a word from the passage which means the same as 'fat'.
(A) Lurking (B) Sliding
(C) Rage (D) Plump
(iv)Who is 'he' here?
(A) The Lion (C) The Camel
(B) The Tiger (D) The Giraffe
(v) The poetic device used in line number four is
(A) Metaphor (B) Alliteration
(C) Personification (D) anaphora
VI. Answer ANY FOUR of the following in about 40-50 words each. (4 × 3 = 12)
(i) Evaluate the line - Some say the world will end in fire/Some say in ice - in the context of volcanic
eruptions, forest fires, meteor collisions, melting ice caps, etc.?
(ii) What details did Valli pick up about the bus journey? How did she pick up these details?
(iii) 'Paper has more patience than people'. Elucidate.
(iv) Why did Lencho write a letter to God?
(v) What did Buddha do after he had attained enlightenment? Why?
VII. Answer ANY TWO of the following in about 40-50 words each:
(2x 3 = 6)
(i) What were the factors which contributed in making Ebright a scientist?
(ii) Why was Dr Herriot confident that Tricki will be in the hospital room soon?
(iii) Why was Matilda Loisel always unhappy?
VIII. Answer ANY ONE of the following in about 100-120 words. (1 × 6 = 6)
(i) His father and mother had come around calling to him shrilly, upbraiding him, threatening him. What
role was played by the family of the seagull to train him in the art of flying?
OR
(ii) 'You seem to have done your homework before coming." Answer the given question in light of this
statement. Rajvir seemed to have a lot of information about tea. What all did he tell? What character trait of
his is revealed? Is it essential for children?

IX. Answer ANY ONE of the following in about 100-120 words. (1x 6 = 6)
170
(i) Why did the thief befriend Anil? Why did he steal Anil's money? Why did he come back and put Anil's
money back?
OR
(il) How does Think Tank compare the Martians with the people on the Earth? What guesses are made by
him about the books found on earth?

Sample Question Paper - 3


Class X Exam (2022-23)
English-Language and Literature (184)
Time allowed: 3 Hrs. Maximum Marks: 80

Section A: READING SKILLS


I. Read the passage given below:
(1) Happiness lies within the mind of an individual. No amount of external wealth may be helpful in
making him happy. Our forefathers had lifestyles based mainly on the concept of “simple living & high
thinking”. Excessive material wealth did not mean much to them.
(2) The structural framework of our forefather’s families was different from those of ours. The bond of
love which they shared cannot be easily found today. The family provided an emotional cushioning effect
against tension and stress. However, in today’s nuclear family, we are detached from feelings towards our
kith and kin. This may finally destroy the family psyche.
(3) The joint family system provided a proper environment for the child to grow up. The values of
respect, tolerance, responsibility, and integrity were internalized in the child. In the long run, they became
better human beings, compared to those in the present generation. Our forefathers felt happiest, if their
children became honest human beings. But today, we are happy only if we attain our coveted material ends
and are ready to resort to any means to attain that end.
(4) Our forefathers had a vision to make India the best. To attain their ambitions, they were ready to
make all sorts of personal sacrifices. On the other hand, today people are ready to migrate to the west, to
enjoy a comfortable life. Often, they become successful in foreign lands. But, in the process, they become
alienated from their motherland. Also, distance from their ailing parents is a worrying factor and keeps them
perturbed. It is not easy for them to return, as their children will not be able to adjust to the Indian
environment and way of life. Thus, this is a crisis and a frame of mind worse than their forefathers.
(5) In our generation, tradition and modernity have intermingled to form a special system. We are
happier than our forefathers in being able to lessen evils like the rigid caste system, untouchability, and
child marriage, but we have failed to totally eradicate them. I fact, electoral politics has made use of the
caste system, through the issuance of party tickets on the basis of caste, community, and religion.
On the basis of your understanding of the passage, answer the questions that follow: (1×10 = 10)
(a) How are our forefathers different from us?
(i) They had lifestyles based mainly on the concept of “simple living and high thinking”
(ii) They had different structural framework of families.
(iii) They made all sorts of personal sacrifices to attain their ambitions
(iv) All of these
(b) What impact does a joint family system has on a child?
(i) It helps a child to become a better human being.
(ii) It internalizes the values of respect, tolerance, responsibility and integrity in a child
(iii) Both (i) & (ii)
(iv) Neither (i) nor (ii)
(c) In our generation, ---------------- and ------------ have intermingled to form a special system.
(i) Untouchability, Intolerance
(ii) Tradition: modernity
(iii) Religion: caste
(iv) Politics; corruption
(d) based on your reading of the passage, choose the incorrect statement from the following.
(i) Our youngsters had a vision to make India the best
(ii) Happiness lies within the mind of an individual.
171
(iii) The family provided an emotional cushioning effect against tension and stress.
(iv) Our forefathers had lifestyles based mainly on the concept of simple living and high thinking.
(e) Choose the option that correctly states the two meanings of ‘cushioning’, as used in the passage
(A) Sharpening (B) Shielding (C) Intensifying (D) Softening (E) Irritating
(i) A, E (ii) B, D (iii) C, E (iv) A, C
(f) The ------------ provided a proper environment for the child to grow up.
(i) Neighbourhood (ii) joint family system (iii) nuclear family system (iv) friends & relatives
(g) “In our generation, tradition and modernity have intermingled to form a special
system.” Substitute the underlined word with the most appropriate option from the
following:
(i) Divided (ii) Blended (iii) Scattered (iv) Detached
(h) We are happier than our forefathers in being able to lessen:
(i) child marriage (ii) unemployment (iii) religious values (iv)none of these
(i) Select the option that makes the correct use of “sorts”, as used in the passage, to fill in the blank space.
(i) Policemen have to know all ----------- of different things.
(ii) The machine ------- it all out.
(iii) The computer -------- the words into alphabetical order.
(iv)He always ---------- the books according to the author’s name.
(j) What are the values internalized in the child?
(i) Respect (ii) Tolerance (iii) Responsibility (iv) All of these
II. Read the passage given below: (10 marks)

Marie was born in 1867 in Warsaw, Poland, where her father was a Professor of Physics. At an early age,
she displayed a brilliant mind and a blithe personality. Her great exuberance for learning prompted her to
continue with her studies after high school. She became disgruntled, however, when she learned that the
University of Warsaw was closed to women. Determined to receive a higher education, she defiantly left
Poland and in 1891, entered the Sorbonne, a French university, where she earned her master’s degree and
a doctorate in physics.

EVENT YEAR

Marie was born on the 7th of November 1867

Her sisters Bronya and Zosia became ill and Zosia passed away. 1876

Curie’s mother died from grief. 1878

She graduates high school. 1883

Leaves Warsaw for a governess position 1884

Her sister Bronya graduated medical school. 1891

Marie graduated with a master’s degree in Mathematics and first in her class for Physics. 1893

172
She married Pierre Curie. 1895

Marie Curie was


a. The first woman with a degree in Physics.
b. The first woman to graduate in France.
c. The first woman to obtain a Nobel Prize.
d. The first woman to obtain a chair at the Sorbonne.
e. The first scientist to obtain two Nobel Prizes.

(2) Marie was fortunate to have studied at the Sorbonne with some of the greatest scientists of her days,
one of whom was Pierre Curie. Marie and Pierre were married in 1895 and spent many productive years
working together in the physics laboratory. A short time after they discovered radium, Pierre was killed by a
horse-drawn wagon in 1906. Marie was stunned by this horrible misfortune and endured heart-breaking
anguish. Despondently she recalled their close relationship and the joy that they had shared in scientific
research. The fact that she had two young daughters to raise by herself greatly increased her distress.
(3) Curie’s feeling of desolation finally began to fade when she was asked to succeed her husband as a
physics professor at the Sorbonne. She was the first woman to be given a professorship at the world-
famous university. In 1911, she received the Nobel Prize in chemistry for isolating radium. Although Marie
Curie eventually suffered a fatal illness from her long exposure to radium, she never became disillusioned
about her work. Regardless of the consequences, she had dedicated herself to science and to revealing
the mysteries of the physical world.
On the basis of your understanding of the passage, answer the questions that follow: 1 × 10 = 10
Q1. In the line “…… she defiantly left Poland,” the word “defiantly” DOES NOT refer to
(a) Coldly (b) Challenging (c) Resistant (d) Openly
Q2. The events in the life of Marie Curie could reveal the following point:
(a) She was determined to receive higher education.
(b) She was eager for women empowerment.
(c) She had a brilliant mind set and rude behaviour.
(d) She never liked her personal and professional life with Pierre Curie.
Q3. Marie …………………… by leaving Poland and travelling to France to enter the Sorbonne.
(a) challenged authority (b) showed intelligence (c) got happy (d) was distressed
Q4. Choose the option that lists the correct answers for the following:
(a) Ashwini, a physicist was working hard on her research on the black hole. She worked really very hard to
make her research a unique one. She did a lot of experimental work to come up with better inferences.
What kind of researcher is she?
(b) Lata was a lady of firm determination. She lost her husband after three years of marriage. She had two
daughters whom she raised all her own. She felt the void in her life but was never disappointed or
disheartened by that. What kind of lady is she?
(a) (1) disappointed and (2) experimental (b) (1) determined (2) independent
(c) (1) independent (2) determined (d) (1) diligent and (2) resolute
Q5. Based on your understanding of the passage, choose the option that lists the qualities of the professor.
(1) Scholar (2) Distressed (3) Exuberant (4) Lethargic (5) A bad student (6) Thoughtful
(a) (2) and (4) (b) (5) and (6) (c) (1) and (3) (d) (3) and (6)
Q6. Even though she became fatally ill by working with radium, Marie Curie was never…
(a) cheerful (b) enthusiastic (c) staunch (d) disillusioned
Q7. Marie became ‘aggrieved’ when she found that
(a) Her husband was dead. (b) The University of Warsaw was closed for women.
(c) She was ill. (d) She will have to leave her birthplace.
Q8. According to the passage, how long did the married life of Marie & Pierre Curie survive?
(a) 12 years (b) 10 years (c) 11 years (d) 15 years
Q9. Marie’s mother was …………………… and died after the death of …………………….
173
(a) heartbroken, her friends (b) depressed, two of her daughters
(c) heartbroken, one of her daughters (d) heartbroken, her cousin
Q10. Choose the option that lists statements that are NOT TRUE.
(a) Marie was brilliant in her studies.
(b) Marie was the first scientist to obtain two Nobel Prizes.
(c) Marie succeeded her husband Pierre at Sorbonne University.
(d) Marie didn’t reveal many facts and mysteries of the physical world.
Section –B (Grammar)
QIII. Attempt ANY TEN of the following:
i) Select the option that identifies the error and supplies the correction for the following line.
Sachin Tendulkar is a best cricketer in India.

Error Correction

ii) Read the conversation given below and complete the line that follows. (01 M)
Sujon: I want to be a doctor. What about you?
Rajib: I want to be an English teacher.
Sujon said that he wanted to be a doctor and, in turn, asked what about him. To this, Rajib replied saying
that ----------.
iii) Put the verb in bracket into correct form.
I___________ (not see) him since yesterday.
iv) Fill in the blank with a suitable form of verb from the bracket.
He was tired and …….. asleep immediately (fell, fall)
v) Fill the blank with appropriate conjunction.
Not only he ……… his father was also present.
a) Is b) but c) can d) also
vi) Communication …… become more effective due to smart phones.
a) Is b) have c) has d) had
vii) The police ……. four thieves last night.
a) Catch b) caught c) is catching d) will catch
viii) My mother often says to me, ‘Honesty is the best policy’.
My mother often says to me that ……………………….
ix) Rajiv …….. Karate class every day.
a) attends b) attending c) attended d) will attend
x) Select the option that identifies the error and supplies the correction for the following line.
Children are fond of coconuts.

Error Correction

IV. Creative Writing Skills


(10 Marks)
(i) You are Siddhartha Sinha, the principal of Vaidya Kanya Public school, Faridabad, Haryana, place
an order for school furniture in the form of a letter to Jasmeet Traders & Co. Sharma Market, Haryana (100-
120 words) (05 Marks)
(ii) The pie chart shows the proportion of people from different households living in poverty in the UK in
2002. Write an analytical paragraph to describe the information in 100-120 words.
(05 Marks)

174
SECTION - C LITERATURE (40 Marks)
V. Reference to the Context (5+5=10)
Read the extract given below and attempt it by answering the questions that follow. (5x1 = 5)
A.Well, it’s time I did some real work, I told myself; I ‘m out of practice. And if I don’t take the money, he’ll
only waste it on his friends. After all, he doesn’t even pay me.
(a) What is referred to as ‘real work’?
(i) Writing articles (ii) cooking (iii) cleaning (iv) stealing
(b) What will he waste on his friend?
(i) Time (ii) Money (iii) Food (iv) All of these
(c) Hari wants to steal Anil’s money because
(i) He has to pay his debt
(ii) He has not stolen anything for a long time
(iii) The money belongs to him
(iv) none of these
(d) Anil is a --------- man.
(i) Careful (ii) alcoholic (iii) careless (iv) hardworking
(e) Which word from the following is the antonym of “useful”
(i) Waste (ii) pay (iii) take (iv) practice
OR
Read the extract given below and attempt it by answering the questions that follow. (5x1 = 5)
B. An ultimate shaking grief fixes the boy
As he stands rigid, trembling, staring down
All his young days in the harbor where
His ball went. I would not intrude on him;
i. The poet uses the word ‘ultimate’ to describe the boy’s reaction. Pick the meaning that DOES NOT
display what ‘ultimate’ means in the context given.
a) Consequent b) final c) conclusive d) fateful
ii.The boy is very young in this poem. As a mature, balanced grown-up, he might look back and think that
his reaction of ‘ultimate shaking grief’ was
1) disproportionate to the loss.
2) Pretend to procure a new toy.
3) according to his exposure and experience then.
4) a reaction to the failure of retrieving the toy.
5) justified and similar to what it would be currently.
a) 5 & 2 b) 1 & 3 c) 2 & 4 d) 3 & 5
iii. Pick the option that lists the boy’s thoughts, matching with the line-As he stands rigid, trembling, and
staring down.

175
a) Option 1 b) Option 2 c) Option 3 d) Option 4

iv. Why does the speaker choose not to intrude? This is so because the poet…
a) knows that it would embarrass the boy in his moment of grief.
b) feels that it’s important that the boy learn an important life lesson, undisturbed.
c) realizes that he doesn’t have sufficient funds to purchase a new ball for the boy.
d) Experiences a sense of distress himself, by looking at the boy’s condition.

v. Choose the option that lists the meaning of “harbour’ as used in the extract.
Noun- (1) A place on the coast where ships may moor in shelter.
(2) A place of refuge
Verb- (3) keep (a thought or feeling, typically a negative one) in one’s mind, especially secretly.
(4) Shelter or hide (a criminal or wanted person)
(i) Option (1) (ii) Option (2) (iii) Option (3) (iv) Option (4)
(30 Marks)
(VI) Answer ANY FOUR questions in 40-50 words each. (4x3=12)
a) Why was the young seagull alone at the ledge?
b) What does courage mean to Mandela?
c) How did Anne justify her being a chatterbox in her essay?
d) How did rain change? What happened to Lencho’s fields?
e) Describe the movement of the Tiger in the cage and in the wild.
f) Is Amanda an orphan? Why does she say so?
(VII) Answer ANY TWO questions in 40-50 words each. (2x3=06)
a) How is Ausable different from other secret agents?
b) Why is Mrs. Pumphrey worried about Tricki?
c) Why does Horace Danby steal every year?
d) Who is Fowler and what is his first authentic thrill of the day?
(VIII) Answer ANY ONE of the following in 100-120 words. (6 Marks)
(a) It is true that without the help of the members of family, the young seagull could not fly. Elaborate.
OR
(b) Our attitude towards a situation evokes both negative and positive responses. Analyse this with
reference to the poem, “Dust of Snow” to bring out the inherent valuable lessons.

(IX) Answer ANY ONE of the following in 100-120 words. (6 Marks)


(a) In the story ‘A Questions of Trust’, Horace Danby carefully planned his theft, but was outwitted by
another thief ‘The lady in red’. Would you agree that over confidence may prove fatal one day? Discuss.

OR
(b) How did Herriot save Tricki’s life?

176
Sample Question Paper - 4
Class X Exam (2022-23)
English-Language and Literature (184)
Time allowed: 3 Hrs. Maximum Marks: 80
Instructions
1. Read the questions carefully.
2. The paper has been divided into 3 sections:
● Section A: Reading (20 marks)
● Section B: Writing Skills & Grammar(20 marks)
● Section C: Literature (40 marks)
3. Write question number before writing your answers.
4. Attempt all the questions as indicated.

SECTION A : READING SKILLS


I. Read the passage given below and answer the questions that follow. (10 Marks)
Improving children's penmanship, legibility, pencil grasp, and working on problem areas are all important
things that need to be addressed—especially in the digital age when most of our written communication is
done through texting on mobile devices or typing on a keyboard. Voice recognition software has even
evolved to the point that commands can now be verbal—making even keystrokes obsolete!
While the American Academy of Pediatrics (AAP) does not currently have a policy on handwriting, here's
what we do know based on the published research available:
• Visual-motor skills, such as eye-hand coordination, are associated with academic achievement. Scientists
have found that developing fine motor skills in early childhood can predict not only writing success, but
better performance in reading and math in elementary school. One study even showed how the
Handwriting Without Tears- Get Set for School (HWT-GSS) program improved fine motor and "pre-writing"
skills of Head Start students.
• Children who can write quickly and legibly are more likely to demonstrate skills in expressing their
thoughts through the written word. When kids struggle to write neatly and efficiently, they are often accused
of being lazy, and this may affect their behaviour and self-esteem.
• A study that followed children in grades two through five, demonstrated that printing, cursive writing, and
typing on a keyboard are all associated with distinct and separate brain patterns—and each result in a
distinct end product. For example, when these children were asked to come up with ideas for a
composition, the ones with better handwriting showed greater neural activation in areas associated with
working memory—and increased overall activation in the reading and writing networks.
• A Pew Research survey of teachers around the country found that today's digital technologies make
middle and high school students more likely to use informal language in formal papers and plagiarize.
• In high school years, kids who struggle with handwriting may also suffer even more as they struggle to
keep up with the volume of written work required.
• A 2012 review suggests that cursive may be particularly effective for individuals with developmental
dysgraphia (motor-control difficulties in forming letters) and that it may aid in preventing the reversal and
inversion of letters.
On the basis of your understanding of the passage, answer the questions given below. (1×10=10)
i. Based on your reading of the text, state the central theme of the passage. 1

ii. Select the option that displays what the writer projects, with reference to the
following: 1
Children who have good writing practice are more likely to demonstrate skills in expressing themselves.
a) caution
b) anticipation
c) denial
177
d) conviction

iii. In this digital age, where our communication is mostly done through electronic devices, the areas of
concern are: 1
1. Improving handwriting
2. Academic achievement
3. Improving performance in reading
4. Improving pencil grasp
5. Improving typing speed
6. Improving readability
a) 1, 4, 5, 6
b) 1, 4, 6
c) 1, 2, 3, 5
d) 1, 3, 5

iv. Complete the following with a phrase from paragraph 1. 1

Opinion Reason

Commands can now be verbal because of Voice Recognition Software

v. Read the statements given below and choose the option that correctly evaluates these statements.
1
1. Child’s struggle to write neatly.
2. Child is accused of being lazy.
3. Child’s self-esteem gets hurt.
a) 1 is the result of 2
b) 2 is the result of 1
c) 3 is the result of 1
d) 1 is the result of 3

vi. As per the text, “A 2012 review suggests that cursive may be particularly effective for individuals
with developmental dysgraphia.” Dysgraphia includes____________. 1

vii. Development of visual-motor skill is about: 1


1. feeling lethargic
2. academic achievement
3. writing success
4. better performance in reading
a) 1, 2, 3.
b) 1, 3, 4
c) 2, 3, 4
d) 1, 2, 4

viii. Based on your reading of the text, list two reasons the writer says how good handwriting helps us.
1
i) _________________________________
ii)_________________________________

ix. Children who_____________express themselves well. 1


a) write legibly
b) write cursive only
c) type fast
178
d) read well

x. Find the word from the passage that means ‘a person’s overall sense of
self-worth’. 1

II. Read the passage given below and answer the questions that follow. 10m
India is home to 18% of the global population but has only 4% of the global water resources. Its per capita
water availability is around 1,100 cubic meters (m3 ), well below the internationally recognised threshold of
water stress of 1,700 m3 per person, and dangerously close to the threshold for water scarcity of 1,000 m3
per person. Paradoxically, India is also the largest net exporter of virtual water (the amount of water
required to produce the products that India exports) and has one of the most water-intense economies.
Despite looming water scarcity, India is one of the largest water users per unit of GDP suggesting that the
way in which India manages its water resources accounts for much of its water woes. The problem lies with
the Government capacities which are lacking in improving water management, while policies and incentives
often favour inefficient and unproductive use of water. This coupled with weak or absent institutions (e.g.,
for water regulation) and poor data collection and assessment results in the increasing state of water woes
in the country. When it comes to improving water service delivery, India can learn
from Brazil, Colombia, Mozambique and New South Wales (Australia), among others. We don‘t have to go
overseas to see good examples of water resources management. The Maharashtra Water Resources
Regulatory Authority, established under a World Bank project, is putting in place policies, regulations,
institutions and incentives that promote more efficient and more productive use of water, e.g., by ensuring
the equitable distribution of water among users, and by establishing water tariffs. Efforts to establish
effective authorities are also underway in other states, and Maharashtra is disseminating the lessons
learned from its experience. Further, The World Bank‘s Country Partnership Framework for India
recognizes the importance of the efficient use of natural resources, including water, in support of the
country‘s ambitious growth targets. Several World Bank projects support India‘s efforts in the water sector:
Through the National Mission for Clean Ganga, the World Bank is helping the Government of India build
institutional capacity for the management and clean-up of the Ganga and investing to reduce pollution.
Another World Bank project, the Dam Rehabilitation and Improvement Project, has improved the safety and
performance of 223 dams in the country. The National Hydrology Project is providing significant support to
strengthen capacities, improve data monitoring and analysis, and laying the foundations for benchmarking
and performance-based water management. There are many more such projects that aid an efficient water
management system.

On the basis of your understanding of the passage, answer the questions given below. (1x10=10)

179
i. Infer one reason for the following, based on information given in the text. 1
India’s per capita water availability is about 1,100 cubic metres (m3).

ii. In the line, ‘‘internationally recognised household’’, the threshold DOES NOT
include 1
(a) the magnitude or intensity
(b) a point of entry
(c) margin
(d) a level, rate, or amount

iii. Find the word from the passage which means the same as ‘about to happen soon and causing worry’.
1

iv. According to the passage, Indian water woes are a result of_________________ 1
(a) lack of potable water
(b) lack of water management system
(c) over pollution in the water bodies
(d) excess usage and wastage of water

v. Based on your understanding of the passage, choose the option that lists the ways in which water can be
managed in India. 1
1. Regulation
2. Recycle
3. Incentives
4. Reuse
5. Policies
6. Treatment
(a) 1,4 and 6
(b) 2,3 and 5
(c) 1,3 and 5
(d) 2,4 and 6

vi. State the two reasons behind the water woes in the country are ever on rise. 1
i)_________________________________
ii)________________________________

vii. Based on your reading of the text, National Mission for clean Ganga is __________. 1
(a) A mission by the World bank for the management of the river Ganga.
(b) A mission by the Government of India in building institutional capacity for the management of the river
Ganga.
(c) A mission to build institutional capacity for the management and clean-up of the Ganga and investing to
reduce pollution.
(d) A mission for cleaning the river Ganga while reducing the pollution causing elements.

viii. How does the Maharashtra water resources regulatory authority serve as an example for the nation?
1
(a) By putting in place policies, regulations, institutions and incentives that promote more efficient use of
water.
(b) By limiting the use of water to certain hours of the day.
(c) By putting strict enforcement of laws against those who pollute or waste potable water.
(d) All of the above

ix. Despite looming capacity, India, being one of the largest water users per unit of GDP implying
__________________________________. 1

x. State whether the following statement is true or false. 1


Several World Bank projects could not able to support India’s efforts in the water sector as it is dangerously
close to the threshold for water scarcity of 1,000 m3 per person.
180
SECTION B -GRAMMAR
III. Attempt ANY TEN of the following questions. 10

i. Fill in the blank by choosing the correct option to complete the news update. 1

More and more people in the country _______________at the sun for cleaner energy options and rightly
so.
a. are looking
b. have looked
c. looking
d. have been looking

ii. Read the conversation between a son and his mother. Complete the sentence by reporting the patient’s
reply correctly. 1

Mother : Have you finished your homework ?


Son : Sorry mom, I fell asleep.

The mother asked the son if he had finished his homework. The son apologized and said
________________.

iii. Select the correct option to fill in the blanks for the given line, from a newspaper article.
1

The article read, “Most of the students opting for Science courses __________ need to include the scores
in Physics, Chemistry, Maths or Biology in their aggregate.

a) should
b) must
c) would
d) could

iv. Select the option that identifies the error and supplies the correction for the following line, from a news
report: 1
Delhi government announced a complete ban on the production, storage and use of some types of
firecrackers.

Option No. Error Correction

A. a the

B. and or

C. some any

D. on upon

v. Complete the given narrative, by filling in the blank with the correct option: 1
181
Can you hear that noise all the time? I don’t know how you __________ it.
a) are putting up with
b) put up with
c) have put up with
d) had put up with

vi. Fill in the blank by using the correct form of the word in the bracket, for the given portion of a letter:
1

Subject- Scarcity of water in our locality


Sir
Through the columns of your___________ (esteem) newspaper, I would like to draw the kind attention of
the concerned authorities towards the scarcity of water in our locality.
vii. Report the dialogue between a shopkeeper and Rohan by completing the sentence: 1

Shopkeeper: What do you want to buy?


Rohan: I want to buy a black wooden table.

In response to the question about his choice, Rohan says that _____________________.

viii. Identify the error in the given sentence, from a newspaper article and supply
the correction. 1

Exercise improves your overall health, and studies suggests that it has a larger effect on life expectancy
than body type.

Use the given format for your response.

Error Correction

ix. Father asked his son, Rajeev, about his health concern. Report the question of the father.
1
Did you consult the doctor today?

x. Fill in the blank by choosing the correct option, to complete the slogan. 1

Mighty tigers hear them roar, for they _________ be no more.

a) shall
b) may
c) might
d) could

xi. Select the correct option to complete the narration of the dialogue between Varun
and Anu. 1

Varun: How is your headache?


182
Anu: It has not improved. I am worried about it.
Varun: Oh! You must go and consult a good doctor.

Varun asked Anu how her headache was. To this Anu replied that it had not improved. Varun exclaimed
surprisingly and suggested that______________________________.

a) she must go and consult a good doctor


b) she might go and consult a good doctor
c) she may go and consult a good doctor
d) she must not delay and consult a good doctor

xii. Identify the error on a shop’s hoarding and write the appropriate correction in your answer sheet
1

Use the given format for your response.

Error Correction

IV. SECTION B- CREATIVE WRITING SKILLS 10

1. Attempt ANY ONE from A and B given below. 5

A. The data given below shows the percentage of student’s responses on aspects of mental health. Write
an analytical paragraph describing the chart in not more than 120 words.

183
OR

B. Your school has 4 buses each with a capacity of 50 students. The total number of students who actually
travel by these buses is 175. Another 150 students have applied for school transport. The school is not in a
position to buy new school buses.
What should school authorities do to ensure that school buses run to full capacity and the students in the
waiting list for school transport get it.
Write an analytical paragraph in about 150 words to briefly describe and to suggest some practical or viable
solutions to the problem.

2. Attempt ANY ONE from A and B given below. 5

A. Gauri saw the following information regarding an exciting discovery. She decides to write a letter to the
editor of a national daily about the advancement in science and its effect on our lives. Write the letter in 120
words using your own ideas.

OR

184
B. You had placed an online order for a pair of blue denim jeans with M/S David Jeans Co., Plot no-1134,
Midc Industrial Park, Bandra, Mumbai but have neither received the jeans nor any acknowledgement of the
order. Write a letter of complaint to the Customer Care Manager of the company complaining about the
non-delivery of the jeans. Seek a refund of the money paid if the company is unable to deliver the order in
five working days. Give necessary details of the order (colour, style and size of jeans, item code number,
price and proof of the credit card payment made for it).

SECTION C LITERATURE
V Reference to the Context 10

1. Attempt ANY ONE of two extracts given. 5

1.A He discovered the joys of being bowled over, tramped on and squashed every few minutes. He became
an accepted member of the gang, an unlikely, silky little object among the shaggy crew, fighting like a tiger
for his share at mealtimes and hunting rats in the old hen house at night. He had never had such a time in
his life. All the while, Mrs. Pumphrey hovered anxiously in the background, ringing a dozen times a day for
the latest bulletins.

i. Read the following statements, each of which describes the gist of the given extract. 1
Select the option that captures the essence of the extract correctly.
Statement I - It highlights the kind of comforts and luxuries that Tricki was used to at home.
Statement II - It brings out a contrast between Tricki and Mrs. Pumphrey’s state of being.
Statement III- It reflects that Tricki was happier at the surgery, and loved being with other dogs.
Statement IV- It shows Tricki’s journey with his peers at the surgery, and documents his recovery.
A) Statements I and II B) Statements III and IV
C) Statements I and III D) Statements II and IV

ii. In the above extract, the narrator describes that Tricki had never had such a time in his life. This is because
________________________. 1

iii. “All the while, Mrs. Pumphrey hovered anxiously in the background.”
Given below are different types of pet parenting styles described in Country Living, an e-magazine.
Choose the option that best reflects the kind of pet owner Mrs. Pumphrey was. 1
a) Traffic Light pet owners have a healthy balance of rules and freedom and give clear
and consistent signals for ‘yes’ and ‘no’.
b) Entranced pet owners have the best intentions, but as soon as their pet locks eyes
with them and gives their command, they are at their pet’s beck and call.
c) The Goose pet owners go all-out in protecting their pet. They often limit their time away
from their pets, especially puppies.
d) The Baggage Handler pet owners love being close to their pets and going on adventures
together. They are always mindful of the pet’s comfort and security.
A) Option (a) B) Option (b) C) Option (c) D) Option (d)

iv. Pick the option that reveals Tricki’s characteristics in the context of ‘fighting like a tiger for his share at
mealtimes and hunting rats in the old hen house at night.’ 1
(1) selfish (2) happy (3) greedy
(4) confident (5) sturdy (6) cruel
A) 2,4 and 5 B) only 2 C) 1 and 5 D) 3, 4 and 6

v. Why does the narrator describe Tricki being “tramped on and squashed” as joys? 1

OR

1.B There’s some demon of contradiction in you today, Ivan Vassilevitch. First you pretend that the Meadows
are yours; now, that Guess is better than Squeezer. I don’t like people who don’t say what they mean,
because you know perfectly well that Squeezer is a hundred times better than your silly Guess.

185
i. Select the option that correctly captures the usage of the word ‘pretend’ from line 1 of the extract.
1
A) I had no option but to pretend ignorance.
B) Raghav does not pretend to be a psychiatrist
C) This is a pretend train for the children to play in
D)Raman had a pretend pal with whom he talked
ii. Choose the option that lists the set of statements that are TRUE according to the given extract:
1
1. Ivan Vassilevitch had a quarrel with the speaker.
2. Ivan Vassilevitch claims that the Oxen Meadows are his.
3. Ivan Vassilevitch claims that Squeezer is a better dog
4. The speaker’s dog is Guess.
5. Ivan Vassilevitch and the speaker quarrelled more than once.
6. The speaker thinks Squeezer is a silly dog.
7. Ivan Vassilevitch agrees with the speaker on the quality of the two dogs.
A) 1,2,3 B) 1,2,5
C) 3,4,6,7 D) all of them

iii. Complete the analogy by selecting the suitable word from the text. 1
hero: villain :: angel:___________.

iv. Which of the following is the meaning of the word ‘contradiction’: 1


A) A combination of statements, ideas, or features which are opposed to one another.
B) A situation in which inconsistent elements are present.
C) The statement of a position opposite to one already made.
D) All of the above

v. State the two reasons for the quarrel between the speaker and Ivan Vassilevitch. 1
i) _________________________
ii) ________________________

2 Attempt ANY ONE of two extracts given. 5

2.A (I am Rapunzel, I have not a care;


life in a tower is tranquil and rare;
I’ll certainly never let down my bright hair!)
i. What is the poet’s tone in the extract? 1
a) analytical
b) despairing
c) peaceful
d) nervous

ii. Complete the sentence appropriately.


It is clear that Allusion is the poetic device used for ‘I am Rapunzel’ because _________. 1

iii. Read the statements A and B given below, and choose the option that correctly evaluates these
statements.
Statement A – The figure ‘I’ imagines a less than realistic view of being Rapunzel.
Statement B – The figure ‘I’ does not like the speaker. 1
a) A is true, B is false, according to the extract
b) A is true, B cannot be clearly inferred from the extract.
c) A cannot be clearly inferred from the extract, B is false.
d) A is true and can be inferred from the poem, B is true too.

iv. State whether the following statement is TRUE OR FALSE: 1


Amanda wants to be Rapunzel because she feels that life in the tower will be peaceful and unusual.

v. The reason behind the above extract written in parenthesis, i.e. () is ____________. 1
186
OR
2.B. The fog comes
on little cat feet.
It sits looking
over harbour and city
on silent haunches
and then moves on.
i. Carl Sandburg applies qualities of the cat to the fog. Select the most appropriate option from the ones given
below.
1. Cats are known to be faithful and loyal companions, protective of their masters and territory.
2. Cats are stealthy, moving in slow motion at times and they appear to be moving in a
mysterious fashion.
3. Cats like to move on at their own pace and before you know it, they’ve disappeared.
4. Cats are distrustful of strangers and can be jealous and moody. 1
a) option 1,2 and 3
b) option 3 and 4
c) option 2 and 3
d) option 1,3 and 4

ii. Choose the option that lists the most appropriate explanation of fog. 1

iii. The poet has used a poetic device in the given lines. What effect does she wish to create by its use?
1
The fog comes
on little cat feet.
It sits looking

A. mimicry
B. comparison
C. rhythm
D. humour

iv. Complete the sentence appropriately. It is clear that free verse is the poetic device used for the poem fog
because_____________________ (Clue: explain how personification applies here) 1

VI. Answer ANY FOUR of the following in about 40-50 words each. (4×3=12)
i. “Death and suffering are eternal truth; one must accept it as a part of life.” Reflect your views on the
statement with reference to ‘The Sermon at Benaras’. 3

187
ii. A paradoxical statement has been used by the poet describing the distinctive features of a chameleon.
Elucidate the statement with reference to the poem ‘How to Tell Wild Animals’. 3
iii. Black often symbolises misfortune. However, in the story, ‘The Black Aeroplane’, it depicts hope, faith and
gratitude. Discuss. 3
iv. Keeping a pet is not all fun but a responsibility too. Comment on the basis of the story ‘Mijbil the Otter’.
3
v. Validate the given statement with reference to the boy’s loss. ‘Sometimes we have to lose something
precious in order to gain something priceless’. (The Ball Poem) 3

VII. Answer ANY TWO of the following in about 40-50 words each. (2×3=6)
i. Presence of mind and intelligence is more powerful than a gun. How far is it true in the case of Ausable,
the secret agent? 3
ii. Money can’t make a man as much as education. Elucidate the statement with reference to the lesson ‘The
Thief’s Story’. 3
iii. Sending Bholi to school was a ‘blessing in disguise’. Discuss. 3

VIII. Answer ANY ONE of the following in about 100-120 words. (6×1=6)
i. Both Anne and Amanda were teenagers and sensitive girls. Analyze the common character traits of Anne
and Amanda with reference to the texts ‘From the Diary of Anne Frank’ and ‘Amanda’ respectively.
6
ii. Valli was so overcome with sadness to see the dead cow that she lost all enthusiasm. Compare and
contrast the plight of the Tiger captivated in the zoo, in the poem ‘A Tiger In The Zoo’ and the dead cow
roaming freely on the road 6

IX. Answer ANY ONE of the following in about 100-120 words. (6×1=6)
i. Horace was a successful thief because he carefully planned his robberies. Should we call him a successful
thief and still appreciate his work? Why or why not? 6

ii. “Confession is good for the soul.” A little confession would have changed the life of Loisel. Examine the
above given statement. 6

Sample Question Paper - 5


Class X Exam (2022-23)
English-Language and Literature (184)
Time allowed: 3 Hrs. Maximum Marks: 80

Section A: READING SKILLS


I. Read the passage given below:
(1) Happiness lies within the mind of an individual. No amount of external wealth may be helpful in making
him happy. Our forefathers had lifestyles based mainly on the concept of “simple living & high thinking”.
Excessive material wealth did not mean much to them.
(2) The structural framework of our forefather’s families was different from those of ours. The bond of
love which they shared cannot be easily found today. The family provided an emotional cushioning effect
against tension and stress. However, in today’s nuclear family, we are detached from feelings towards our
kith and kin. This may finally destroy the family psyche.
(3) The joint family system provided a proper environment for the child to grow up. The values of respect,
tolerance, responsibility, and integrity were internalized in the child. In the long run, they became better
human beings, compared to those in the present generation. Our forefathers felt happiest, if their children
became honest human beings. But today, we are happy only if we attain our coveted material ends and are
ready to resort to any means to attain that end.
(4) Our forefathers had a vision to make India the best. To attain their ambitions, they were ready to make
all sorts of personal sacrifices. On the other hand, today people are ready to migrate to the west, to enjoy a
comfortable life. Often, they become successful in foreign lands. But, in the process, they become alienated
from their motherland. Also, distance from their ailing parents is a worrying factor and keeps them perturbed.
It is not easy for them to return, as their children will not be able to adjust to the Indian environment and way
of life. Thus, this is a crisis and a frame of mind worse than their forefathers.

188
(5) In our generation, tradition and modernity have intermingled to form a special system. We are happier
than our forefathers in being able to lessen evils like the rigid caste system, untouchability, and child marriage,
but we have failed to totally eradicate them. I fact, electoral politics has made use of the caste system, through
the issuance of party tickets on the basis of caste, community, and religion.
On the basis of your understanding of the passage, answer questions that follow: 1 × 10 = 10
(a) How are our forefathers different from us?
(i) They had lifestyles based mainly on the concept of “simple living and high thinking”
(ii) They had different structural framework of families.
(iii) They made all sorts of personal sacrifices to attain their ambitions
(ivi) All of these
(b) What impact does a joint family system has on a child?
(i) It helps a child to become a better human being.
(ii) It internalizes the values of respect, tolerance, responsibility and integrity in a child
(iii) Both (i) & (ii)
(ivi) Neither (i) nor (ii)
(c) In our generation, ---------------- and ------------ have intermingled to form a special system.
(i) Untouchability, Intolerance
(ii) Tradition: modernity
(iii) Religion: caste
(iv) Politics; corruption
(d) based on your reading of the passage, choose the incorrect statement from the following.
(i) Our youngsters had a vision to make India the best
(ii) Happiness lies within the mind of an individual.
(iii) The family provided an emotional cushioning effect against tension and stress.
(iv) Our forefathers had lifestyles based mainly on the concept of simple living and high thinking.
(e) Choose the option that correctly states the two meanings of ‘cushioning’, as used in the passage
(A) Sharpening (B) Shielding (C) Intensifying (D) Softening (E) Irritating
(ii) A, E (ii) B, D (iii) C, E (iv) A, C
(f) The ------------ provided a proper environment for the child to grow up.
(i) Neighbourhood (ii) joint family system (iii) nuclear family system (iv) friends & relatives
(g) “In our generation, tradition and modernity have intermingled to form a special system.” Substitute the
underlined word with the most appropriate option from the following:
(i) Divided (ii) Blended (iii) Scattered (iv) Detached
(h) We are happier than our forefathers in being able to lessen:
(i) child marriage (ii) unemployment (iii) religious values (iv)none of these
(i) Select the option that makes the correct use of “sorts”, as used in the passage, to fill in the blank space.
(i) Policemen have to know all ----------- of different things.
(ii) The machine ------- it all out.
(iii) The computer -------- the words into alphabetical order.
(iv)He always ---------- the books according to the author’s name.
(j) What are the values internalized in the child?
(i) Respect (ii) Tolerance (iii) Responsibility (iv) All of these

II. Read the passage given below: (10 marks)

Marie was born in 1867 in Warsaw, Poland, where her father was a Professor of Physics. At an early age,
she displayed a brilliant mind and a blithe personality. Her great exuberance for learning prompted her to
continue with her studies after high school. She became disgruntled, however, when she learned that the
University of Warsaw was closed to women. Determined to receive a higher education, she defiantly left
Poland and in 1891, entered the Sorbonne, a French university, where she earned her master’s degree and
a doctorate in physics.

EVENT YEAR

189
Marie was born on the 7th of November 1867

Her sisters Bronya and Zosia became ill and Zosia passed away. 1876

Curie’s mother died from grief. 1878

She graduates high school. 1883

Leaves Warsaw for a governess position 1884

Her sister Bronya graduated medical school. 1891

Marie graduated with a master’s degree in Mathematics and first in her class for
Physics. 1893

She married Pierre Curie. 1895

Marie Curie was


a. The first woman with a degree in Physics.
b. The first woman to graduate in France.
c. The first woman to obtain a Nobel Prize.
d. The first woman to obtain a chair at the Sorbonne.
e. The first scientist to obtain two Nobel Prizes.

(2) Marie was fortunate to have studied at the Sorbonne with some of the greatest scientists of her days, one
of whom was Pierre Curie. Marie and Pierre were married in 1895 and spent many productive years working
together in the physics laboratory. A short time after they discovered radium, Pierre was killed by a horse-
drawn wagon in 1906. Marie was stunned by this horrible misfortune and endured heart-breaking anguish.
Despondently she recalled their close relationship and the joy that they had shared in scientific research. The
fact that she had two young daughters to raise by herself greatly increased her distress.
(3) Curie’s feeling of desolation finally began to fade when she was asked to succeed her husband as a
physics professor at the Sorbonne. She was the first woman to be given a professorship at the world-famous
university. In 1911, she received the Nobel Prize in chemistry for isolating radium. Although Marie Curie
eventually suffered a fatal illness from her long exposure to radium, she never became disillusioned about
her work. Regardless of the consequences, she had dedicated herself to science and to revealing the
mysteries of the physical world.
On the basis of your understanding of the passage, answer any ten questions from the twelve that follow: 1
× 10 = 10
Q1. In the line “…… she defiantly left Poland,” the word “defiantly” DOES NOT refer to
(a) Coldly (b) Challenging (c) Resistant (d) Openly
Q2. The events in the life of Marie Curie could reveal the following point:
(a) She was determined to receive higher education.

190
(b) She was eager for women empowerment.
(c) She had a brilliant mind set and rude behavior.
(d) She never liked her personal and professional life with Pierre Curie.
Q3. Marie …………………… by leaving Poland and travelling to France to enter the Sorbonne.
(a)challenged authority (b) showed intelligence (c) got happy (d) was distressed
Q4. Choose the option that lists the correct answers for the following:
(a) Ashwini, a physicist was working hard on her research on the black hole. She worked really very hard to
make her research a unique one. She did a lot of experimental work to come up with better inferences. What
kind of researcher is she?
(b) Lata was a lady of firm determination. She lost her husband after three years of marriage. She had two
daughters whom she raised all her own. She felt the void in her life but was never disappointed or
disheartened by that. What kind of lady is she?
(a) (1) disappointed and (2) experimental (b) (1) determined (2) independent
(c) (1) independent (2) determined (d) (1) diligent and (2) resolute
Q5. Based on your understanding of the passage, choose the option that lists the qualities of the professor.
(1) Scholar (2) Distressed (3) Exuberant (4) Lethargic (5) A bad student (6) Thoughtful
(a) (2) and (4) (b) (5) and (6) (c) (1) and (3) (d) (3) and (6)
Q6. Even though she became fatally ill by working with radium, Marie Curie was never…
(a) cheerful (b) enthusiastic (c) staunch (d) disillusioned
Q7. Marie became ‘aggrieved’ when she found that
(a) Her husband was dead. (b) The University of Warsaw was closed for women.
(c) She was ill. (d) She will have to leave her birthplace.
Q8. According to the passage, how long did the married life of Marie & Pierre Curie survive?
(a) 12 years (b) 10 years (c) 11 years (d) 15 years
Q9. Marie’s mother was …………………… and died after the death of …………………….
(a) heartbroken, her friends (b) depressed, two of her daughters
(c) heartbroken, one of her daughters (d) heartbroken, her cousin
Q10. Choose the option that lists statements that are NOT TRUE.
(a) Marie was brilliant in her studies.
(b) Marie was the first scientist to obtain two Nobel Prizes.
(c) Marie succeeded her husband Pierre at Sorbonne University.
(d) Marie didn’t reveal many facts and mysteries of the physical world.
SECTION -B
III Grammar (10 Marks)
i) Select the option that identifies the error and supplies the correction for the following line.
Sachin Tendulkar is a best cricketer in India.

Error Correction

ii) Read the conversation given below and complete the line that follows. (01 M)
Sujon: I want to be a doctor. What about you?
Rajib: I want to be an English teacher.
Sujon said that he wanted to be a doctor and, in turn, asked what about him. To this, Rajib replied saying
that ----------.
iii) Put the verb in bracket into correct form.
I___________ (not see) him since yesterday.
iv) Fill in the blank with suitable form of verb from the bracket.
He was tired and …….. asleep immediately (fell, fall)
v) Fill the blank with appropriate conjunction.
Not only he ……… his father was also present.
b) Is b) but c) can d) also
vi) Communication …… become more effective due to smart phones.
b) Is b) have c) has d) had
191
vii) The police ……. four thieves last night.
b) Catch b) caught c) is catching d) will catch
viii) My mother often says to me,‘Honesty is the best policy’.
My mother often says to me that ……………………….
ix) Rajiv …….. Karate class every day.
b) attends b) attending c) attended d) will attend
x) Select the option that identifies the error and supplies the correction for the following line.
Children are fond of coconuts.

Error Correction

IV. Creative Writing Skills (10 Marks)


(iii) You are Siddhartha Sinha, the principal of Vaidya Kanya Public school, Faridabad, Haryana, place
an order for school furniture in the form of a letter to Jasmeet Traders & Co. Sharma Market, Haryana (100-
120 words) (05 Marks)
(iv) The pie chart shows the proportion of people from different households living in poverty in the UK in
2002. Write an analytical paragraph to describe the information in 100-120 words.
(05 Marks)

SECTION - C LITERATURE (40 Marks)


V. Reference to the Context (5+5=10)
A. Read the extract given below and attempt it by answering the questions that follow. (5x1 = 5)
Well, it’s time I did some real work, I told myself; I ‘m out of practice. And if I don’t take the money, he’ll only
waste it on his friends. After all, he doesn’t even pay me.
(a) What is referred to as ‘real work’?
(ii) Writing articles (ii) cooking (iii) cleaning (iv) stealing
(b) What will he waste on his friend?
(i) Time (ii) Money (iii) Food (iv) All of these
(c) Hari wants to steal Anil’s money because
(i) He has to pay his debt
(ii) He has not stolen anything for a long time
(iiii) The money belongs to him
vi) none of these
(d) Anil is a --------- man.
(ii) Careful (ii) alcoholic (iii) careless (iv) hardworking
(e) Which word from the following is the antonym of “useful”
(ii) Waste (ii) pay (iii) take (iv) practice
B.Read the extract given below and attempt it by answering the questions that follow. (5x1 = 5)
An ultimate shaking grief fixes the boy
As he stands rigid, trembling, staring down
All his young days in the harbour where
His ball went. I would not intrude on him;
i. The poet uses the word ‘ultimate’ to describe the boy’s reaction. Pick the meaning that DOES NOT display
what ‘ultimate’ means in the context given.
a) Consequent b) final c) conclusive d) fateful
ii.The boy is very young in this poem. As a mature, balanced grown-up, he might look back and think that his
reaction of ‘ultimate shaking grief’ was
1) disproportionate to the loss.
2) pretension to procure a new toy.
3) according to his exposure and experience then.
4) a reaction to the failure of retrieving the toy.
5) justified and similar to what it would be currently.
192
a) 5 & 2 b) 1 & 3 c) 2 & 4 d) 3 & 5
iii. Pick the option that lists the boy’s thoughts, matching with the line-As he stands rigid, trembling, and
staring down.

a) Option 1 b) Option 2 c) Option 3 d) Option 4

iv. Why does the speaker choose not to intrude? This is so because the poet…
a) knows that it would embarrass the boy in his moment of grief.
b) feels that it’s important that the boy learn an important life lesson, undisturbed.
c) realises that he doesn’t have sufficient funds to purchase a new ball for the boy.
d) Experiences a sense of distress himself, by looking at the boy’s condition.

v. Choose the option that lists the meaning of “harbour’ as used in the extract.
Noun- (1) A place on the coast where ships may moor in shelter.
(2) A place of refuge
Verb- (3) keep (a thought or feeling, typically a negative one) in one’s mind, especially secretly.
(4) Shelter or hide (a criminal or wanted person)
(i) Option (1) (ii) Option (2) (iii) Option (3) (iv) Option (4)
VI. Short & very long Answer Questions. (30 Marks)
Answer ANY FOUR questions in 40-50 words each. (4x3=12)
a) Why was the young seagull alone at the ledge?
b) What does courage mean to Mandela?
c) How did Anne justify her being a chatterbox in her essay?
d) How did rain change? What happened to Lencho’s fields?
e) Describe the movement of the Tiger in the cage and in the wild.
f) Is Amanda an orphan? Why does she say so?
(vi) Answer ANY TWO questions in 40-50 words each. (2x3=06)
a) How is Ausable different from other secret agents?
b) Why is Mrs. Pumphrey worried about Tricki?
c) Why does Horace Danby steal every year?
d) Who is Fowler and what is his first authentic thrill of the day?
(vii) Answer ANY ONE of the following in 100-120 words. (6 Marks)
(a) It is truethat without the help of the members of family, the young seagull could not fly. Elaborate.
OR
(b) Our attitudetowards a situation evokes both negative and positive responses. Analyse this with
reference to the poem, “Dust of Snow” to bring out the inherent valuable lessons.

(viii) Answer ANY ONE of the following in 100-120 words. (6 Marks)


(a) In the story ‘A Questions of Trust’, Horace Danby carefully planned his theft, but was outwitted by
another thief ‘The lady in red’. Would you agree that over confidence may prove fatal one day? Discuss.

OR
(b) How did Herriot save Tricki’s life?

Marking Scheme SAMPLE QUESTION PAPER -1


Class X Exam (2022-23)
English-Language and Literature (184)
Time Allowed: 3 Hrs. Maximum Marks: 80

Section A: Reading Skills (Marks-20)


I. Read the passage given below (10 Marks)
I. Why does the writer say that mountains inspire ‘awe’ in humans? (Paragraph 1)
193
B. They evoke the wish in us, to master them.
ii. Select the option that corresponds to the following relation below:
The more incredible the mountains—the greater the thrill (Paragraph 1)
D. The heavier the luggage—the higher the penalty
iii. Select the option that displays what the writer projects, with reference to the following:
So, mountain climbing is undoubtedly one of the most popular adventure sports (Paragraph 1)
C. conviction
iv Complete the following with a phrase from paragraph 1.
hard to put into words.
V. The writer compares training to penance in the line --Then comes the penance of the rigorous training.
(Paragraph 2) State 1 point of similarity between training and penance.
very difficult / requires perseverance (Any other relevant)
VI. Based on your reading of the text, list 2 reasons why the writer says that “mountaineering is not a sport
that can be embraced without preparation”. (Paragraph 2)
Solution: Because mountaineering includes difficulties like having to walk on icy sheets that cannot be
accomplished without proper preparation of equipment
● Because mountaineering includes dealing with several health hazards that cannot be managed
without preparation.
● Because managing unpredictable weather is essential in mountaineering and cannot be
accomplished without being prepared with specific training.
● Because mountaineering presents the risk of fatality due to faulty decision-making and cannot be
addressed without being prepared by accompanying/ engaging experienced climbers. (Any other relevant/
correct from text).
VII. What connection does the writer draw out between unpredictable weather and the setting up of camps?
(Paragraph 3)
Solution: If the weather is unpredictable, it makes it difficult to decide when to set up camp as mountaineers
would prefer to climb when it is sunny and camp when it’s snowing.
VIII. The writer says, “A true mountaineer may challenge the mountain, yet is always respectful to the
powerful forces of nature.” (Paragraph 3) Select the reason the mountaineer is respectful to the forces of
nature, up in the mountains.
A. survival
IX. Supply 1 point to justify the following: While mountain-climbing, an impulsive mountaineer is either
disaster-prone or as good as dead
Solution: Survival is key in mountain climbing and it can be done with meticulously planning / careful
decision-making/ careful application of training (any one or more) It has no room for rash or impulsive
decisions/ actions— these would lead to accidents or fatalities.
X. Evaluate the INAPPROPRIATE reason for the feeling of exhilaration on reaching a summit, that the
mountain climbers experience.
D. Opportunity to use sophisticated mountaineering equipment
II. Read the passage given below. (10 Marks)
I. Infer one reason for the following, based on information in paragraph 1. The rate of tourism in the North-
East of India puzzles tourism officials.
Solution: This is so because these states are lesser explored as compared to the rest of the country, in
spite of having lots to offer.
ii. Select the appropriate options to fill in the blanks.
From paragraph 1 we can infer that the _____________ and _________________ of the North-Eastern
A. 1 & 3
iii. Complete the following analogy correctly with a word/ phrase from paragraph 1: aroma: cooking::
_________: painting
Solution: hues and shades
iv.Select the correct option to complete the sentence.
Travellers advocating the ‘experience economy’ seek a holiday package with ______________
B. a wholesome experience within the budget they have planned for.
V. Select the chart that appropriately represents the trend of foreign tourist travels in the North-East, from
2011-2014, as per paragraph 2.
B. Option 2
vi. Fill in the blank by selecting the correct option.
The study of tourist travel statistics in the North-East, from 2005 to 2014 showed ____________ results.

194
B. encouraging
VII. Substitute the word ‘witnessed’ with ONE WORD similar in meaning, in the following sentence
from paragraph 2: Foreign tourist arrivals in the Northeast witnessed a growth of...
Solution: observed/ recorded/ showed/ displayed (Any other similar relevant)
VIII. List any 2 examples of ‘tourist facilities as referred to, in Paragraph 3.
Solution: Accommodation—hotels, hostels, camps
Recreation—Parks, Gardens, Museums, Shopping areas
Essential- eating outlets, toilets, water points, and kiosks for
maps/currency exchange (if needed), certified travel guide availability
Transport-dedicated shuttle service, sight-seeing buses, cards or passes, car hiring stations
Digital upgrades –WiFi availability, websites, ticketing, forums
(Any other relevant)
IX. List one reason why the researchers recommend that the formulation of a tourism strategy in the North
Eastern States of India be sustainable.
Solution:
● To create socio-economic benefits for the local community
● To reduce the negative impacts caused on the destination/s
● To ensure the protection of culture and heritage/ To ensure minimal intervention in the cultural
aspects
● To protect natural wildlife and resources (Any other relevant)
Section B –Grammar (10 Marks)
Attempt ANY TEN of the following questions.
I. Fill in the blank by choosing the correct option to complete an online update.
The climate control comment by an activist on social media yesterday.
B. blew up
ii Read the conversation between a doctor and his patient. Complete the sentence by reporting the patient’s
reply correctly.
The doctor, while trying to figure out his patient’s ailment, asked about his well-being,
to which, the patient affirmed……………..
Solution: ii. that he does not stay in a good mood. (Addition of sometimes/time to time acceptable)
iii Select the correct option to fill in the blank for the given line, from a health magazine.
The advertisement read, ‘If you smoke, statistically your story end 15% before it should.
C. will
iv Select the option that identifies the error and supplies the correction for the following line, from a news
report:
Last week a child was not allowed to board the plane at Ranchi airport.
iv. C. the/a
V. Complete the given narrative, by filling in the blank with the correct option:
As I was standing on the dock, looking out at the lake for the last time, a feeling of emptiness was over me
like darkness.
D. washed
VI. Fill in the blank by using the correct form of the word in the bracket, for the given portion of a letter:
Subject: Request for Approval
Dear Sir
This is to respectfully submit that I……………(seek) approval for organizing a tree
plantation drive to be undertaken by the club.
vi. seek
vii Report the dialogue between a grandson and his grandfather, by completing the sentence:
In response to the question about his superheroes, grandfather says that………….
vii. it is anyone who shows kindness and compassion to others
viii Identify the error in the given sentence, from a school magazine report and supply the correction.
In order to balancing the sentiments of the Eagles and the Hawks, the Student Council suggested a
rematch between the teams.
Error Correction
balancing balance
ix Sunil shared some information, with Tariq, about a holiday at sea. Report Tariq’s question: Did you enjoy
traveling by sea?
Solution: Tariq asked Sunil if/whether he had enjoyed traveling by sea.

195
x. Fill in the blank by choosing the correct option, to complete the slogan by the ministry for Child Welfare.
……………WE AFFORD TO NEGLECT CHILDREN? THINK TWICE!!
D. CAN
xi. Select the correct option to complete the narration of the dialogue between Latha and her father.
Father asked Latha the reason for the many questions she was asking. Latha exclaimed
good-humouredly that in event of not knowing the answer one should………….……..
A. keep asking till one does.
xii . Identify the error on a shop’s hoarding and supply the correction, for the following sales offer:
Gumnaam & Daughters Pvt. Ltd.
Bindapur, Jharkhand
Massive discount for all senior citizen vaccinated with the precautionary dose.

Solution:
Error Correction
all each

Section B –Creative Writing Skills ( Mark – 10)


1. Attempt ANY ONE from A and B given below.
A. You are Sunidhi Prakash, the Vice Captain of Brilliant Vidyalaya, Barra, Kanpur. You have recently
noticed several posters around your school premises conveying the hazardous message: Lose weight in
just a month! “A WONDER DIET comes to your rescue … A privilege available for only a few!” Write a letter
to the Editor of The DWA, in not more than 120 words, drawing attention to harm caused by such
advertising. Propose the implementation of “Wholesome Lunch Month’ in schools as an idea to address
such practices, mention the advantages and share suggestions to foster healthy eating routines and
develop positive body image among youngsters.

Solution:
Brilliant Vidyalaya, BarraKanpur

29 August 2022

The Editor
The DWA
K-21, Anjana Pura
Kanpur

Subject: Need for Promoting Healthy Eating Routines

Dear Madam
This is with reference to posters bearing the message of ‘crash diet’ being posted around our school
premises. Such posters impact youngsters negatively and can be hazardous to their self-esteem. I would
like to propose the idea of implementing “Wholesome Lunch Month’ for all school students to counter the
implications of such misleading advertisements. This initiative is sure to encourage all students to bring
nutritious and healthy lunches daily and develop healthy eating routines. To ensure that this project gains
strength, schools may organize puppet shows, street plays, and Ted Talks (by Nutritionist/ Psychologist),
encompassing the theme, to foster healthy eating routines and a positive body image. I hope that the
publishing of my letter in the columns of your renowned Daily helps spread awareness and promote a
healthy lifestyle among students.

Thank you
Yours truly
Sunidhi Prakash
Vice-Captain

OR
B. You are Zac Skaria, a resident of # 412, Magna Greens Apartments, Gandhi Marg, Jonpara, Mumbai.
Three students of grade 10 from your residential complex have rescued and rehabilitated a few old beggars
from the neighborhood. You think that their work deserves appreciation and recognition. Write a letter to the
196
President of the RWA, seeking recommendations for these youth, to be nominated for the ‘Serving Citizens’
Award’. Suggest other ways such acts of kindness could be recognized and awarded in the future.
Solution:
# 421, Magna Greens Apartments
Gandhi Marg, Jonpara
Mumbai
19 July 2022

The President
RWA, Magna Greens Apartments
42, Gandhi Marg, Jonpara
Mumbai

Subject: Seeking Recommendation for ‘Serving Citizens’ Award’ Nominations.

Dear Sir
This is with reference to the empathetic social service done by Miss Jiya, Mas Adwait, and Master Pranit
(residents of our complex), in rescuing and rehabilitating a few old beggars from our neighborhood. These
students ensured that the beggars were rehabilitated at ‘Seva Sadan’- an NGO which takes care of the
needy of our city. This selfless initiative carried out with dedication and responsibility deserves due
recognition. I, therefore, request you to issue a letter of recommendation for these students to be
nominated for the ‘Serving Citizens’ Award’ organized by the local Municipal Corporation.
I would also like to submit that the RWA set up a special committee that looks into such acts in the future.
This would aid dedicated attention and appropriate screening of nominations. Arrangement of academic
sponsorships for such children would also be an encouraging gesture.
I entreat you to address this at your earliest convenience and issue the recommendation letters.

Thank you
Yours sincerely
Zac Skaria
(Designation)

2. Attempt ANY ONE from A and B given below.


A. Gurmeet Kaur is an aspiring candidate for a public-funded engineering college in the suburbs.
She belongs to a nearby village, has minimal technical skills and exposure, has the required cut-off
percentage, and is looking for a complete or partial scholarship. Write a paragraph in about 100-120 words,
analyzing her SWOT notes to support your stand on whether she should join/not join the college.
Solution: In support of the decision:
The given information illustrates the options Gurmeet would weigh in order to take the right decision about
her admission in a public-funded engineering college. With availability of strong curriculum, quality faculty
and vibrant Activity Clubs, she will be assured of an enriching educational journey. Though hostel facility is
unavailable, she may take up accommodation in the suburbs or choose to travel daily from her village.
Good opportunities of practice-based research, partnership with professional companies and international
students exchange program will enhance her professional and interpersonal skills. Hard work, responsible
behaviour and prudent decision-making could help Gurmeet thrive in the college even though it has a rigid
and conventional culture.The strengths and opportunities work in favour of Gurmeet. Taking this
opportunity will allow her to mend the incorrect public perception towards public-funded colleges and
students’ outlook towards technical subjects. (129 words)
Against the decision:
The given information indicates Gurmeet’s dilemma about seeking admission in a public- funded college.
Though equipped with a strong curriculum, engaging activity clubs and an able faculty, the college lacks
diversity and good conduct among the students,giving rise to concerns of safe environment for a novice like
Gurmeet. Underutilization of IT services will be further detrimental to the progress of Gurmeet’s educational
journey and add to extra costs related to research work. If she does not qualify for a complete or partial
scholarship, arrangement of own accommodation and additional expense of students exchange
programme will increase her expenditure, too. Such a college environmentmay dampen her endeavouring
spirit. The weakness and threats outweigh the strengths, in case of Gurmeet. So, it is recommended that
Gurmeet does not apply for admission to the said college. (139 words)
197
B. Read the following excerpt from an online post of a website on educational practices.
Kids who appreciate how much effort, time and care goes into growing food will understand how important
farmers are, and why it's important to take care of our Earth. In the world of today, gardening needs to be
given more importance than sports, music and dance in all schools because it creates environmental
stewards and outdoor learning laboratories that help the child and community for years to come. Write a
paragraph in 100-120 words to analyse the given argument. You could think about what alternative
explanations might weaken the given conclusion and include rationale / evidence that would strengthen /
counter the given argument.
Solution: Argument FOR the subject of the statement:
In the world of today, gardening needs to be given more importance than sports, music and dance in all
schools. While sports, music and dance contribute towards personal growth, the current times mandate
attention towards an issue that is global ---nature and natural processes. With growing food wastage in
many homes today and the urban young believing that vegetables are grown, harvested at the super
markets, the efforts of the farmers are discredited. Gardening at school will open a world of first-hand
learning experiences of sowing, watering and harvesting processes. Waiting for the saplings to grow will
inculcate sensitivity, patience, empathy, gratitudeand value for one’s hard work. They will feel accountable
for their piece of Mother Earth, resulting in making them efficient and enterprising environmental stewards.
Unlike sports, music or dance, gardening goes beyond just enjoyment to create aware and responsible
citizens of the future. (148 words)
Argument AGAINST the subject of the statement:
Gardening,certainly, should not be given precedence over sports, music and dance in all
schools.Gardening at school requires good planning with hands-on guidance and continued supervision by
the teachers. Students tend to lose interest due to the slow and natural growth progress of plants as well as
the investment of continuous hard work.Small targets or goals would be missing whereas the danger of
destruction of their work due to rains, intrusion of grazing animals or a pest attack is like to set in a feeling
of defeat. It may be noted that sports, music and dance are uplifting activities which display faster results,
are enjoyable and inculcate team spirit, collaboration and confidence. Unlike gardening, setbacks in these
activities can be addressed with some sense of personal control. These activities help students express
and de-stress successfully. So, maintaining their due importance in the school’s co-curriculum is
imperative.

Section C - Literature (Marks - 40)


Attempt ANY ONE of the two extracts given. (5 marks)
I. Why was Pranjol not as excited as Rajvir about the tea gardens?
C. He had grown up in and around tea gardens.
II. What does Pranjol mean by saying that Assam has the largest concentration of plantations in the world?
Solution: ii. ...that the cultivation/harvesting (or any similar suitable word) (of tea) is the highest in one place
namely Assam.
III. Fill in the blank with ONE WORD only.
Pranjol ……………. comes through clearly when he exclaims, “You will see enough gardens to last you a
lifetime!”
Solution: iii. Frustration/irritation/ exasperation (or any suitable word) compatible with the exclamation mark
in the sentence.
IV. How according to Rajvir does the world know about the discovery of tea?
B. Traditional tales
v. Select the option that correctly captures the application of the word ‘cried’ as used in line 1 of the extract.
B. Jaspreet cried out loud when she saw a white tiger in the sanctuary.
OR
B OMEGA:
(The Book that Saved the Earth)
i. Select the option that correctly captures the usage of the word ‘present’ from line 1 of the extract.
B. Iota needs to present his opinion firmly.
ii. Complete the analogy by selecting the suitable word from the text frown: smile:: gloomily:……………
Solution: ii. Frown: smile: gloomily: brightly
iii. Select the option that displays the reason why all crew members were asked to have vitamins. In order
to -
D. accomplish a specific task.

198
iv. According to the extract, what did THINK-TANK most likely want OMEGA to do when he said
‘Transcribe…’?
B. 2 & 4
v. The playwright places certain words and sentences in brackets in the given extract.
Solution:
● Help actors and director gain clarity about the emotions and gestures required while performing
/directing
● Helps with understanding stage setting and movements (Or any other suitable explanation) (accept
any two for full 1 mark)
2. Attempt ANY ONE of the two extracts given. (5 marks)
i. Complete the sentence appropriately.
Solution: i. ...the sun, which is non-human, is attributed to the human feature of having feet. (Accept any
synonyms giving a similar/ correct meaning)
ii. The poet has used a poetic device in the given lines. What effect does she wish to create by its use?
…no bird could sit no insect hide no sun…
A. emphasis
iii. State whether the following statement is TRUE or FALSE:
Solution: iii. FALSE
iv. Select the appropriate option to complete the sentence, according to the extract. The idea of a forest
that has been ‘empty all these days’ is.
A. unnatural
V. How does the use of enjambment impact this extract?
C. It builds momentum.
OR
i. What is the poet’s tone in the extract?
C. 2, 4
ii. What causes the young men to ‘despair’, according to the extract?
Solution: ii. Being hopelessly in love / the uncertain in love/ Unsure of the return of their affections
iii. Identify the reason for the speaker’s need to color her hair, as per the extract.
D. Her conviction that she is beautiful inside
iv. Complete the analogy about the speaker’s hair.
Solution: yellow: blonde:: orange /red : carrot (either one can be accepted for full one mark)
v. Select the sentence in which the word ‘set’ is used in a similar manner as line 2 of the extract.
C. The dessert needs to set for two hours before being served.
VI Answer ANY FOUR of the following in about 40-50 words each. (3x4=12 marks)
i. Validate the given statement with reference to baby seagull’s fear. ‘Fear doesn’t exist anywhere else
other than one’s mind.’ (His First Flight- Two Stories about Flying)
Solution:
● The baby seagull could not take his first flight as he was scared that his wings will not support his
body weight
● Saw his family fly –Yet, the fear of falling down gripped his mind—it overpowered any sense of
assurance or example
Result—was too scared to even try
II. Explain why the poet personally holds the conviction that the world will primarily end in fire? (Fire and
Ice)
Solution:
● He was a victim of the fiery aspect of desire.
● By his own admission, (From what I’ve tasted ……) he had experienced its destructive effects in his
life.
iii. Valli’s unique maiden bus ride experience could be possible because she belonged to a small village. Do
you agree? Why? /Why not? (2 reasons) (Madam Rides a Bus)
Solution:
Agree:
● A bus ride seemed like a fascinating means of recreation and adventure ---unlike cities and bigger
towns.
● She could travel alone safely –unfortunately, not recommended in larger townships or cities.
● There was only one bus that Valli observed several times – cities have varied means of transport
that might seem more adventurous

199
Disagree:
● The fascination for riding a bus or an automobile can exist in children of Valli’s age even in big
cities.
● Traveling unnoticed is easier in large cities than in small towns or villages due to familiarity
● Cities would offer more opportunities for a bus ride due to the availability and frequency of several
buses on the same route.
iv. Give one reason why ‘The Tale of Custard the Dragon’ is more a fable than a ballad.
Solution:
● A fable is a fictitious narrative usually with animals, birds, etc as characters, and shares a strong
message whereas a ballad is a narrative verse that can be silly or heroic.
● The Tale of Custard the Dragon includes animals, is surely fictitious, and shares a meaningful
message. Hence, better qualifies as a fable.
v. How can we say that Natalya was continuously successful in maintaining an upper hand during her
arguments with Lomov? (Any one example) (The Proposal)
Solution: Upper hand –
● She was able to answer every query and present an argument defeating the one presented by
Lomov.
Arguments--
● Argument about ownership of Oxen meadows –Natalya argued that it is a matter of principle and
not greed.
● She showed conviction and belief while arguing.
OR
● Argument about dogs – Natalya argued that her dog was cheaper, was of better breed and could
run faster.
● Never lost cool while presenting her arguments.
VII Answer ANY TWO of the following in about 40-50 words each. (2x3=6 marks)
i. Dr. Herriot knew his patients as well as their owners really well. Discuss. (The Triumph of Surgery)
Solution: Patients --Dr. Herriot could understand the problems of his patients (dogs) just be observing-- saw
Tricki in the market and understood that the dog requires help.
● Owners--He understood the owner (Mrs. Pumphrey) well and never spoke any harsh and advising
words on the seriously obese dog- knowing fully well that she was responsible for this condition
ii. State one likely reason the writer of The Midnight Visitor chose to characterise Ausable as short and fat.
Solution:
● Ausable-- characterised as short and fat as the writer wants to draw attention to and emphasise his
wits and mental ability to handle any grave situation [Ausable was able to successfully plant a story of a
non-existent balcony and also made the agile and smart Max believe it ---didn’t require an attractive
physique to do this.]
● The writer, perhaps, wanted to give a strong message that the brain is what counts more than
brawn/ muscle power OR brain power is far more potent/effective than any other type of power
muscle/arms etc.
iii. Validate the importance of small, fun learning tasks towards successful careers, in the context of Richard
Ebright in The Making of a Scientist.
Solution:
● Started the collection and breeding of butterflies as a fun activity.
● Got curious about the gold spots and the secretion from them.
● Was able to develop a theory of cell structure and later DNA because of it.
● Fun learning resulted in a career as a renowned successful scientist.
VIII Answer ANY ONE of the following in about 100-120 words. (6 marks)
i. Mijbil and the Tiger, both were looked after by humans. Assume they both meet each other in the zoo and
have a conversation about their lifestyle and feelings. Write this conversation as per your understanding of
Mijbil the Otter and A Tiger in the Zoo. You may begin like this Tiger: Thanks for visiting me, though I don’t
usually like visitors. Mijbil: Oh? I would love visitors, I think.
Solution: Mijbil-
● Happy about his life with his owner as a pet—perhaps would be scared to be out in the wild –
wouldn’t know how to survive
● Gets to play/go for walks.
● Enjoys meals and company of the owner – exists peacefully in the company of his owner
Tiger

200
● Unhappy in captivity-- blames humans for caging him—wished they’d know the value of peaceful
coexistence.
● Longing to be free and in natural habitat –perhaps wouldn’t wish to be tamed as a pet.
● Gets disturbed by the human activity
OR
ii. “Not from weeping nor from grieving will anyone obtain peace of mind’. If you had to use the message of
the given quote from the Buddha’s sermon (The Sermon at Benares) to help the boy cope with the loss of
his ball and what it signifies (The Ball Poem), what would you include in your advice? Also, evaluate why it
might be difficult for him to understand the notion.
Solution: The learning from the referenced quote of Buddha--
● the loss of irreplaceable things brings grief and sorrow.
● learning to stay calm and understanding the perishable/mortal nature of things helps in living life
normally and forgetting loss.
To help the boy cope with the loss --- that loss is an important part of life –important to learn from
experience –adapt and move on.
Difficult for the boy to understand the notion--- The boy is too young to understand the depth of these
words-- is alone in his loss has no one to explain and must learn from his experience painstakingly--
requires time to cope ---easy to feel disheartened at that age.
IX Answer ANY ONE of the following in about 100-120 words. (6 marks)
i. Fiction writers prefer creating grey characters rather than black and white. Analyze this in detail, with
reference to both the characters of The Thief’s Story
Solution: Introductory – Both characters in the story—neither completely black (negative) nor white
(ideal)—have redeeming qualities as well as those that need improvement—the story reveals both their
personas as grey.
Hari Singh • Thief—artful - too smart for his age - fooled his victims and police --Wasn’t ashamed in lying
and stealing –made no efforts to confess that he had given in to the temptation of stealing the notes,
However, • Changed due to love, affection and Anil’s trust The character of Hari Singh—not a flat character
who is either black or white.
Anil -While we can admire Anil for his qualities --helpful and simple --easily trusting --forgave the thief --
never uttered a word despite knowledge of the theft, However, Not perfect--casual about money --ignored
Hari Singh’s pilfering The character of Hari Singh—not a flat character who is either black or white.
Conclusion—Grey characters present a challenge to the readers, allow a sense of unpredictability and
present a moral challenge. Hari Singh and Anil are created as grey characters.
OR
ii. ‘Honor among thieves’ is considered a popular code. Examine ‘A Question of Trust’ as a story woven
around this code.
Solution: Honor among thieves---It is believed that thieves never double cross each other/ commit crimes
against each other—would be inclined to help rather than betray each other
The protagonist, Horace Danby didn’t anticipate the role of the lady in red.was careful and meticulous in his
theft which he conducted once a year
The Lady in red made a fool of him -- in spite of being a thief herself, she did not follow the code of honor
normally existing between two thieves This resulted in Horace Danby going to prison for the first time in his
life---he felt betrayed.

Marking Scheme SAMPLE QUESTION PAPER -2


Class X Exam (2022-23)
English-Language and Literature (184)
QI.

QII.

Section-B (Grammar)
(i) Option (A) is correct

201
(ii) that he was going
(iii) error- there; correction- here
(iv) Option (A) is correct.
(v) Option (B) is correct.
(vi) he was not in his office
(vii) (vii) decided
(viii) he would have to pay only six lakhs
(ix) (ix) Option (A) is correct.
(x) error- express; correction- expressed
(xi) to wait till his class 12 board results.
(xii) (xii) Option (D) is correct.
Section-B: Grammar
IV. Section-C: Creative Writing Skills

1.A. [Note: No marks to be awarded if only the format is given. Credit should be given to the candidate's
creativity in the presentation of the ideas, appropriate use of language and skills of reasoning.]
Letter to place an order
Format: (Sender's address, 2. Date, 3. Receiver's address, 4. Subject Heading, 5. Salutation, 6.
Complimentary Close)
Detailed Answer:
Army Public School
New Delhi

15 September, 20XX

The Sales Manager


Apple Inc.
Nehru Place
New Delhi
Subject: Placing an order for computers and accessories
Sir
Through this letter of mine, I wish to inform you that our school wishes to buy some computers and
accessories from your showroom for our school computer lab. The details are as follows:
S. No. Name of the item Quantity
1 Computer 20
2 Mouse 10
3 UPS 20
4 Speaker 20 pairs
5 Keyboard 20
6 USB Cable 20
7 Printer 05
Since it is a bulk order, we expect a reasonable discount from your end. Please ensure that all the
computers and the accessories are in good condition and free from all defects. Please arrange to send
everything well packed, preferably in bubble-packing. We expect the goods to reach us by the end of this
month. An early action from your end will be highly appreciated.
Thank you
Yours sincerely
Nihal Singh
(Computer In charge)
OR
B. 3-B, DDA Flats, Punjabi Bagh, Delhi
29 August, 20XX
The Editor i-Next
Delhi
Subject: Awareness to Keep Metro Clean
Sir
Through the columns of your esteemed newspaper, first of all, I would like to thank the DMRC, Delhi for
providing the citizens of Delhi with so much ease in commuting. I frequently use the Metro and find it very
202
convenient. While travelling, I have noticed that though there is strict vigilance, still people deface the Metro
trains and stations. I feel that if the DMRC, Delhi collaborates with the Directorate of Education and runs
awareness programmes in schools, we would be able to cover a wide spectrum of people and spread the
message. This had been successful in the past like for the pollution-free Diwali campaign, as well. It would
be a great service to the city if you address this issue in your daily.
Thank you
Yours sincerely
Sweety
2. A. The line graph given above clearly depicts the number of people who visited Salarjung Museum in
Hyderabad between 2000 and 2010. In 2000 the number of students visiting the museum was 2.75 million
as compared to domestic visitors which was a little less than a million in comparison to overseas visitors as
their number touched 8 million in 2010 which earlier, i.e., in 2000 was 3.75 million. The line graph shows a
steady rise in the number of overseas visitors but as far as domestic visitors are concerned, the number
increased by only one million in 2010. It might rise to 4 million in 2021 whereas almost 12 million visitors
are expected by the end of 2020. The total number of visitors kept increasing from 8.5 million in 2000 to
12.5 million in 2005 and 19 million in 2015. The number of total visitors is expected to touch the scale of 23
million by 2020. As many as 7 million students are expected to visit it by 2020 as compared to their number
of 5 million in the year 2015
OR
B) A survey was conducted in Bilaspur to find out the changes that occurred between 1995 and 2010
regarding women's education. It was found that there was a rise in the number of girl's schools. Earlier, in
1995 there were only 10 schools. A slight increase was seen in the number which rose to 15 in 2010. Not
much growth was recorded in the number of women's colleges which rose from three in 1995 to four in
2010. However, significant growth was recorded in the number of students in school which increased from
10,000 in 1995 to 16,000 in 2010. There was a sharp increase of 4000 students in colleges also. The
percentage of literacy rate increased steadily from 30 to 55 in these years. There were no Distance
Education Centres at the beginning of the period. However, three such centres were found by the end of
2010. We can conclude by saying that an overall increasing trend was seen in women's education.
Section-D: Literature
V. 1.A.
(i)Option (C) is correct.
(i) Option (B) is correct.
Explanation: My aunt's grandmother gave the meadows for the temporary and free use of your
grandfather's peasants.
(iii) Option (A) is correct.
Explanation: The peasants used the land for forty years.
(iv) Option (D) is correct.
(v) Option B) is correct.
Explanation: The Oxen Meadows were in dispute.
OR
B. (i) Option (B) is correct.
Explanation: Lencho showed not the slightest surprise on seeing the money; such was his confidence.
(ii) Option (C) is correct.
Explanation: The postmaster and his employees could manage just a little more than half the amount asked
by Lencho from God.
(jii) Option (A) is correct.
Explanation: Lencho did not show the slightest offence towards God. "God could not have made a mistake,
nor could he have denied Lencho what he had requested."
(iv) Option (C) is correct.
Explanation: It was the postman himself who handed the letter to him.
(v) Option (C) is correct.
2. A. (i) Option (C) is correct.
Explanation: It was the pirate holding a pistol in each hand.
(ii) Option (B) is correct.
Explanation: He had a black beard.
(iii) Option (C) is correct.
Explanation: He had a wooden leg. He was lame.
(iv) Option (A) is correct.

203
(v) Option (C) is correct.
Explanation: Ogden Nash wrote The Tale of Custard the Dragon.
OR
B. (i) Option (A) is correct.
Explanation: He is passing through long grass.
(il) Option (C) is correct.
Explanation: Tiger loves to eat deer. It is his favourite food.
(iii) Option (D) is correct.
Explanation: The poem is A Tiger in the Zoo.
(iv) Option (B) is correct.
(v) Option (B) is correct.
Explanation: alliteration which means words of some sound used. In this line, those words are 'plump' and
'pass', both having the 'p' sound.
VI. (i) In the poem 'Fire and Ice' the poet considers the age-old question of whether the world will end in fire
or in ice. This is similar to another age-old question of whether it would be preferable to freeze to death or
burn to death. Either option would achieve its purpose sufficiently, may be in storm of volcanic eruptions,
forest fires or melting ice caps
(ii) The details that Valli picked up were about the distance that the bus travelled, the duration and the fare.
She gathered that information by hearing the conversation of others and by asking a discreet question or
two.
(iii) According to Anne Frank paper has more patience than people as people are not always interested in
listening to what you are telling them, they get bored and lose patience but it is not so with paper. One can
go on writing for as long as one likes and whatever one wishes to write.
(iv)

Detailed answer: Lencho's entire crop was destroyed in the hailstorm, but he had a deep faith in God and
He was his only hope. Therefore, he wrote a letter to God asking for monetary help so that he could sow
his field again and support his family.
(v) Prince Siddhartha Gautama was deeply pained by the sufferings he saw around him and left the house
to seek the truth of life. After wandering for seven years, he finally sat under a Peepal tree to meditate till he
received the enlightenment. Wisdom of the law that governs the cycle of birth and death dawned on him
and 'The Buddha' set out to share it with the world to relieve the mortals of their sufferings.
VII. (i) Richard Ebright had some innate character traits that are the prerequisites for the making of a
scientist. He had, besides an intelligent mind, the curiosity to seek information, keen observing powers,
perseverance, patience as well as self-discipline, which are the basic requirements for any scientific
research. And the credit also goes to his mother for recognising his inclinations and curiosity. She not only
encouraged and supported him but also provided the required stimulus through books and other materials.
(ii) Dr Herriot was shocked to see Tricki's condition. He knew that the latter would soon be hospitalised as
he was unable to move. He had become hugely fat, like a bloated sausage and his tongue lolled from his
jaws. He understood the very cause of Tricki's condition and knew that the situation will not be changing
any time soon.
(iii)

Detailed answer: Matilda Loisel was very pretty. She considered herself to be born for a luxurious life but
the reality was different from her dreams. In reality, she was married to a petty clerk and had to lead a very
simple life. Her fantasy over ambitiousness made her unhappy. Matilda's desire to belong to the rich and
affluent section of society was responsible for her downfall. If she had accepted her condition with a smile
and without any complaints, she would not have had to live 10 long years of her life in such abject poverty.
VIII. (i) To train the young seagull in the art of flying, the entire family helped out. His sister and brothers
encouraged him to muster up the courage and try to fly. The parents called out in a shrill voice, along with
cajoling and scolding him in turn. Then they threatened to let him starve. When all this proved
unsuccessful, his mother tempted him by flying close to his ledge with a piece of fish in her mouth.

204
Maddened by hunger, he dived forward to get that piece and fell into the space. To save himself, he spread
his wings and found that he was able to fly. Thus, the entire family helped him to learn the art of flying. His
parents and his brothers and sister had landed on the green surface of water ahead of him. They were
calling him to do the same thing. They were flying around him. They were roaring and diving. When he was
tired and weak with hunger, he could not rise. Then his family came around him crying and praising him.
They offered him scraps of dog-fish because he had made his first flight successfully.
(ii) Rajvir had been invited to spend his vacations with his classmate Pranjol, whose father was the
manager of a tea estate in Assam. He was quite excited about the trip and had gathered a considerable
amount of relevant information, i.e., where tea originated and how it became such a popular beverage. Re-
informed that about 80 crore cups of tea are consumed daily and that the name 'tea' itself has originated
from the Chinese language. He narrated two popular legends about the origin of tea.
Rajvir exhibits an important value trait, which is the pre-requisite to gain knowledge - curiosity and
eagerness to know. Moreover, he is not a passive learner but is proactive in collecting prior information to
any real experience. He likes to do his homework thoroughly before going to a place. This is something
very appreciable and children should develop this habit of finding out about the place they are going to visit.
These are extremely desirable traits for all learners.
IX. (i) The thief befriended Anil because he wanted to have an acquaintance with him so as to make him his
next victim. Moreover, he needed a place to live. He started to work for Anil who was a credulous person.
Anil started teaching him how to read and write. One day Anil brought a bundle of notes. The thief stole the
money though he was out of practice and had not robbed anyone for so many days. After stealing the
money, he planned to go to Lucknow. But at the station, his inner conscience pricked him and did not allow
him to go. His heart changed as he wanted to be an educated man. So, he returned back and placed the
money back in its place. Hari was grateful to Anil because Anil had taught him to read and write and was
helping him in becoming an educated man. With education, Hari would be able to achieve much more in
life. So, he did not want to hurt Anil by losing his trust.
OR
(1) Think Tank feels that Martians are more beautiful than the people on the Earth. The Earthlings have a
smaller head when compared to the Martians who have great and mighty balloon brain which has made
them cleverer and more intelligent than the Earthlings. Earthlings are fond of eating so they always keep
eating sandwiches. Think Tank mockingly calls the Earth a meaningless place where the people are not so
intelligent and brave as the Martians and himself. So, he decided to capture the Earth.
Think Tank makes guesses that the books are the kind of crude refreshment stand and calls it a sandwich,
or a communication satellite. On Noodle's suggestion, chemical vitamins are taken which enable Omega to
transcribe the code of nursery rhymes on 'Mistress Mary, 'Humpty Dumpty’ and 'The Cat and the Fiddle. All
the poems make Think Tank come to the conclusion that Earthlings have a high level of civilization and that
they have been targeted by them. So, decides to escape from Mars. And they go to Alpha century.

Marking Scheme SAMPLE QUESTION PAPER - 3


Class X Exam (2022-23)
English-Language and Literature (184)

Answer Key – Passage -01


(a) Iv. All of these
(b) (iii) Both (i) & (ii)
(c) (ii) Tradition: modernity
(d) (i) Our youngsters had a vision to make India the best
(e) (ii) B, D
(f) (ii) joint family system
(g) (ii) Blended
(h) (i) child marriage
(i) (i) Policemen have to know all ----------- of different things.
(j) (iv) All of these-
Passage 02 Answers
1. (d) Openly
2. ANS- (a) She was determined to receive a higher education.
3. ANS- a) challenged authority
4. ANS- (d) (1) diligent and (2) resolute
205
5. ANS- (c) (1) and (3)
6. ANS- (d) disillusioned
7. ANS- (a) Her husband was dead.
8. ANS-(c) 11 years
9. ANS- (c) heart broken, one of her daughters
10. ANS-(d) Marie didn’t reveal much facts and mysteries of the physical world.
III. Grammar

i) Error Correction
(a) A the
ii) He wanted to be an English teacher
iii) Have not seen
iv) Fell
v) But
vi) Has
vii) Caught
viii) My mother often says to me that honesty is the best policy.
ix) Attends
x) To – of

IV. Creative Writing Skills


Placing the order letter (05 marks)
Format -01
Content -02
Expression -02
Analytical paragraph Answer (05 marks)
The given pie chart illustrates seven different categories of households living in poverty in the UK in 2002.It
is clearly evident from the pie chart that 26% of the total poverty-stricken households are those of sole
parents. Single people without children account for the second highest proportion with 24%. In contrast to
couples without children that accounts for just 9%, couples with children account for 15% of the poor
households. Single aged persons and aged couples’ proportion for 12% together for poor households.
Overall, 14% of all households in the UK were living under poverty. The younger generation had a greater
poor percentage than their aged counterparts. Couples without children had better economic conditions
than those with children.
SECTION - C
V. Reference to the Context
MCQ (A) Answer Key
1. A) (iv) stealing
b) (ii) Money
c) (ii) he has not stolen anything for a long time
d) (iii) careless
e) (i) Waste
2. i) d) fateful
ii) d) 3 & 5
iii) d) Option 4
iv) b) feels that it’s important that the boy learn an important life lesson, undisturbed.
v) Noun (1) A place on the coast where ships may moor in shelter

Que 06 Answers
1. The young seagull was alone on his ledge because he was the only one who was not gathering the
courage to attempt to fly. All his siblings have learned the art of flying. The young seagull was very afraid
about attempting to fly because he thought that his wings would not support him.
2. For Mandela, courage does not mean the absence of fear but a victory over fear. According to him,
brave men need not be fearless but should be able to conquer fear.
3. Anne justified her being a chatterbox in her essay by explaining that it is due to her mother who
was also very talkative and nobody could do anything about their inherited traits
4. The rain changed into hailstones as a strong wind began to blow and huge hailstones began to fall
along with the rain. All the crops in Lencho’s field got destroyed because of the weather conditions.
206
5. The jungles are vast in space where the tiger could mark his own territory and hunt, rest, and sleep
peacefully.The cage in the zoo cannot be an alternative for the jungle in any sense because of its objective
nature
6. No, Amanda is not an orphan. She says so because she wants to be alone. She wants to roam
around in the street alone and pattern dust with her bare feet. She finds silence 'golden' and freedom
'sweet'.
Answers
1) Ausable is different from other secret agents in more ways than one. He has a small room in the
musty corridor of a gloomy French hotel. It was on the sixth and top floor and it was scarcely the setting for
a romantic adventure. He was extremely fat. In spite of living in Paris for over twenty years, he spoke
French and German with difficulty and had an American accent. Instead of getting messages passed
secretly to him by beautiful girls, he got only a routine telephone call making an appointment. In these
ways, he was different from the conventional notion of a secret agent.
2) Pumphrey is worried about Tricki because he would eat nothing and spend all his time lying on the
rug and panting. He refused to eat his favourite dishes and would have bouts of vomiting. Tricki did not
want to go for walks or do anything at all.
3) He stole every year so that he could buy the rare and expensive books that he loved to collect. Each
year he planned carefully so as to steal enough to last twelve months.
4. Fowler is a writer and he had come to meet Ausable. Fowler's first authentic thrill of the day came when
he saw a man in Ausable's room pointing a pistol towards Ausable and himself.
Long Answers
a) Answers
The young seagull's parents had tried almost everything to help him overcome his fear of flying.
They scolded him, threatened and even left him to starve at the ledge unless he makes an attempt for his
flight.
The young seagull was afraid of flying over the sea because he thought he would drown. So the family
decided to teach him a lesson.
Seagull's mother tore a piece of fish that lay at her feet. She threw it in front of his brothers and sisters and
they made their flights and caught it. She threw a piece in front of young seagull as well.
He was too hungry and starving. This hunger compelled him to attempt his first flight and he flapped his
wings downwards and outwards. He was scared but soon he realised that his wings had automatically
opened wide.
He landed safely on the sea, floated on it and overcame his fear.
He was successful. His family was screaming in joy all around him for his first ever flight
b) Whatever situation life throws at us, it evokes both positive and negative responses. The poet, in
the poem, was in a sorrowful mood. However, when a crow shook down the dust of snow on him, his bitter
mood changed into a more optimistic one. Now, being in a sorrowful mood already, the poet could have got
angry at the crow also. However, the dust of snow made him cheerful to some extent and helped him forget
his sadness. Thus, we see that instead of seeing the negative aspects of a happening, one should see the
positive aspects and ones response may greatly affect the outcome of incidents.
Long Answers
a) in the story ‘A Questions of Trust’, Horace Danby carefully planned his theft, but was outwitted by
another thief ‘The lady in red’. Would you agree that over confidence may prove fatal one day? Discuss.
Answer:
It is true that we keep on planning things in our life. We wish thing would materialise as we had planned but
Man proposes God disposes. Horace Danby was a very careful planner. He made all plans very carefully
before the robbery. He drew picture of wiring of electricity and even took every efforts of befriending pets.
But fate had planned something different. Lady in red turned out to be smarter and duped him. Horace,
without any suspicion, opened the safe without gloves. This mistake landed him in prison. So one lesson
that we learn that we should not trust anyone.
b) Herriot saved Tricki's life by saying to Mrs. Pumphrey that he got too fat and due to that he is ill and
if he will not be cured quickly then he can also loose hi life hearing this Mrs. Pumphrey asked the doctor for
the solution he said her for the surgery and said he will keep him for 14 days hearing this Mrs. Pumphrey
was shocked but she was not having any way so she was forced to send Tricki for the surgery and the
doctor kept the dog for a water diet for two days and after that he started to give him a little amount of food
the doctor knew that Tricki was not I'll but due to taking large amount of food he gained the weight and after
14 days the dog's owner came and took the dog with her

207
Marking Scheme SAMPLE QUESTION PAPER -4
Class X Exam (2022-23)
English-Language and Literature (184)
ANSWER KEY
SECTION A
Passage 1
i. a) Importance of handwriting or any appropriate title
ii. b) anticipation
iii. b) 1, 4, 6
iv. Keystrokes are outdated.
v. b) 2 is the result of 1
vi. reversal of letters, inversion of letters and difficulty in forming of letters
vii. c) 2, 3, 4
viii. i) expressing the thoughts effectively
ii) activating reading networks
ix. a) write legibly
x. Self-esteem

Passage 2
i. Policies and incentives often favouring inefficient and unproductive use of water
iii. looming
vi. i) Lacking the improvement of water management
ii) Poor data collection and assessment
ix. that the way in which India manages its water resources accounts for much of its water woes.
x. False

SECTION B GRAMMAR
i. a) are looking
ii. that he had fallen asleep
iii. c) would
iv. c) Error-some Correction- any
v. b) put up with
vi. esteemed
vii. he wants to buy a black wooden table
viii.
Error Correction
suggests suggest

ix. if he had consulted the doctor that day


x. b) may
xi. a) she must go and consult a good doctor
xii.
Error Correction
the A
Writing Skills
Letter Writing
Format
Sender’s address, Date, Receiver’s address, Subject and Salutation/ Salutation and Subject,Letter,
Complimentary close (largely accepted -Yours truly– editor&Yours sincerely- formal /business)
FORMAT – 1 mark
NOTE-full credit if all aspects included. Partial credit (½ mark) if one-two aspects are missing. No credit if
more than
two aspects are missing.
NOTE FOR GIVEN DESCRIPTORS---Dedicated marks at a level are to be awarded only if ALL descriptors
match. If one
or more descriptors do not match, the marks are awarded at a level lower.
CONTENT – 2 marks
208
● All points included
● Well-developed with sustained clarity
1½ marks
● Almost all points incorporated
● Reasonably well-developed
1 mark
● Some points incorporated
● Fair attempt at developing ideas with some impact on clarity of response
● ORGANISATION OF IDEAS -1 mark
1 mark-- Consistent to frequent display of the listed parameters.
● Highly effective style capable of conveying the ideas convincingly with appropriate layout of a formal
● letter viz. addresses, salutation, subscription, and ending
● Carefully structured content with organised paragraphing presented cohesively.
● Highly effective register (formal tone, tense, and vocabulary), relevant and appropriate sentences
● For conveying the ideas precisely and effectively.
½ mark – Limited display of listed parameters.
● Inconsistent style, expression sometimes awkward, layout barely accurate.
● Poor sequencing of ideas; often sporadically clear and related to the given topic in an attempt to
● maintain a general overall cohesion. Displays disjointed portions, exhibiting a lack of coherence of
● ideas.
● Range of vocabulary is limited but manages to convey, largely, the overall meaning
● and the purpose of the writing. Large portion of vocabulary copied from question.
ACCURACY -1 mark
1 mark
● Spelling, punctuation and grammar consistently/largely accurate, with occasional minor errors, that
● do not impede communication.
● ½ mark Most of the points of the given task not incorporated
● Limited awareness of task development
● Spelling, punctuation and grammar display some errors spread across, causing minor impediments
● to the message communicated.
No credit
Frequent errors in spelling, punctuation, and grammar, impeding communication
SECTION C LITERATURE
V Reference to the Context 10

1.A hovered anxiously in the background, ringing a dozen times a day for the latest bulletins.
i.
B) Statements III and IV

ii. In the above extract, the narrator describes that Tricki had never had such a time in his life. This is
because Tricki got lots of exercise and played with other dogs which he could not get at its home.
1

iii. C) Option (iii)


The Goose pet owners go all-out in protecting their pet. They often limit their time away
from their pets, especially puppies.
iv) The Baggage Handler pet owners love being close to their pets and going on adventures
together. They are always mindful of the pet’s comfort and security.

iv.
B) only 2

v. Why does the narrator describe Tricki being “tramped on and squashed” as joys? 1

OR

209
1.B There’s some demon of contradiction in you today, Ivan Vassilevitch. First you pretend that the
Meadows are yours; now, that Guess is better than Squeezer. I don’t like people who don’t say what they
mean, because you know perfectly well that Squeezer is a hundred times better than your silly Guess.

i. Select the option that correctly captures the usage of the word ‘pretend’ from line 1 of the extract.
1
A) I had no option but to pretend ignorance.
B) Raghav does not pretend to be a psychiatrist
C) This is a pretend train for the children to play in
D)Raman had a pretend pal with whom he talked
ii. Choose the option that lists the set of statements that are TRUE according to the given extract:
1
1. Ivan Vassilevitch had a quarrel with the speaker.
2. Ivan Vassilevitch claims that the Oxen Meadows are his.
3. Ivan Vassilevitch claims that Squeezer is a better dog
4. The speaker’s dog is Guess.
5. Ivan Vassilevitch and the speaker quarrelled more than once.
6. The speaker thinks Squeezer is a silly dog.
7. Ivan Vassilevitch agrees with the speaker on the quality of the two dogs.
A) 1,2,3 B) 1,2,5
C) 3,4,6,7 D) all of them

iii. Complete the analogy by selecting the suitable word from the text. 1
hero: villain :: angel:__devil.

iv. Which of the following is the meaning of the word ‘contradiction’: 1


A) A combination of statements, ideas, or features which are opposed to one another.
B) A situation in which inconsistent elements are present.
C) The statement of a position opposite to one already made.
D) All of the above

v. State the two reasons for the quarrel between the speaker and Ivan Vassilevitch. 1
i) they fought over a small piece of land called oxen meadows
ii) they started fighting over dogs and claimed each of their dog better than the other ones.
2 Attempt ANY ONE of two extracts given. 5

2.A (I am Rapunzel, I have not a care;


life in a tower is tranquil and rare;
I’ll certainly never let down my bright hair!)
(Amanda)

i. What is the poet’s tone in the extract? 1


a) analytical
b) despairing
c) peaceful
d) nervous

ii. Complete the sentence appropriately.


It is clear that Allusion is the poetic device used for ‘I am Rapunzel’ because it is an indirect reference to a
character, Rapunzel, from a fairy tale. 1
iii. Read the statements A and B given below, and choose the option that correctly evaluates these
statements.
Statement A – The figure ‘I’ imagines a less than realistic view of being Rapunzel.
Statement B – The figure ‘I’ does not like the speaker. 1
a) A is true, B is false, according to the extract
b) A is true, B cannot be clearly inferred from the extract.
c) A cannot be clearly inferred from the extract, B is false.
d) A is true and can be inferred from the poem, B is true too.
210
iv. State whether the following statement is TRUE OR FALSE: 1
Amanda wants to be Rapunzel because she feels that life in the tower will be peaceful and unusual.
True

v. The reason behind the above extract written in parenthesis, i.e. () is the above lines are spoken by
Amanda which are in Amanda’s imagination and not in reality. 1

OR
2.B The fog comes
on little cat feet.
It sits looking
over harbour and city
on silent haunches
and then moves on.
(Fog)

i. Carl Sandburg applies qualities of the cat to the fog. Select the most appropriate option from the ones
given below.
1. Cats are known to be faithful and loyal companions, protective of their masters and territory.
2. Cats are stealthy, moving in slow motion at times and they appear to be moving in a
mysterious fashion.
3. Cats like to move on at their own pace and before you know it, they’ve disappeared.
4. Cats are distrustful of strangers and can be jealous and moody. 1
a) option 1,2 and 3
b) option 3 and 4
c) option 2 and 3
d) option 1,3 and 4

ii. Choose the option that lists the most appropriate explanation of fog. 1
A B

C D

iii. The poet has used a poetic device in the given lines. What effect does she wish to create by its use?
1
The fog comes
on little cat feet.
It sits looking

A. mimicry
B. comparison
C. rhythm
D. humour

iv. Complete the sentence appropriately. It is clear that free verse is the poetic device used for the poem
fog because there is no rhyme scheme (Clue: explain how free verse applies here)
1
VI. Answer ANY FOUR of the following in about 40-50 words each. (4×3=12)
i. “Death and suffering are eternal truth; one must accept it as a part of life.” Reflect your views on the
statement with reference to ‘The Sermon at Benaras’. 3
Value Points:
‘Sermon At Benares’ is the story of an unfortunate woman Kisa Gotami. She had lost her only one. In her
grief, she carried the dead body of her son from one place to another. In the end, she came to Lord
Buddha. She needed the medicine that could cure her son. The Buddha asked her to bring him a handful of
mustard seed from a house where no one had lost a child, husband, parent or friend. Kisa Gotami didn’t
find a house where some beloved one had not died in it. She thought to herself that it was the fate of
211
mankind. Death was inevitable. Nobody can avoid dying. The world is afflicted with death and decay. The
wise don’t grieve. `He who has overcome all soon will become free from sorrow, and be blessed.’
ii. A paradoxical statement has been used by the poet describing the distinctive features of a chameleon.
Elucidate the statement with reference to the poem ‘How to Tell Wild Animals’.
Value Points:
z3
The poet tells us that a chameleon is found on a tree. It is a creature which can change its colour according
to its surrounding. It is very difficult to see a chameleon on the tree because it changes its colour according
to the colour of the tree.
iii. Black often symbolises misfortune. However, in the story, ‘The Black Aeroplane’, it depicts hope, faith
and gratitude. Discuss.
he Black Aeroplane' is definitely a mystery story. When the pilot is flying towards his home in Dakota,
suddenly he is surrounded by a storm. As his plane moved inside the dark clouds, his compass stopped
working and he was not able to see anything. Even his radio was not working and he was not able to
contact ground control. Then suddenly, he saw another plane flying next to him and the pilot gesturing to
him to follow. The pilot gladly followed and soon was guided to safety. But as mysteriously as it had arrived,
the plane disappeared. The pilot was not able to locate it and it remained a mystery.
3
iv. Keeping a pet is not all fun but a responsibility too. Comment on the basis of the story ‘Mijbil the Otter’.

Value Points:

We come to know that otters are fun pets to have. They are friendly and fun-loving. They are very intelligent
and can play with toys often inventing their own games. They are very fond of playing in water.
3
v. Validate the given statement with reference to the boy’s loss. ‘Sometimes we have to lose something
precious in order to gain something priceless’. (The Ball Poem) 3
Value Points:
Gain and loss are the two sides of the same coin. Getting, spending and losing things form a natural cycle
of life. The boy is inconsolable at the loss of his ball. Actually, it is not the ordinary ball but his long
association and attachment with it that makes the loss so unbearable. It is like the good sweet days of
childhood that the boy cherishes so much but are lost and gone forever. They will never come back again.
So, what is the remedy? He can bear this loss by understanding the epistemology or nature of the loss. In
this world of material wealth and possessions, it seems that money can buy anything. However, it is a false
conception. Money has its own limitations. Its nature is external. It cannot compensate for the losses that a
person suffers emotionally or internally. No wealth can buy back the ball that has been lost forever.
Similarly, no wealth can buy back the lost childhood. The child will have to move ahead and stand up in life.
He has to stop weeping over his past losses and start living life as it should be lived

VII. Answer ANY TWO of the following in about 40-50 words each. (2×3=6)
. Presence of mind and intelligence is more powerful than a gun. How far is it true in case of Ausable,
the secret agent?
i. Value Points:
Max lay in wait for Ausable to steal the important paper from him. As Fowler observed, apart from the
pointed gun he didn’t look very menacing. Even this proved useless for him as Ausable outwitted him with
his presence of mind and intelligence. Ausable cleverly found out how Max had entered the room and
created an extremely believable story about a non-existent balcony. He kept calm, and, hearing the waiter
knock, declared it to be the police. Max, in an attempt to hide, fell to his death. Thus, Ausable got rid of Max
without moving a muscle.
3
iii. Money can’t make a man as much as education. Elucidate the statement with reference to the
lesson ‘The Thief’s Story’.
Value Points:
iv. The statement stands true in almost all the aspects of life. Money may but us all the luxuries and
fulfil our needs but it cannot buy us knowledge, civilised thinking, skill and abilities to achieve our dreams.
Education lays the platform for all to act upon our goals according to our abilities. Education enables us to
keep up with the fast-moving world. It opens the door to opportunities we do not know even exist. Money,

212
on the other hand, can assist us to a certain level. It can buy us a plan but education gives us the
knowledge of its execution. Just as in the story ‘A Thief’s Story’.
Hari Singh befriends the struggling author ‘Anil’, in the plans of making a steal. Anil, the author is a good
fellow and wants to teach Hari Singh how to read and write. Gradually, he develops trust in Hari Singh. But
Hari Singh has been waiting for the right moment to steal the valuables of Anil. After stealing money from
Anil, Hari Singh tries to go away forever. But some inner voice stops him from doing so. He returned to Anil
because he wants to live a life of respect. This shows that Hari Singh prioritised the chance of being literate
over a few hundred rupees; we must understand that education can help us to achieve whatever we desire.
3
iii. Sending Bholi to school was a ‘blessing in disguise’. Discuss. (Bholi) 3
● Bholi – a slow learner, her face marred by pock marks, stammering child was sent to school
● Mother thought that there was little chance of her getting married as she had ugly face and she
decided to send her school – ‘let the teacher worry about her’.
● Education transformed Bholi completely she grew up confident, bold humble woman.
VIII. Answer ANY ONE of the following in about 100-120 words. (6×1=6)
i. Both Anne and Amanda were teenagers and sensitive girls. Analyse the common character traits of Anne
and Amanda with reference to the texts ‘From the Diary of Anne Frank’ and ‘Amanda’ respectively.
6
● Amanda and Anne Frank are two girls with different perspectives on life.
● While Anne is a teenager who is mature and grounded,
● Amanda is a young eleven-year-old who lives in her fantasy world
● Amanda imagines herself to be an orphan and feels they lead a carefree life without anybody to
question or instruct them. Orphans, according to Amanda, can draw patterns on soft dust, play and wander
aimlessly all day along without a care.
● Amanda wishes she too could do that and live peacefully and on other hand Anne Frank was a
teenage Jewish girl who kept a diary while her family was in hiding from the Nazis during World War II. For
two years, she and seven others lived in a "Secret Annex" in Amsterdam before being discovered and sent
to concentration camps.

ii. Valli was so overcome with sadness to see the dead cow that she lost all enthusiasm. Do you feel the
same way? Compare and contrast the plight of the Tiger captivated in the zoo and the dead cow roaming
freely on the road 6
● The incident of the playful cow, being hit by a fast moving vehicle, indeed disturbed Valli a lot.
● It was a rude shock that the animal that she’d been laughing at a while ago, enjoying fully its
clumsy efforts to dodge the vehicles, lay in the stillness of death.
● Stray animals on the road are definitely a menace and can endanger the safety of many commuters
in addition to their own.
● The municipal corporation must see to it that the cattle are not let loose on the road just because
they are not beneficial to their owners any more.
● Precaution should be taken that speed limits are strictly followed in the larger interest of safety for
all.
● The poem – A tiger in the zoo – talks about the life and mental status of a tiger who is put behind
bars in a cage.
● He is calm and helpless.
● He is strolling inside a limited area in his cage.
● He might be missing his days of freedom.
● He is angry with visitors at the zoo but he prefers to ignore them and does not roar or attack
anyone.

IX. Answer ANY ONE of the following in about 100-120 words. (6×1=6)
i. Horace was a successful thief because he carefully planned his robberies. Should we call him a
successful thief and still appreciate his work? Why or why not? 6
Value Points:
● Yes, as a thief, Horace is successful because he carefully planned his robberies and completes
them well.
● He was living his life as a good and honest citizen.
● However, the wealth he gathered due from his successful robberies did not belong to him.
● By stealing other peoples valuables, he may have become successful but he is actually a criminal.
213
● He may be efficient in conducting his-crimes so that he is successful, but we still cannot appreciate
his work.
ii. “Confession is good for the soul.” A little confession would have changed the life of Loisel. Examine the
above given statement. (The Necklace)

● If Matilda would have confessed to her friend she had lost her necklace, she might have been in
lesser trouble than what she had to face after having replaced the necklace.
● Her friend would have definitely been angry with her. Most likely, she would also have asked
Matilda to replace it and given her the details from where she had bought the necklace and how much it
had cost her.
● Matilda would have thus known that the jewels in the necklace were actually not real diamonds.
● It would have cost her a far lesser amount to replace it.
● Matilda would thus have saved herself and her husband of all the trouble they went through and life
would have been much better and easier for them.

Marking Scheme SAMPLE QUESTION PAPER - 5


Class X Exam (2022-23)
English-Language and Literature (184)
ANSWER KEY M.M. 80

Answer Key – Passage -01


(k) Iv. All of these
(l) (iii) Both (i) & (ii)
(m) (ii) Tradition: modernity
(n) (i) Our youngsters had a vision to make India the best
(o) (ii) B, D
(p) (ii) joint family system
(q) (ii) Blended
(r) (i) child marriage
(s) (i) Policemen have to know all ----------- of different things.
(t) (iv) All of these-
Passage 02 Answers
2. (d) Openly
2. ANS- (a) She was determined to receive a higher education.
3. ANS- a) challenged authority
4. ANS- (d) (1) diligent and (2) resolute
5. ANS- (c) (1) and (3)
6. ANS- (d) disillusioned
7. ANS- (a) Her husband was dead.
8. ANS-(c) 11 years
9. ANS- (c) heart broken, one of her daughters
10. ANS-(d) Marie didn’t reveal much facts and mysteries of the physical world.
III. Grammar

xi) Error Correction


(b) A the
xii) He wanted to be an English teacher
xiii) Have not seen
xiv) Fell
xv) But
xvi) Has
xvii) Caught
xviii) My mother often says to me that honesty is the best policy.
xix) Attends
xx) To – of
214
IV. Creative Writing Skills
Placing the order letter (05 marks)
Format -01
Content -02
Expression -02
Analytical paragraph Answer (05 marks)
The given pie chart illustrates seven different categories of households living in poverty in the UK in 2002.It
is clearly evident from the pie chart that 26% of the total poverty-stricken households are those of sole
parents. Single people without children account for the second highest proportion with 24%. In contrast to
couples without children that accounts for just 9%, couples with children account for 15% of the poor
households. Single aged persons and aged couples’ proportion for 12% together for poor households.
Overall, 14% of all households in the UK were living under poverty. The younger generation had a greater
poor percentage than their aged counterparts. Couples without children had better economic conditions
than those with children.
SECTION - C
V. Reference to the Context
MCQ (A) Answer Key
2. A) (iv) stealing
b) (ii) Money
c) (ii) he has not stolen anything for a long time
d) (iii) careless
e) (i) Waste
2. i) d) fateful
ii) d) 3 & 5
iii) d) Option 4
iv) b) feels that it’s important that the boy learn an important life lesson, undisturbed.
v) Noun (1) A place on the coast where ships may moor in shelter

Que 06 Answers
7. The young seagull was alone on his ledge because he was the only one who was not gathering the
courage to attempt to fly. All his siblings have learned the art of flying. The young seagull was very afraid
about attempting to fly because he thought that his wings would not support him.
8. For Mandela, courage does not mean the absence of fear but a victory over fear. According to him,
brave men need not be fearless but should be able to conquer fear.
9. Anne justified her being a chatterbox in her essay by explaining that it is due to her mother who
was also very talkative and nobody could do anything about their inherited traits
10. The rain changed into hailstones as a strong wind began to blow and huge hailstones began to fall
along with the rain. All the crops in Lencho’s field got destroyed because of the weather conditions.
11. The jungles are vast in space where the tiger could mark his own territory and hunt, rest, and sleep
peacefully. The cage in the zoo cannot be an alternative for the jungle in any sense because of its objective
nature
12. No, Amanda is not an orphan. She says so because she wants to be alone. She wants to roam
around in the street alone and pattern dust with her bare feet. She finds silence 'golden' and freedom
'sweet'.
Answers
3) Ausable is different from other secret agents in more ways than one. He has a small room in the
musty corridor of a gloomy French hotel. It was on the sixth and top floor and it was scarcely the setting for
a romantic adventure. He was extremely fat. In spite of living in Paris for over twenty years, he spoke
French and German with difficulty and had an American accent. Instead of getting messages passed
secretly to him by beautiful girls, he got only a routine telephone call making an appointment. In these
ways, he was different from the conventional notion of a secret agent.
4) Pumphrey is worried about Tricki because he would eat nothing and spend all his time lying on the
rug and panting. He refused to eat his favourite dishes and would have bouts of vomiting. Tricki did not
want to go for walks or do anything at all.
3) He stole every year so that he could buy the rare and expensive books that he loved to collect. Each
year he planned carefully so as to steal enough to last twelve months.

215
4. Fowler is a writer and he had come to meet Ausable. Fowler's first authentic thrill of the day came when
he saw a man in Ausable's room pointing a pistol towards Ausable and himself.
Long Answers
c) Answers
The young seagull's parents had tried almost everything to help him overcome his fear of flying.
They scolded him, threatened and even left him to starve at the ledge unless he tries for his flight.
The young seagull was afraid of flying over the sea because he thought he would drown. So, the family
decided to teach him a lesson.
Seagull's mother tore a piece of fish that lay at her feet. She threw it in front of his brothers and sisters and
they made their flights and caught it. She threw a piece in front of young seagull as well.
He was too hungry and starving. This hunger compelled him to attempt his first flight and he flapped his
wings downwards and outwards. He was scared but soon he realised that his wings had automatically
opened wide.
He landed safely on the sea, floated on it and overcame his fear.
He was successful. His family was screaming in joy all around him for his first ever flight
d) Whatever situation life throws at us, it evokes both positive and negative responses. The poet, in
the poem, was in a sorrowful mood. However, when a crow shook down the dust of snow on him, his bitter
mood changed into a more optimistic one. Now, being in a sorrowful mood already, the poet could have got
angry at the crow also. However, the dust of snow made him cheerful to some extent and helped him forget
his sadness. Thus, we see that instead of seeing the negative aspects of a happening, one should see the
positive aspects and ones response may greatly affect the outcome of incidents.
Long Answers
c) in the story ‘A Questions of Trust’, Horace Danby carefully planned his theft, but was outwitted by
another thief ‘The lady in red’. Would you agree that over confidence may prove fatal one day? Discuss.
Answer:
It is true that we keep on planning things in our life. We wish thing would materialise as we had planned but
Man proposes God disposes. Horace Danby was a very careful planner. He made all plans very carefully
before the robbery. He drew picture of wiring of electricity and even took every efforts of befriending pets.
But fate had planned something different. Lady in red turned out to be smarter and duped him. Horace,
without any suspicion, opened the safe without gloves. This mistake landed him in prison. So one lesson
that we learn that we should not trust anyone.
d) Herriot saved Tricki's life by saying to Mrs. Pumphrey that he got too fat and due to that he is ill and
if he will not be cured quickly then he can also loose hi life hearing this Mrs. Pumphrey asked the doctor for
the solution he said her for the surgery and said he will keep him for 14 days hearing this Mrs. Pumphrey
was shocked but she was not having any way so she was forced to send Tricki for the surgery and the
doctor kept the dog for a water diet for two days and after that he started to give him a little amount of food
the doctor knew that Tricki was not I'll but due to taking large amount of food he gained the weight and after
14 days the dog's owner came and took the dog with her

216
217
218

You might also like